0% found this document useful (0 votes)
1K views460 pages

Combine Clinical Compressed

Uploaded by

Hasan Al Mokdad
Copyright
© © All Rights Reserved
We take content rights seriously. If you suspect this is your content, claim it here.
Available Formats
Download as PDF, TXT or read online on Scribd
0% found this document useful (0 votes)
1K views460 pages

Combine Clinical Compressed

Uploaded by

Hasan Al Mokdad
Copyright
© © All Rights Reserved
We take content rights seriously. If you suspect this is your content, claim it here.
Available Formats
Download as PDF, TXT or read online on Scribd
You are on page 1/ 460

Endodontics

Q1: The labiolingual dimension of maxillary canine pulp chamber is _______its mesiodistal dimension.

A. shorter than

B. wider than

C. equal to

D. similar to

Answer: wider than

Q2: In mandibular incisors, dimensions of the pulp canal remain similar at all levels.

Mandibular central incisor is the smallest tooth in the arch.

A. both statements are true

B. both statements are false

C. the first statement is true, the second is false

D. the first statement is false, the second is true

Answer: the first statement is false, the second is true

Q3: Which of the following are the morphological characteristics of maxillary first premolar?

A. presence of pulp horn under each cusp

B. larger palatal orifice compared to buccal orifice

C. pulp chamber wider buccolingually than mesiodistally

D. all of the above

Answer: all of the above

Q4: Compared to the buccal pulp horn, the lingual pulp horn is larger in the mandibular first premolar and
smaller in the mandibular second premolar.

As the mandibular second premolar has a smaller lingual inclination compared to the mandibular first
premolar, less extension up the buccal cusp is needed to achieve a straight-line access.

A. both statements are true

B. both statements are false

C. the first statement is true, the second is false

D. the second statement is true, the first is false

Answer: the second statement is true, the first is false


Q5: After drawing a line to connect the three main canal orifices, which of the following molars appear to
have formed a molar triangle? Select all that apply.

A. maxillary first molar

B. mandibular first molar

C. mandibular second molar

D. maxillary second molar

Answer: maxillary first molar, maxillary second molar

Q6: The root canal orifices in the mandibular first molar are smaller than those of the mandibular second
molar.

The access cavity in a two-canal mandibular second molar is rectangular and wider mesiodistally than
buccolingually.

A. both statements are true

B. both statements are false

C. the first statement is true, second is false

D. the first statement is false, second is true

Answer: the first statement is false, second is true

Q7: Which of the following is a clinical diagnosis based on subjective and objective findings indicating that
the vital inflamed pulp is incapable of healing and has the following additional descriptors: lingering thermal
pain, spontaneous pain, and referred pain?

A. reversible pulpitis

B. asymptomatic irreversible pulpitis

C. symptomatic irreversible pulpits

D. none of the above

Answer: symptomatic irreversible pulpitis


Q8: Which of the following is an inflammatory reaction to pulpal infection and necrosis characterized by
rapid onset, spontaneous pain, tenderness of the tooth to pressure, pus formation and swelling of
associated tissues?

A. symptomatic apical periodontitis

B. acute apical abscess

C. chronic apical abscess

D. asymptomatic apical periodontitis

Answer: acute apical abscess

Q9: Which of the following are related to vital teeth and usually do not warrant endodontic therapy? Select
all that apply.

A. apical scar

B. cementoma

C. traumatic bone cyst

D. radicular cyst

Answer: cementoma, traumatic bone cyst

Q10: The most superior of all other retrofilling material – mineral trioxide aggregate (MTA) has all of the
following advantages, EXCEPT two. Which TWO are not properties of MTA?

A. radiopaque

B. easy to manipulate

C. hydrophilic

D. biocompatible

E. not toxic

F. short setting time

G. induction of hard tissue formation

Answer: easy to manipulate, short setting time


Q11: The earliest and most common symptom associated with an inflamed pulp is:

A. a dull throbbing pain on mastication

B. sensitivity to hot and/or cold stimuli

C. a persistent feeling of discomfort

D. mild bleeding

E. pain on percussion

Answer: sensitivity to hot and/or cold stimuli

Q12: A phoenix abscess is also known as a:

A. recrudescent abscess

B. granuloma

C. cyst

D. none of the above

Answer: recrudescent abscess

Q13: The chronic apical abscess is an inflammatory reaction to pulpal infection and necrosis characterized
by gradual onset, little or no discomfort, and the intermittent discharge of pus through an associated sinus
tract.

The acute apical abscess is an inflammatory reaction to pulpal infection and necrosis characterized by
rapid onset, spontaneous pain, tenderness of the tooth to pressure, pus formation, and swelling of
associated tissues.

A. both statements are true

B. both statements are false

C. the first statement is true, the second is false

D. the first statement is false, the second is true

Answer: both statements are true

Q14: A patient is diagnosed with symptomatic apical periodontitis and refuses treatment due to fear of
needles. Your statement to the patient should include the fact that:

A. eventually, the acute nature of the lesion will progress into a chronic and nonpainful lesion

B. this lesion can progress into the bone causing osteomyelitis, a more severe condition

C. the apical lesion has been there for years and the tooth needs treatment immediately

D. none of the above

Answer: this lesion can progress into the bone causing osteomyelitis, a more severe condition
Q15: An acute apical abscess will not respond to pulp vitality tests.

An acute apical abscess is only observed in association with a necrotic pulp.

A. both statements are true

B. both statements are false

C. the first statement is true, the second is false

D. the first statement is false, the second is true

Answer: both statements are true

Q16: Appearance of a “pink spot” in the crown portion of a tooth is indicative of which of the following
pathologies?

A. cracked tooth syndrome

B. internal resorption

C. irreversible pulpitis

D. external resorption

Answer: internal resorption

Q17: A 7-year-old boy arrives at the office with a complaint that tooth #8 is draining pus into his mouth.
The tooth had been traumatized earlier. The vitality tests reveal no response. What is the treatment of
choice? Select all that apply.

A. extraction

B. apexogenesis/pulpotomy

C. pulpectomy

D. periodontal surgery to remove sinus tract

E. it is only necessary to give the child analgesics and antibiotics for pain and infection

F. apexification

Answer: pulpectomy, apexification


Q18: All of the following statements regarding adjuncts to endodontic treatment are true EXCEPT one.
Which one is the EXCEPTION?

A. transplanted teeth with partial root development have a better prognosis than do those with
developed roots

B. orthodontic extrusion is commonly indicated prior to implant placement

C. intentional replantation is a viable alternative to endodontic surgery

D. a major disadvantage of endodontic implants is the lack of an apical seal

Answer: intentional replantation is a viable alternative to endodontic surgery

Q19: Which of the following is the most characteristic radiographic evidence of a vertical root fracture?

A. a persistent periodontal defect

B. a radiolucent halo surrounding the root of the fracture

C. a radiopaque lesion at the sight of the fracture

D. a visible fracture when transillumination is used

Answer: a radiolucent halo surrounding the root of the fracture

Q20: A patient complains of a slight tooth ache that has been "on and off" for a week. The tooth in question
#18. Which of the following teeth would be optimum to use as a baseline?

A. #19 virgin, #31 occlusal sealant

B. #15 primary cavitation on occlusal and #19 virgin

C. #3 FGC and #31 virgin

D. #30 occlusal amalgam and #31 virgin

Answer: #19 virgin, #31 occlusal sealant

Q21: The main concept of the cone shift technique is that as the vertical or horizontal angulations of the x-
ray tube head changes, the object buccal or closest to the tube head moves to the _________ side of the
radiograph when compared to the lingual object.

A. same

B. opposite

Answer: opposite
Q22: Your practice is involved with a local minor hockey team, the Millwrights. A player gets hit with a
stick, and his central incisors are intruded. Which of the following is the least useful examination
procedure?

A. soft tissue exam

B. hard tissue exam

C. radiograph

D. vitality test

E. percussion test

Answer: vitality test

Q23: ________ require endodontic treatment more often than any other tooth, while________ have the
highest endodontic failure rate.

A. mandibular first molars, maxillary first molars

B. mandibular first molars, maxillary second molars

C. maxillary second molars, mandibular first molars

D. maxillary first molars, mandibular first molars

Answer: mandibular first molars, maxillary first molars

Q24: All of the following are correct associations EXCEPT one. Which one is the EXCEPTION?

A. maxillary incisors most often refer pain to the forehead region

B. maxillary second premolars most often refer pain to the temporal region

C. maxillary molars most often refer pain to the ear

D. mandibular molars most often refer pain to posterior region of the neck

Answer: maxillary molars most often refer pain to the ear

Q25: Which of the following teeth is most likely to have two canals (in fact, it has two canals most of the
time)?

A. tooth #4

B. tooth #12

C. tooth #20

D. tooth #28

Answer: tooth #12 – maxillary first premolars almost always have two canals

Q26: One year after performing endodontic treatment on tooth #3, you take a new periapical radiograph
and notice that there is still a lesion present. What is the most likely problem?

A. you failed to locate a second mesiobuccal canal

B. you failed to locate a second distobuccal canal

C. you failed to locate a second palatal canal

D. nothing, it takes more than 12 months for the bone to heal

Answer: you failed to locate a second mesiobuccal canal

Q27: Match the teeth on the left with their respective characteristics on the right.

• 1. maxillary canine
• 2. maxillary second molar
• 3. mandibular first molar
• 4. maxillary first molar
• 5. maxillary lateral incisor
• A. palatal root has the largest diameter and is the longest
• B. mesial root almost always has two canals
• C. largest labiolingual root dimension
• D. tooth that is usually peg-shaped
• E. more closely related to the maxillary sinus

Answer: 1. C, 2. E, 3. B, 4. A, 5. D

Q28: Approximately what percentage of mandibular first premolars may have two canals with two apical
foramina?

A. 5%

B. 20%

C. 45%

D. 65%

Answer: 20%
Q29: A mandibular canine typically requires a triangular access preparation. The access should be
directed slightly toward the lingual surface due to slight labial axial inclination of the crown.

A. both statements are true

B. both statements are false

C. the first statement is true, the second is false

D. the first statement is false, the second is true

Answer: the first statement is false, the second is true

Q30: Which of the following teeth is most likely to have a curved root?

A. maxillary central incisor

B. maxillary lateral incisor

C. maxillary canine

D. mandibular central incisor

Answer: maxillary lateral incisor

Q31: While doing a vital pulpotomy on a young, immature permanent tooth, the hemorrhage after pulp
amputation could not be controlled with cotton pellets, even after several minutes. What is the next step in
completing this treatment?

A. control the hemorrhage with hemostatic agents

B. apply formocresol with cotton pellets at the amputation site

C. irrigate the canal with sodium hypochlorite then apply calcium hydroxide

D. perform the amputation at a more apical level

E. stop the procedure and close the tooth with an interim restoration

F. all of the above

Answer: perform the amputation at a more apical level

Q32: Which of the following situations offer better success for pulp capping? Select all that apply.

A. accidental exposure of the pulp

B. pulp of a middle-aged person

C. carious exposure of the pulp

D. pulp of a young child

Answer: accidental exposure of the pulp, pulp of a young child


Q33: In which of the following scenarios would you consider using solvent-softened custom cones? Select
all that apply.

A. lack of an apical stop

B. an abnormally large apical portion of the canal

C. an irregular apical portion of the canal

D. after an apexification procedure

Answer: lack of an apical stop, an abnormally large apical portion of the canal, an irregular apical
portion of the canal, after an apexification procedure

Q34: During the master cone fitting procedure in the endodontic treatment of a patient's tooth, the patient
says he has a "sharp shooting pain in the same tooth that ached earlier." What should be your response
and why?

A. continue with obturation, the anesthetic is simply wearing off

B. continue with obturation, this is a normal complaint during this part of the procedure

C. consider looking for an accessory canal and refiling, there is likely pulpal tissue that has not been
properly debrided

D. irrigate further, the sodium hypochlorite should take care of this problem

E. temporize the tooth and obturate at a later date

Answer: consider looking for an accessory canal and refiling, there is likely pulpal tissue that has
not been properly debrided

Q35: All of the following are suggested as irrigants during root canal therapy EXCEPT one. Which one is
the EXCEPTION?

A. urea peroxide (Gly-Oxide)

B. hydrogen peroxide

C. sodium hypochlorite

D. calcium hydroxide
Answer: calcium hydroxide
Q36: You are retreating a previously root canal treated tooth. Which of the following might you possibly
use? Select all that apply.

A. rotary files

B. chloroform

C. glass bead sterilizer

D. ultrasonic

E. heated instruments

Answer: rotary files, chloroform, ultrasonic, heated instruments

Q37: Which of the following statements regarding ethylene diamine tetra-acetic acid (EDTA) are true?
Select all that apply.

A. it is a chelating agent with the capability to remove the mineralized portion of the smear layer

B. it can decalcify up to a 50 mm thin layer of the root canal wall

C. it is also an excellent irrigation solution

D. normally used in a concentration of 17%

E. RC-Prep and EDTAC are other preparations of EDTA

F. it can be used in place of NaOCl

Answer: it is a chelating agent with the capability to remove the mineralized portion of the smear
layer, it can decalcify up to a 50 mm thin layer of the root canal wall, normally used in a
concentration of 17%, RC-Prep and EDTAC are other preparations of EDTA

Q38: The most acceptable method to achieve adequate root canal debridement is:

• to obtain clean shavings of the canal

• to attain a clean irrigating solution

• to achieve glassy smooth walls of the canal

• all of the above criteria are reliable

• none of the above criteria is acceptable

Answer: to achieve glassy smooth walls of the canal


Q39: While cleaning and shaping the canal, an instrument separates in the canal. As you attempt to
retrieve it, the broken instrument passes partially through the apex, thus partly protruding into the
periapical lesion. How do you manage this case?

A. use a smaller H file to bypass it and try retrieving it

B. use Gates Glidden drills to widen the canal and then try retrieving it

C. raise a flap and remove the instrument surgically followed by filling the canal with gutta-percha

D. extract the tooth as irreparable damage has occurred to the apex

E. just inform the patient, fill the canal with gutta-percha, and monitor

Answer: raise a flap and remove the instrument surgically followed by filling the canal with gutta-
percha

Q40: Which of the following are acceptable methods to clean and shape a canal using nickel titanium
instruments?

A. push and pull stroke

B. reaming motion

C. engine-driven rotary motion

D. all of the above

Answer: all of the above

Q41: The major advantage of zinc oxide-eugenol based sealer types is:

A. non-staining property

B. fast setting time

C. adhesion

D. insolubility

E. long history of successful usage

Answer: long history of successful usage

Q42: Which tooth is IMPROPERLY matched with the reason for difficulty of its access preparation?

A. maxillary first premolar - mesial concavity

B. maxillary molar - proximity of canals to distobuccal line angle

C. mandibular molar - mesiolingual tilt of tooth

D. mandibular incisor - narrow mesial distal dimension

Answer: maxillary molar - proximity of canals to distobuccal line angle


Q43: The action of using a file often dictates the shape of the canal.
A reaming action produces a canal that is relatively round in shape.

A. both statements are true

B. both statements are false

C. the first statement is true, the second is false

D. the first statement is false, the second is true

Answer: both statements are true

Q44: Which of the following statements are true regarding files? Select all that apply.

A. broaches can be used for canal enlargement

B. K-type files can be machined or twisted

C. significant apical pressure is needed when using a broach

D. stainless steel files are less flexible than NiTi files

Answer: K-type files can be machined or twisted, stainless steel files are less flexible than NiTi files

Q45: All of the following cells characterize the cellular response at the onset of chronic pulpal inflammation
EXCEPT one. Which one is the EXCEPTION?

A. plasma cells

B. macrophages

C. lymphocytes

D. polymorphonuclear (PMN) leukocytes

Answer: polymorphonuclear (PMN) leukocytes

Q46: A patient presents with all the characteristics of pulpal pathosis. Your assistant hands you an x-ray
that shows no evidence of any restoration or caries. At first you don't believe that the x-ray is from the right
patient, but it is. This scenario is pathognomonic of:

A. condensing osteitis

B. a vertical fracture of the tooth

C. periodontal abscess

D. secondary occlusal trauma

Answer: a vertical fracture of the tooth


Q47: Hydrogen peroxide is a key ingredient to internal and external bleaching.

The walking bleach technique requires the sodium perborate to be changed everyday.

A. both statements are true

B. both statements are false

C. the first statement is true, the second is false

D. the first statement is false, the second is true

Answer: the first statement is true, the second is false

Q48: Tooth #9 requires root-end surgery. Which flap design is generally NOT indicated?

A. a submarginal curved flap (semilunar)

B. a submarginal scalloped flap (Ochsenbein-Luebke)

C. a full mucoperiosteal flap (triangular, rectangular, trapezoidal, horizontal)

D. none of the above

Answer: a submarginal curved flap (semilunar)

Q49: In which of the following cases could a dentist choose not to perform root canal therapy even when it
is advised?

A. on a nonrestorable tooth

B. on a periodontally insufficient tooth

C. on a tooth with a vertical root fracture

D. on an asymptomatic tooth with a calcified chamber

E. on a tooth that has massive external resorption

Answer: on an asymptomatic tooth with a calcified chamber

Q50: A periodontal probing defect that may not be managed by endodontic treatment alone is:

A. a conical shaped probing

B. a narrow sinus tract type probing

C. a blow-out type probing

D. none of the above

Answer: a conical shaped probing — this is typical primarily of a periodontal problem


Q51: Regarding the restoration of endodontically treated teeth, all of the following are generally believed to
be true EXCEPT one. Which one is the EXCEPTION?

A. a major disadvantage of posts/dowels is that they do not reinforce the tooth structure, in fact, they
weaken it

B. all post designs are predisposed to leakage

C. at least 4 mm of gutta-percha must remain to preserve the apical seal

D. threaded screw posts are preferred over parallel-sided and tapered posts

E. pins add to stresses and microfractures in dentin and should not be used

F. cusps adjacent to lost marginal ridges should be restored with an onlay

Answer: threaded screw posts are preferred over parallel-sided and tapered posts

Q52: Retreating a tooth with a post is the most common reason for an apicoectomy and retrograde filling.

Whenever a reverse filling procedure is to be used, apicoectomy is mandatory to provide a table into which
the preparation and filling will be placed.

A. both statements are true

B. both statements are false

C. the first statement is true, the second is false

D. the first statement is false, the second is true

Answer: both statements are true

Q53: Endodontic procedures involve taking multiple radiographs. How should you protect yourself or your
staff while taking radiographs if there is no barrier available to stand behind?

A. stand at least 4 feet away anywhere around the patient

B. stand at least 5 feet away exactly opposite the x-ray beam source

C. stand at least 6 feet away and in the area that lies between 90 and 135 degrees to x-ray beam

D. stand at least 7 feet away and in the area that lies between 60 and 90 degrees to x-ray beam

E. never take an x-ray without a barrier

Answer: stand at least 6 feet away and in the area that lies between 90 and 135 degrees to x-ray
beam
Q54: Most bacteria in endodontic infections are strict aerobes.

The diversity of polymicrobial endodontic infections has been well established isolating anywhere from 3 to
12 species of microbes in the majority of endodontic infections.

A. both statements are true

B. both statements are false

C. the first statement is true, the second is false

D. the first statement is false, the second is true

Answer: the first statement is false, the second is true

Q55: During a routine radiographic evaluation, you notice bone loss extending from the cementoenamel
junction to the apex of tooth #21. Further evaluation reveals that probing depths are above normal limits all
around the tooth. However, at one point, the probe drops precipitously to an even greater depth. Vitality
test is negative. This patient may require:

A. extensive periodontal treatment followed by vitality reassessment

B. endodontic treatment only

C. endodontic treatment followed by periodontic treatment

D. root-end surgery

E. periodontic treatment followed by endodontic treatment

Answer: endodontic treatment followed by periodontic treatment

Q56: Which of the following are NOT found in the pulp?

A. reticulin fibers

B. collagen fibers

C. unmyelinated nerve fibers

D. myelinated nerve fibers

E. proprioceptor nerve fibers

Answer: proprioceptor nerve fibers


Q57: The absence of which layer of dentin predisposes it to internal resorption by cells present in the
pulp?

A. mantle dentin

B. circumpulpal dentin

C. predentin

D. secondary dentin

E. tertiary dentin

Answer: predentin

Q58: The _______________ in the apical portion of the pulp helps to form the pulp into a semisolid mass,
facilitating a ______________.

A. collagen, pulpectomy

B. network of capillaries and nerves, pulpectomy

C. collagen, pulpotomy

D. network of capillaries and nerves, pulpotomy

Answer: collagen, pulpectomy

Q59: The cervical cross-section of the pulp cavity below represents which tooth?

A. the permanent maxillary right first molar

B. the permanent maxillary right second molar

C. the permanent maxillary right third molar

D. the permanent maxillary right first premolar

Answer: the permanent maxillary right second molar

Q60: The cervical cross section of the pulp cavity below represents which tooth?
A. the permanent maxillary right first premolar

B. the permanent maxillary right second premolar

C. the permanent maxillary right first molar

D. the permanent maxillary right third molar

Answer: the permanent maxillary right first premolar

Q61: The cervical cross-section of the pulp cavity below represents which tooth?

A. the permanent maxillary right first premolar

B. the permanent maxillary right canine

C. the permanent maxillary right lateral incisor

D. the permanent maxillary right second premolar

Answer: the permanent maxillary right canine

Q62: The cervical cross-section of the pulp cavity below represents which tooth?

A. the permanent mandibular right third molar

B. the permanent mandibular right second molar

C. the permanent mandibular right second premolar

D. the permanent mandibular right first molar

Answer: the permanent mandibular right first molar

Q63: The cervical cross section of the pulp cavity below represents which tooth?

A. the permanent mandibular right canine

B. the permanent mandibular right lateral incisor

C. the permanent mandibular right first premolar

D. the permanent mandibular right second premolar


Answer: the permanent mandibular right second premolar

Q64: The cervical cross-section of the pulp cavity below represents which tooth?

A. the permanent maxillary right canine

B. the permanent maxillary right first premolar

C. the permanent maxillary right central incisor

D. the permanent maxillary right second premolar

Answer: the permanent maxillary right central incisor

Q65: The cervical cross-section of the pulp cavity below represents which tooth?

A. the permanent maxillary right third molar

B. the permanent maxillary right second molar

C. the permanent maxillary right first premolar

D. the permanent maxillary right first molar

Answer: the permanent maxillary right third molar

Q66: The cervical cross-section of the pulp cavity below represents which tooth?
A. the permanent maxillary right central incisor

B. the permanent maxillary right lateral incisor

C. the permanent maxillary right canine

D. the permanent maxillary right first premolar

Answer: the permanent maxillary right lateral incisor

Q67: An emergency patient walks into your office with a chief complaint of an avulsed maxillary central
incisor following a game of basketball less than thirty minutes ago. Which of the following actions would be
contraindicated? Select all that apply.

A. immediate re-implantation of the avulsed tooth followed by placement of a flexible splint for 7-10 days

B. soaking the tooth in neutral pH distilled water solution prior to re-implantation

C. thorough curettage of socket prior to re-implantation to remove any debris

D. beginning a root canal therapy ten days after re-implantation

Answer: Soaking the tooth in neutral pH distilled water solution prior to re-implantation, Thorough
curettage of socket prior to re-implantation to remove any debris

Q68: Which of the following situations may be indications for intentional replantation? Select all that apply.

A. when routine endodontic therapy is impractical

B. when a canal is obstructed

C. when perforating internal or external resorption is present

D. when previous treatment has failed

E. All of the above

Answer: All of the above


Q69: The main benefit of primary incisor replantation is:

A. maintenance of a normal anterior dentition

B. to relieve parental guilt

C. to maintain child's self-esteem

D. to maintain child's social acceptance

Answer: maintenance of a normal anterior dentition

Q70: Internal resorption of a tooth is generally believed to be caused by inflammation due to an infected
coronal pulp.

This condition is frequently precipitated by traumatic injury to the tooth.

A. both statements are true


B. both statements are false

C. the first statement is true, the second is false

D. the first statement is false, the second is true

Answer: both statements are true

Q71: "The external resorption in which an infected pulp may further complicate the resorptive process" is
termed as:

A. surface resorption

B. inflammatory resorption

C. replacement resorption

Answer: inflammatory resorption

Q72: Which of the following is NOT a key feature of replacement resorption?

A. lack of mobility

B. lack of PDL on x-ray

C. pink appearance

D. infraocclusion

Answer: pink appearance


Scanned with CamScanner
Scanned with CamScanner
Scanned with CamScanner
Scanned with CamScanner
Scanned with CamScanner
Scanned with CamScanner
Scanned with CamScanner
Scanned with CamScanner
Scanned with CamScanner
Scanned with CamScanner
Scanned with CamScanner
Scanned with CamScanner
Scanned with CamScanner
Scanned with CamScanner
Operative Dentistry

Q1: All Class III lesions should be filled with composite resin, because they are esthetically important.

A. both the statement and the reason are correct and related

B. both the statement and the reason are correct but not related

C. the statement is correct, but the reason is not

D. the statement is not correct, but the reason is correct

E. neither the statement nor the reason is correct

Answer: neither the statement nor the reason is correct

Q2: What is the ideal depth of an amalgam preparation and dentin remaining for adequate pulpal
insulation?

A. 1.0 mm into dentin, 2.0 mm dentin remaining

B. 2.0 mm into dentin, 1.0 mm dentin remaining

C. 0.5 mm into dentin, 2.0 mm dentin remaining

D. 2.0 mm into dentin, 0.5 mm dentin remaining

Answer: 0.5 mm into dentin, 2.0 mm dentin remaining

Q3: A patient comes in claiming that their holistic doctor told him that he has an allergy to mercury and
needs only white fillings. Your best response is:

A. there is no such thing as a mercury allergy; plus, there is no mercury in amalgam fillings

B. you might have mercury allergy, but that is very rare; plus, there is no mercury in amalgam fillings

C. there is no such thing as a mercury allergy; plus, with proper isolation and technique, your exposure
will be minimal

D. you might have a mercury allergy, but that is very rare; plus, with proper isolation and technique your
exposure will be minimal

Answer: you might have a mercury allergy, but that is very rare; plus, with proper isolation and
technique your exposure will be minimal

Q4: A patient presents to your office with interproximal caries involving #4 disto-occlusal. The decision is
made to do a Class II Inlay instead of a traditional amalgam restoration. The inlay preparation will include all
of the following features EXCEPT one. Which one is the EXCEPTION?

A. sharp Internal line angles

B. slightly divergent walls occlusally

C. proximal retentive grooves

D. reverse bevel at the axial gingival line angle


Answer: proximal retentive grooves
Q5: You would prefer your assistant overtriturate the amalgam rather than undertriturate it.

This is because an undertriturated amalgam has less strength, faster corrosion, and a rougher finished
surface.

A. both statements are true

B. both statements are false

C. the first statement is true, the second is false

D. the first statement is false, the second is true

Answer: both statements are true

Q6: Class V amalgam restorations rarely require retentive grooves, but if they are used, they are placed at
the incisoaxial and gingivoaxial line angles.

A. both statements are true

B. both statements are false

C. the first statement is true, the second is false

D. the first statement is false, the second is true

Answer: the first statement is false, the second is true

Q7: Delayed expansion of amalgam restorations is associated with which two factors?

A. insufficient trituration and condensation

B. high residual mercury

C. the contamination of the amalgam by moisture during trituration and condensation

D. the failure to use a cavity varnish

Answer: insufficient trituration and condensation, the contamination of the amalgam by moisture
during trituration and condensation

Q8: Proper condensation and carving makes an amalgam restoration stronger because it removes the
mercury-rich gamma-two matrix.

A. both the statement and the reason are correct and related

B. both the statement and the reason are correct but not related

C. the statement is correct, but the reason is not

D. the statement is not correct, but the reason is correct

E. neither the statement nor the reason is correct

Answer: both the statement and the reason are correct and related
Q9: A high-copper amalgam is preferred over a lower-copper amalgam due to the undesirable clinical
properties that present in which phase of the reaction?

A. gamma

B. gamma-one

C. gamma-two

D. gamma-three

Answer: gamma-two

Q10: Which of the following properties would not be beneficial to the retention and resistance of a Class II
amalgam preparation?

A. occlusal dovetail

B. convergent buccal and lingual walls

C. retentive proximal grooves

D. axiogingival line angle greater than 90 degrees

Answer: axiogingival line angle greater than 90 degrees

Q11: Beveling the gingival cavosurface margin of the proximal box of a Class II amalgam preparation on a
permanent tooth:

A. should result in a long bevel

B. is contraindicated because of the low edge strength of amalgam

C. should remove unsupported enamel that may fracture

D. is unnecessary since the tooth structure in this area is strong

Answer: should remove unsupported enamel which may fracture

Q12: Which tooth requires special attention when preparing the occlusal aspect for a restoration?

A. mandibular first bicuspid

B. mandibular second bicuspid

C. maxillary first molar

D. maxillary first bicuspid

Answer: mandibular first bicuspid


Q13: All of the following correctly describe the ideal preparation for a Class I amalgam EXCEPT one. Which
one is the EXCEPTION?

A. convergent buccal and lingual walls

B. pulpal floor extending 0.5 mm into dentin

C. divergent mesial and distal walls 1 mm away from proximal surfaces

D. rounded internal line angles

Answer: divergent mesial and distal walls 1 mm away from proximal surfaces

Q14: The matrix band should be removed after condensation of the amalgam but prior to the final carving of
the restoration. This is because the wedge compensates for the thickness of the matrix band.

A. both the statement and the reason are correct and related

B. both the statement and the reason are correct but not related

C. the statement is correct, but the reason is not

D. the statement is not correct, but the reason is correct

E. neither the statement nor the reason is correct

Answer: both the statement and the reason are correct but not related

Q15: The diagonal slot opening on the Tofflemire matrix retainer (also called the Universal matrix system) is
always placed facing the gingiva. This:

A. permits easy separation of the retainer from the band in an occlusal direction

B. allows for better contour of band to tooth

C. allows for easier wedge placement

D. is less harmful on the gingiva

Answer: permits easy separation of the retainer from the band in an occlusal direction

Q16: Which of the following are true statements regarding the polishing of amalgam? Select all that apply.

A. it reduces marginal discrepancy

B. it should be done about 10 minutes after placement

C. occlusion should not be checked prior to polishing

D. it prevents tarnishing of the restoration

E. it improves the appearance of the restoration

F. it should be done with a dry polishing powder


Answer: it reduces marginal discrepancy, it prevents tarnishing of the restoration, it improves the
appearance of the restoration
Q17: All of the following are true concerning a Class V amalgam preparation EXCEPT one. Which one is
the EXCEPTION?

A. the outline form is determined primarily by the location of the free gingival margin

B. the mesial, distal, gingival, and incisal walls of the cavity preparation diverge outward

C. the retention form is provided by the gingival retention groove along the gingivo-axial line angle and an
incisal retention groove along the incisoaxial line angle

D. a cervical clamp is usually necessary to retract gingival tissues

Answer: the outline form is determined primarily by the location of the free gingival margin

Q18: In regard to the selection and placement of amalgam restorations, all of the following are true
EXCEPT one. Which one is the EXCEPTION?

A. high-copper amalgams are preferred over low-copper amalgams due to elimination of the gamma two
phase

B. increased trituration time will increase compressive strength and decrease setting expansion compared
to decreased trituration time

C. ideal mercury content should range between 55% and 60%

D. tensile strength is 1/5 to 1/8 the compressive strength of set amalgam

E. the smaller the particle size the less expansion

Answer: ideal mercury content should range between 55%-60%

Q19: Amalgam restorations require an obtuse cavosurface margin because amalgam is a brittle material.

A. both the statement and the reason are correct and related

B. both the statement and the reason are correct but not related

C. the statement is correct, but the reason is not

D. the statement is not correct, but the reason is correct

E. neither the statement nor the reason is correct

Answer: the statement is not correct, but the reason is correct

Q20: High copper amalgams reduce gamma-two formation and marginal breakdown.

Unreacted AgSn gamma phase is the strongest and prevalent in triturated amalgam.

A. both statements are true

B. both statements are false

C. the first statement is true, the second is false


D. the first statement is false, the second is true

Answer: Both statements are true


Q21: Which of the following statements are true regarding creep?
Select all that apply.

A. creep is a process that happens over time

B. creep gradually increases the marginal integrity of an amalgam restoration

C. undertrituration tends to increase the creep rate

D. overtrituration tends to decrease the creep rate

E. increasing the condensation pressure decreases the creep rate

Answer: creep is a process that happens over time, undertrituration tends to increase the creep
rate, increasing the condensation pressure decreases the creep

Q22: There is no free mercury in triturated amalgam because trituration causes the alloy to dissolve in the
mercury.

A. both the statement and the reason are correct and related

B. both the statement and the reason are correct but not related

C. the statement is correct, but the reason is not

D. the statement is not correct, but the reason is correct

E. neither the statement nor the reason is correct

Answer: the statement is correct, but the reason is not

Q23: Please rank the following in order of application thickness and consider which would be used to
replace a large amount of dentin that has been destroyed.

A. base
B. cavity liner
C. cement

Answer: cavity liner, cement, base

Q24: Which of the following statements are true regarding glass ionomer cements? Select all that apply.

A. release fluoride

B. good chemical adhesion

C. good thermal insulator

D. thermal expansion similar to tooth

E. high solubility after initial setting


Answer: release fluoride, good chemical adhesion, good thermal insulator, thermal expansion
similar to tooth

Q25: Glass ionomer cements generally contain fluoro-alumino-silica powder.

The fluoro-alumino-silica is the portion responsible for one of the major advantages of glass ionomer.

A. both statements are true

B. both statements are false

C. the first statement is true, the second is false

D. the first statement is false, the second is true

Answer: both statements are true

Q26: ZOE cements make good temporary sedative restorations because their pH is very basic.

A. both the statement and the reason are correct and related

B. both the statement and the reason are correct but not related

C. the statement is correct, but the reason is not

D. the statement is not correct, but the reason is correct

E. neither the statement nor the reason is correct

Answer: the statement is correct, but the reason is not

Q27: IRM (Intermediate Restorative Material) will interfere with subsequent placement of a resin filling. This
is because IRM is a form of zinc oxide-eugenol.

A. both the statement and the reason are correct and related

B. both the statement and the reason are correct but not related

C. the statement is correct, but the reason is not

D. the statement is not correct, but the reason is correct

E. neither the statement nor the reason is correct

Answer: both the statement and the reason are correct and related

Q28: Zinc phosphate cement can cause irreversible pulpal damage because it shrinks slightly on setting.

A. both the statement and the reason are correct and related

B. both the statement and the reason are correct but not related

C. the statement is correct, but the reason is not

D. the statement is not correct, but the reason is correct

E. neither the statement nor the reason is correct


Answer: both the statement and the reason are correct but not related

Q29: Zinc phosphate cements should be mixed on a cool glass slab, adding a small amount of powder to
the liquid every 20 seconds. This is done to gain which of the following advantages.
Select all that apply.

A. stronger final set

B. lower solubility

C. greater viscosity

Answer: stronger final set, lower solubility

Q30: A high caries risk patient with multiple anterior class five lesions presents to your office. All of the
following would be reasons to choose a glass ionomer over composite EXCEPT one. Which one is the
EXCEPTION?

A. fluoride-releasing capability

B. chemical adherence to tooth structure

C. superior finishing potential

D. acceptable esthetic result

Answer: superior finishing potential

Q31: What does the glossy appearance of zinc polycarboxylate cement signify?

A. the presence of insufficient carboxyl groups

B. the presence of free carboxylic acid groups

C. an increased powder–liquid ratio

D. none of the above

Answer: the presence of free carboxylic acid groups

Q32: All of the following are advantages for using a zinc polycarboxylate cement over a traditional zinc
phosphate cement EXCEPT one. Which one is the EXCEPTION?

A. higher tensile strength

B. chemical bonding to tooth structure

C. thin film thickness

D. low pulpal irritation

Answer: thin film thickness


Q33: When using a zinc phosphate cement you place the varnish first. This is because zinc phosphate
cements are not biocompatible.

A. both the statement and the reason are correct and related

B. both the statement and the reason are correct but not related

C. the statement is correct, but the reason is not

D. the statement is not correct, but the reason is correct

E. neither the statement nor the reason is correct

Answer: both the statement and the reason are correct and related

Q34: A/An ___________ base is a base that is typically placed over a calcium hydroxide base that has
been placed over a pulp exposure.

A. primary base

B. secondary base

C. direct base

D. indirect base

Answer: secondary base

Q35: Solution liners should not be placed under composite restorations because composites do not require
the pulpal protection.

A. both the statement and the reason are correct and related

B. both the statement and the reason are correct but not related

C. the statement is correct, but the reason is not

D. the statement is not correct, but the reason is correct

E. neither the statement nor the reason is correct

Answer: the statement is correct, but the reason is not

Q36: Suspension liners, for example calcium hydroxide, harden intraorally by the:

A. chemical reaction of acids and bases

B. chemical reaction involving polymerization

C. chemical reaction involving chelation

D. physical reaction of drying

E. physical reaction of a sol-gel transformation

Answer: physical reaction of drying


Q37: When removing caries, which of the following layers of dentin are affected, but not infected, and
therefore do not need to be removed?
Select all that apply.

A. turbid dentin

B. infected dentin

C. transparent dentin

D. normal dentin

E. subtransparent dentin

Answer: transparent dentin, subtransparent

Q38: The rate of senile caries is increasing, in part, because of the increase in gingival recession.

A. both the statement and the reason are correct and related

B. both the statement and the reason are correct but not related

C. the statement is correct, but the reason is not

D. the statement is not correct, but the reason is correct

E. neither the statement nor the reason is correct

Answer: both the statement and the reason are correct and related

Q39: Chronic caries is characterized by which of the following.


Select all that apply.

A. pain is common

B. slowly progressing or arrested

C. common in adults

D. entrance to the lesion is small

E. lesion is deep and narrow

Answer: slowly progressing or arrested, common in adults

Q40: _________ is degraded by Streptococcus mutans into ________ and ________, thereby causing
caries initiation and progression.

A. sucrose, glucans, lactic acid

B. glucose, glucans, lactic acid

C. sucrose, glucose, acetic acid

D. glucose, sucrose, acetic acid


Answer: sucrose, glucans, lactic acid
Q41: The initiation of caries requires four entities; choose the four entities from the following choices.

A. host

B. bacteria

C. carbohydrates

D. saliva

E. time

Answer: host, bacteria, carbohydrates, time

Q42: There is abundant evidence that the initiation of dental caries requires a high proportion of:

A. Lactobacillus within saliva

B. Streptococcus mutans within dental plaque

C. Lactobacillus within dental plaque

D. Streptococcus mutans within saliva

Answer: Streptococcus mutans within dental plaque

Q43: Streptococcus mutans are ________ and therefore ________.

A. acidogenic, cariogenic

B. aciduric, cariostatic

C. acidogenic, cariostatic

D. aciduric, cariogenic

Answer: acidogenic, cariogenic

Q44: All of the following describe VLC systems EXCEPT one. Which one is the EXCEPTION?

A. 410-500 nm wavelengths

B. 3 output

C. 2.5 maximum cure depth

D. camphorquinone initiators

Answer: <300 mW/cm3 output


Q45: Which of the following is true regarding composite resin restorations?

A. low polymerization shrinkage

B. increased occlusal wear

C. reduced microleakage

D. all of the above

Answer: all of the above

Q46: Which property of filled resins is primarily to blame for the failure of Class II composite restorations?

A. low flexural strength

B. low compressive strength

C. low tensile strength

D. low wear resistance

Answer: low wear resistance

Q47: Which of the following are the properties of microfill composites? Select all that apply.

A. high modulus of elasticity

B. high wear resistance

C. less receptive to plaque or extrinsic staining

D. small filler particle size

Answer: high wear resistance, less receptive to plaque or extrinsic staining, small filler particle size

Q48: Composite filler particles function to do all of the following EXCEPT one. Which one is the
EXCEPTION?

A. reduce the coefficient of thermal expansion

B. decrease the tensile strength and compressive strength

C. reduce the polymerization shrinkage

D. increase the hardness

Answer: decrease the tensile strength and compressive strength


Q49: When comparing the physical properties of filled resins to unfilled resins, all of the following are true
EXCEPT one. Which one is the EXCEPTION?

A. filled resins are harder

B. unfilled resins have a higher coefficient of thermal expansion

C. filled resins have a higher compressive strength

D. unfilled resins have a lower modulus of elasticity

E. filled resins have a lower tensile strength

Answer: filled resins have a lower tensile strength

Q50: When restoring a darker shade of composite, keep in mind the following:

A. UV light is better than visible light

B. you must keep the light 2 mm away or more

C. you should cure for longer than normal

D. darker shades have less chemical bonding

Answer: you should cure for longer than normal

Q51: Which of the following is/are considered as common complications during mixing of chemically
activated composite resins?

A. air incorporation during mixing

B. no control over working time

C. oxygen-inhibited layer formation

D. all of the above

Answer: all of the above

Q52: The main ingredient in traditional acrylic resin temporary materials for intraoral fabrication is:

A. ethyl methacrylate

B. isobutyl methacrylate

C. Bis-GMA

D. Ethylene imine

E. Methyl methacrylate

Answer: methyl methacrylate


Q53: When considering an unfilled resin material versus a filled resin product, please compare the
properties of thermal conductivity and marginal leakage, respectively:

A. lower, lower

B. lower, higher

C. higher, lower

D. higher, higher

Answer: lower, higher

Q54: Acid etching enamel prior to placement of a composite restoration is required for all of the following
reasons EXCEPT one. Which one is the EXCEPTION?

A. conserves tooth structure

B. reduces microleakage

C. provides micromechanical retention

D. increases esthetics

Answer: increases esthetics

Q55: Enamel etching is typically completed with:

A. 37% phosphoric acid

B. 13% phosphoric acid

C. 37% hydrochloric acid

D. 13% hydrochloric acid

Answer: 37% phosphoric acid

Q56: When preparing a Class V composite preparation, you can have _______rounded internal line angles
because it is ________ to compress composite into them than amalgam.

A. more, easier

B. less, harder

C. more, harder

D. less, easier

Answer: more, easier


Q57: Which one of the following is the most important event for dentin bonding?

A. smear layer removal

B. smear plug removal

C. peritubular dentin decalcification

D. intertubular dentin decalcification

Answer: intertubular dentin decalcification

Q58: A gold onlay you placed last week fails. Which of the following reasons is most likely responsible for
the failure?

A. you "capped" the functional cusp

B. you "shoed" the functional cusp

C. you "capped" the nonfunctional cusp

D. you "shoed" the nonfunctional cusp

Answer: you "shoed" the functional cusp

Q59: Rapid cooling (by immersion in water) of a dental casting from the high temperature at which it has
been shaped is referred to as:

A. annealing

B. tempering

C. quenching

D. none of the above

Answer: quenching

Q60: Which of the following situations defines an indication for a Class II gold inlay?

A. a young patient with high caries rate

B. a patient with little money to invest in dental work

C. a patient very concerned about esthetics

D. a patient with a large lesion, buccal-lingually

E. a patient with low caries rate but a history of periodontal problems

Answer: a patient with low caries rate but a history of periodontal problems
Q61: Gold alloys _________ upon solidification in the investment. This needs to be compensated for by an
equal amount of _________ of the mold.

A. shrink, expansion

B. expand, shrinkage

C. shrink, shrinkage

D. expand, expansion

Answer: shrink, expansion

Q62: Which of the following finishing margins is essentially a hollow ground bevel, creating more bulk of
restorative material near the margin and providing a greater cavosurface angle?

A. knife edge

B. beveled shoulder

C. chamfer

D. shoulder

Answer: chamfer

Q63: Which of the following allows for proper retention when preparing a tooth for a disto-occlusal Class II
gold inlay?

A. undercut on mesial

B. undercut on buccal and lingual walls

C. occlusal lock (dovetail)

D. none of the above

Answer: occlusal lock (dovetail)

Q64: A patient arrives at your office with their full gold crown in hand. They explain to you that another
dentist delivered it just last week. You then examine the crown and the preparation. What is the most likely
reason the crown fell off?

A. there was very little cement in the crown

B. the preparation was only 4 mm high

C. the preparation walls were tapered at about 15°

D. the margins were jagged and undefined

Answer: the preparation walls were tapered at about 15°


Q65: The lab calls your office and asks if you want a particular casting done in a base or a noble metal.
Which of the following responses is appropriate?

A. base, because the patient has a bad bruxing habit

B. noble, because the patient has a bad bruxing habit

C. base, because the patient wants a gold tooth

D. noble, because the patient wants a gold tooth

Answer: base, because the patient has a bad bruxing habit

Q66: Of the following, which is a correct match between the gold cast alloy component and its effect?
Select all that apply.

A. gold — decreases ductility and malleability

B. gold — increases resistance to tarnish and corrosion

C. copper — hardens the alloy

D. silver — color modifying

Answer: gold — increases resistance to tarnish and corrosion, copper — hardens the alloy, silver —
color modifying

Q67: High-gold alloys used for cast restorations are:

A. greater than 20% gold or other noble metals

B. greater than 30% gold or other noble metals

C. greater than 50% gold or other noble metals

D. greater than 75% gold or other noble metals

Answer: greater than 75% gold or other noble metals

Q68: The following statements describe an MOD gold cast onlay preparation. Which would you have to
change to ensure that the onlay will be successful?

A. the mesial box has an axiopulpal line angle that is longer from facial to lingual than the axiogingival line
angle

B. from facial to lingual, the distal axiopulpal line angle is longer than the mesial axiopulpal line angle

C. the mesial and distal axial walls converge

D. the distal box has an axiopulpal line angle that is shorter from facial to lingual than the axiogingival line
angle

Answer: the distal box has an axiopulpal line angle that is shorter from facial to lingual than the
axiogingival line angle
Q69: When preparing a Class V cavity preparation for direct filling gold, you should ensure that you have all
of the following EXCEPT one. Which one is the EXCEPTION?

A. rounded internal line and point angles

B. small retentive undercuts placed in the axio-occlusal and axio-gingival line angles

C. mesial and distal walls that flare and meet the cavosurface at a 90° angle

D. an axial wall that is convex and follows the external contour of the tooth 0.5 mm into dentin

Answer: rounded internal line and point angles

Q70: The purpose of a sprue former is to create a passage for material to flow into the investment.

The diameter of the sprue pin should be equal to or greater than the thickest portion of the pattern.

A. both statements are true

B. both statements are false

C. the first statement is true, the second is false

D. the first statement is false, the second is true

Answer: both statements are true

Q71: Dental wax patterns (i.e., inlays, onlays, crowns) should be invested as soon as possible after
fabricating to minimize change in the shape caused by:

A. reduced flow

B. drying out of the wax

C. relaxation of internal stress

D. continued expansion of the wax

Answer: relaxation of internal stress

Q72: After waxing a pattern for a full-gold crown on #30, you proceed to creating an investment using a
gypsum-bonded material. You read that the investment contains 65% silicon dioxide filler in the material.
What is the primary purpose of this material?

A. provide strength for the set investment to withstand casting forces

B. increase the setting time to give you more time to work

C. create a detailed negative reproduction of your casting pattern

D. provide thermal expansion properties to compensate for casting shrinkage

Answer: provide thermal expansion properties to compensate for casting shrinkage


Q73: All of the following are indications for a cast gold onlay EXCEPT one. Which one is the EXCEPTION?

A. restoration of large lesions

B. restoration of ideal occlusion in cases of drifting, hypo- and hyper eruption, etc.

C. restoration of optimum contour and proximal contact

D. restoration of brittle teeth (endodontically treated)

E. restoration of a tooth as an abutment for removable prosthesis, creating ideal guiding planes, rest
seats, and undercuts

F. restoration of teeth to meet patient preference for gold

G. restoration of a tooth with minimal ferrule, where a full-coverage crown would not have enough
retention

Answer: restoration of a tooth with minimal ferrule, where a full-coverage crown would not have
enough retention

Q74: On delivery of your first crown, you notice that the margins are open when you attempt to seat the
crown in the mouth. Which of the following should you check first?

A. the occlusal contacts

B. the proximal contacts

C. for a void on the crown's interior

D. for a nodule on the crown's interior

Answer: the proximal contacts

Q75: If a lab is getting a high occurrence of surface nodules on the castings they make, what might you
suspect about their investing process?

A. they are vacuum-investing

B. they are hand-investing

C. nothing in particular, hand- and vacuum-investing are the same thing

D. none of the above

Answer: they are hand-investing


Q76: You are preparing tooth #19 for a MODB gold onlay and tooth #18 for a MODB amalgam restoration.
Which of the following TWO statements correctly describe the difference in your preparations of the
functional cusp?

A. you will need 2.5 to 3 mm of reduction for gold

B. you will need 2.5 to 3 mm of reduction for amalgam

C. you will need 1.5 mm of reduction for gold

D. you will need 1.5 mm of reduction for amalgam

Answer: you will need 2.5 to 3 mm of reduction for amalgam, you will need 1.5 mm of reduction for
gold

Q77: The modified pen grasp is the most common instrument grasp in dentistry; this is because it allows for
the greatest intricacy and delicacy of touch.

A. both the statement and the reason are correct and related

B. both the statement and the reason are correct but not related

C. the statement is correct, but the reason is not

D. the statement is not correct, but the reason is correct

E. neither the statement nor the reason is correct

Answer: both the statement and the reason are correct and related

Q78: Soft materials, such as acrylics, are cut most effectively with:

A. zero rake angle burs

B. positive rake angle burs

C. negative rake angle burs

Answer: positive rake angle burs

Q79: The ___________ angle is the angle formed between the ___________ face and the ___________
face.

A. edge, rake, clearance

B. rake, edge, clearance

C. clearance, rake, edge

Answer: edge, rake, clearance


Q80: What is the blade length of a cutting instrument with the following formula: 10 - 85 - 8 - 14

A. 10 mm

B. 1.0 mm

C. 0.85 mm

D. 8.0 mm

Answer: 8.0 mm

Q81: Chisels are used primarily to cut dentin.

Hatchets are primarily used to cut enamel.

A. both statements are true

B. both statements are false

C. the first statement is true, the second is false

D. the first statement is false, the second is true

Answer: the first statement is false, the second is true

Q82: Which of the following instruments is used for preparing retentive areas and sharpening internal line
angles on anterior teeth? Select all that apply.

A. curved chisel

B. bin-angle chisel

C. angle-former

D. hatchet

Answer: angle-former, hatchet

Q83: You set down the hand piece after preparing a Class II amalgam on tooth #4. Your assistant hands
you a _______________ so you can remove the last bit of caries, and then the ____________ so you can
plane the facial and lingual walls of the prep.

A. spoon excavator, enamel hatchet

B. spoon excavator, straight chisel

C. gingival margin trimmer, enamel hatchet

D. gingival margin trimmer, straight chisel

Answer: spoon excavator, enamel hatchet


Q84: To polish a restoration, you will likely use a bur with less cutting blades. This is because a bur with
less cutting blades will cut more efficiently.

A. both the statement and the reason are correct and related

B. both the statement and the reason are correct but not related

C. the statement is correct, but the reason is not

D. the statement is not correct, but the reason is correct

E. neither the statement nor the reason is correct

Answer: the statement is not correct, but the reason is correct

Q85: There are several types of bleaching products available for use at home, which can either be
dispensed by a dentist or purchased over-the-counter. Currently, only dentist-dispensed home-use
_______________tray-applied gels carry the ADA Seal of Acceptance.

A. 30% carbamide peroxide

B. 15% hydrogen peroxide

C. 20% hydrogen peroxide

D. 10% carbamide peroxide

Answer: 10% carbamide peroxide

Q86: All of the following are a zone in four-handed dentistry EXCEPT one. Which one is the EXCEPTION?

A. operator zone

B. assistant zone

C. transfer zone

D. patient zone

E. static zone

Answer: patient zone

Q87: Xerostomia is the most common adverse side effect of medications.

This is attributed to the cholinergic effects of many medications.

A. both statements are true

B. both statements are false

C. the first statement is true, the second is false

D. the first statement is false, the second is true

Answer: the first statement is true, the second is false, provide arch stability
Q89: A tooth was restored 3 months ago by a new associate of yours. The patient is complaining of mobility
and thermal sensitivity. You take a periapical radiograph. You could see all of the following in that
radiograph to confirm your suspicions, EXCEPT one. Which one is the EXCEPTION?

A. hypercementosis

B. root resorption

C. periodontal pockets

D. alteration of the lamina dura

E. widening of the periodontal ligament space

Answer: periodontal pockets

Q90: When restoring the embrasures of posterior teeth, the contact should be formed slightly buccal from
center.

This will create a wider facial embrasure.

A. both statements are true

B. both statements are false

C. the first statement is true, the second is false

D. the first statement is false, the second is true

Answer: the first statement is true, the second is false

Q91: A patient presents to your office complaining of pain upon biting on her lower right side. Upon
examination, the offending tooth appears to have disto-occlusal clinical caries. The patient reports a positive
response to cold testing lasting 12 seconds. The pulpal diagnosis is:

A. normal pulp

B. reversible pulpitis

C. irreversible pulpitis

D. pulpal necrosis

Answer: reversible pulpitis


Q92: On the first day in your solo private practice, you have a pulp exposure. Which of the following are
favorable factors in avoiding root canal treatment? Select all that apply.

A. it is a mechanical exposure of 1 mm

B. the tooth had never been symptomatic

C. the pulp tissue appears pink

D. the hemorrhage is severe

E. it is a pinpoint carious exposure

Answer: it is a mechanical exposure of 1 mm, the tooth had never been symptomatic, the pulp
tissue appears pink

Q93: The ideal amount of time from placing an indirect pulp cap until reopening the tooth to remove the
remaining decay is:

A. 7 to 10 days

B. 2 to 3 weeks

C. 1 month

D. 3 to 4 months

E. 1 year

F. never, unless the tooth becomes symptomatic there is no need for further treatment

Answer: 3 to 4 months

Q94: A patient walks into your office for an emergency visit. He asks the receptionist for a cold glass of
water and seems to be tilting his head sideways as to hold the water on one side of his mouth. Immediately,
you suspect which reason for his visit?

A. pulp necrosis

B. reversible pulpitis

C. symptomatic irreversible pulpitis

D. asymptomatic apical periodontitis

Answer: symptomatic irreversible pulpitis

Q95: A cold test reveals a lingering pain. You ask the patient to raise her hand until the pain subsides. The
patient raises her hand for about 8 seconds. What does this data suggest?

A. pulp necrosis

B. symptomatic irreversible pulpitis

C. reversible pulpitis

D. symptomatic apical periodontitis


Answer: reversible pulpitis
Q96: What was previously an MOD amalgam in #20 now shows that the entire lingual portion of the tooth
has fractured off. You believe that the tooth is restorable with a pin-retained amalgam restoration/core. How
many pins will you likely use and why?

A. one, because you are missing the lingual cusp only

B. two, because you are missing both the mesio- and disto-lingual line angles

C. three, because you are missing the mesial, distal, and lingual walls

Answer: two, because you are missing both the mesio- and disto-lingual line angles

Q97: The most retentive style of pin is the self-threaded pins because they are cemented into pinholes that
are smaller than the pin itself.

A. both the statement and the reason are correct and related

B. both the statement and the reason are correct but not related

C. the statement is correct, but the reason is not

D. the statement is not correct, but the reason is correct

E. neither the statement nor the reason is correct

Answer: the statement is correct, but the reason is not

Q98: A patient presents with DL cusp fracture of #14 which you decide to repair with a pin-retained
amalgam core. Regarding pin placement, you are likely to do all of the following EXCEPT one. Which one is
the EXCEPTION?

A. use one pin at the DL axial line angle

B. place the pin 2 mm into dentin and restorative material

C. place the pin at the DEJ to ensure adequate distance from the pulp

D. bend the pin axially to allow for Amalgam condensation

Answer: place the pin at the DEJ to ensure adequate distance from the pulp

Q99: When preparing a pin channel, you perforate into the vital pulp chamber. What best describes your
next step.

A. explain to the patient the need for root canal treatment

B. allow bleeding to stop, dry with paper point, place calcium hydroxide

C. allow bleeding to stop, dry with paper point, place pin to depth of 1 mm as to not enter the pulp
chamber

D. allow bleeding to stop, dry with paper point, place pin elsewhere, and restore with amalgam

Answer: allow bleeding to stop, dry with paper point, place calcium hydroxide
Q100: The two most frequently quoted disadvantages of using the rubber dam are:

A. time consumption

B. patient objection

C. cost

D. staff allergies to material

Answer: time consumption, patient objection

Q101: The rubber dam can still be used effectively even if teeth are crowded and overlapped because the
hole punch pattern does not always have to be followed.

A. both the statement and the reason are correct and related

B. both the statement and the reason are correct but not related

C. the statement is correct, but the reason is not

D. the statement is not correct, but the reason is correct

E. neither the statement nor the reason is correct

Answer: both the statement and the reason are correct and related

Q102: The Young rubber dam frame is used more than the Woodbury version, because it provides less soft
tissue retraction.

A. both the statement and the reason are correct and related

B. both the statement and the reason are correct but not related

C. the statement is correct, but the reason is not

D. the statement is not correct, but the reason is correct

E. neither the statement nor the reason is correct

Answer: both the statement and the reason are correct but not related

Q103: Put the following steps in order for proper sealant placement.

A. apply acid etch


B. apply bonding agent
C. place the low viscosity sealant material
D. use rubber prophy cup with pumice
E. wash acid etch away

Answer: use rubber prophy cup with pumice, apply acid etch, wash acid etch away, apply bonding
agent, place the low viscosity sealant material
Q104: The light cured sealants require UV light.

The light cured sealant types are shown to be clinically better than chemical cured sealants.

A. both statements are true

B. both statements are false

C. the first statement is true, the second is false

D. the first statement is false, the second is true

Answer: both statements are false

Q105: Regarding the benefits of fluoride, all of the following are true EXCEPT one. Which one is the
EXCEPTION?

A. fluoride increase the hardness of the enamel

B. fluoride reduces the rate of enamel solubility

C. fluoride ions have greater affinity for hydroxyapatite than hydroxyl ions

D. fluoride inhibits bacterial intracellular polysaccharide formation

Answer: fluoride increase the hardness of the enamel

Q106: A low-viscosity pit-and-fissure sealant is a preferred sealing technique as it offers better adaptation of
the sealant with the tooth surface.

The pit-and-fissure sealants should be highly wear-resistant.

A. both statements are true

B. both statements are false

C. the first statement is true, the second is false

D. the first statement is false, the second is true

Answer: the first statement is true, the second is false

Q107: Which of the following is the correct order (initial procedure to the final procedure) of placement of
pit-and-fissure sealants?

A. etching→ washing→ sealant application→ occlusal interference check→ reevaluation

B. etching → sealant application→ washing → occlusal interference check→ reevaluation

C. sealant application→ washing→ etching→ occlusal interference check→ reevaluation

D. washing →etching → sealant application→ occlusal interference check→ reevaluation

Answer: etching→ washing→ sealant application→ occlusal interference check→ reevaluation


Q108: Which of the following treatment options can successfully manage dentin hypersensitivity?

A. fluoride pastes or gel

B. polymerizing agents

C. fluoride iontophoresis

D. all of the above

Answer: all of the above

Q109: Which of the following types and concentrations of fluoride should be recommended for home-care
custom tray use by a patient with head and neck cancer?

A. 1.23% acidulated phosphate fluoride and 1% neutral sodium fluoride

B. 0.4% stannous fluoride and 1.23% acidulated phosphate fluoride

C. 0.4% stannous fluoride and 1% neutral sodium fluoride

D. 1.23% acidulated phosphate fluoride, 0.4% stannous fluoride, and 1% neutral sodium fluoride

Answer: 0.4% stannous fluoride and 1% neutral sodium fluoride

Q110: Match the following fluoride application options with their respective key point.

A. Acidulated phosphate fluoride (APF)


B. Sodium fluoride (NaF)
C. Stannous fluoride (SNF2)

1. Causes the most severe staining


2. Most common in-office fluoride
3. Most common over-the-counter fluoride

Answer: Acidulated phosphate fluoride (APF) — Most common in-office fluoride, Stannous fluoride
(SNF2) — Causes the most severe staining, Sodium fluoride (NaF) — Most common over-the-counter
fluoride

Q111: The pH of acidulated phosphate fluoride is approximately:

A. 1.0

B. 3.5

C. 7.0

D. 9.2

Answer: 3.5
Q112: A 40 kg child gets into the bathroom cabinet and eats one tube of toothpaste containing 230 mg of
fluoride. The mother calls you first before calling poison control asking whether her son would be OK. You
tell the parent the toxic amount of fluoride based on the child's body weight is:

A. 160 mg

B. 200 mg

C. 240 mg

D. 280 mg

Answer: 200 mg

Q113: Sealants can be effective when placed over incipient carious lesions because caries is caused by
anaerobic microorganisms.

A. both the statement and the reason are correct and related

B. both the statement and the reason are correct but not related

C. the statement is correct, but the reason is not

D. the statement is not correct, but the reason is correct

E. neither the statement nor the reason is correct

Answer: the statement is correct, but the reason is not

Q114: Which of the following is least associated with the short-term strength of a material?

A. creep

B. modulus of elasticity

C. resilience

D. brittleness

Answer: creep

Q115: A particular material is said to have a high modulus of elasticity and low resilience. This means the
material is:

A. flexible with low elastic limit

B. stiff with low elastic limit

C. flexible with high elastic limit

D. stiff with high elastic limit

Answer: stiff with low elastic limit


Q116: In general, ductility _________________ in temperature, whereas malleability________________ in
temperature.

A. increases with increase, decreases with increase

B. decreases with increase, increases with increase

C. none of the above

Answer: decreases with increase, increases with increase

Q117: Wetting is a way to measure adhesive potential of a material. In general, the______ the contact
angle, the _____ the wetting, and _____ potential for adhesion.

A. smaller, greater, lesser

B. smaller, lesser, greater

C. larger, lesser, greater

D. larger, lesser, lesser

Answer: larger, lesser, lesser

Q118: Which of the following properties most accurately describes the resistance to crack propagation of a
dental prosthesis made with brittle materials?

A. strength

B. compression

C. shear stress and strain

D. fracture toughness

Answer: fracture toughness

Q119: Secondary dentin is formed in response to caries, operative procedures, or wear.

The dentin formed in response is formed by secondary odontoblasts.

A. both statements are true

B. both statements are false

C. the first statement is true, the second is false

D. the first statement is false, the second is true

Answer: the first statement is false, the second is true


Q120: Which of the following secondary retentive features provides additional incisal support in case of
class III amalgam restorations?

A. skirts

B. beveled enamel margins

C. amalgam pins

D. grooves

Answer: grooves

Q121: The outline form of a cavity preparation is defined as:

A. that form the cavity takes to resist the forces of mastication

B. that form the cavity takes to resist dislodgement or displacement of the restoration

C. the shape or form of the cavity on the surface of the tooth

D. the shape or form of the preparation after carious dentin has been excavated

E. the shape or form the preparation assumes after the retention form has been completed

Answer: the shape or form of the cavity on the surface of the tooth

Q122: A patient returns to your office only 24 hours after you cemented her new gold crown on tooth #19.
She claims to feel a sharp electrical sensation in both her upper and lower jaw on the left side. When you
explain to her what might be happening, you call it:

A. electromagnetic pulse

B. alternating current corrosion

C. electrolyte explosion

D. galvanic shock

Answer: galvanic shock

Q123: Please rank the following according to their coefficient of thermal expansion (least to greatest).
Which material would allow the least amount of percolation and possibility of recurrent decay?

A. amalgam
B. composite
C. direct gold
D. tooth
E. unfilled resin

Answer: tooth, direct gold, amalgam, composite, unfilled resin


Q124: Which cavities can involve any teeth, anterior or posterior?
Select all that apply.

A. Class I

B. Class II

C. Class III

D. Class IV

E. Class V

F. Class VI

Answer: Class I, Class V, Class VI


Oral Pathology

Q1: The congenital absence of six or more teeth is known as:

A. hypodontia

B. oligodontia

C. diphyodontia

D. anodontia

Answer: oligodontia

Q2: Which type of dentinogenesis imperfecta features multiple pulp exposures, periapical radiolucencies,
and a variable radiographic appearance?

A. Type I

B. Type II

C. Type III

Answer: Type III

Q3: The permanent maxillary centrals in the x-ray below are vital. What is the most probable interpretation
of the condition illustrated?

A. mesiodens

B. concrescence

C. fusion

D. dens in dente

Answer: mesiodens

Q4: All of the following are differential diagnoses for hypercementosis EXCEPT one. Which one is the
EXCEPTION?

A. cemento-osseous dysplasia

B. cementoblastoma

C. condensing osteitis

D. odontogenic keratocyst

Answer: odontogenic keratocyst


Q5: An enamel defect resulting from the incomplete formation of the enamel matrix is called:

A. enamel pearls

B. enamel hypocalcification

C. enamel hypoplasia

D. regional odontodysplasia

Answer: enamel hypoplasia

Q6: A new patient presents to your office and on examination, you notice an abnormally wide maxillary
incisor and a total of three maxillary incisors instead of four. The condition is most likely due to which of the
following anomalies?

A. gemination

B. twinning

C. concrescence

D. fusion

Answer: fusion

Q7: All of the following statements concerning amelogenesis imperfecta are true EXCEPT one. Which one
is the EXCEPTION?

A. it is an inherited condition that is transmitted as a dominant trait

B. because of the enamel malformation, the teeth of individuals with amelogenesis imperfecta are often
discolored, sensitive to temperature changes, and painful to brush

C. it only affects the permanent teeth

D. it causes the enamel to be soft and thin

E. the teeth appear yellow because the dentin is visible through the thin enamel

F. the teeth are easily damaged and susceptible to decay

Answer: it only affects the permanent teeth

Q8: The abnormal loss of tooth structure due to non-masticatory physical friction is referred to as:

A. erosion

B. abfraction

C. attrition

D. abrasion

Answer: abrasion
Q9: Erythropoietic porphyria–related discoloration is a type of extrinsic dental stain.

The tooth stain in congenital erythropoietic porphyria is red-brown in color.

A. both statements are true

B. both statements are false

C. the first statement is true, the second is false

D. the first statement is false, the second is true

Answer: the first statement is false, the second is true

Q10: All of the following statements regarding internal root resorption are true EXCEPT one. Which one is
the EXCEPTION?

A. is usually accompanied by pain and swelling

B. radiograph shows uniform radiolucent pulp canal enlargement

C. is carried out by multinucleated giant cells near the pulp granulation tissue

D. is rare as compared to external root resorption

Answer: is usually accompanied by pain and swelling

Q11: Your patient has been diagnosed with amelogenesis imperfecta. His teeth demonstrate enamel that
varies from thin and smooth to normal thickness with grooves, furrows, and pits. This patient's hereditary
condition is an example of the _____ defect.

A. type I (hypoplastic)

B. type II (hypomaturation)

C. type III (hypocalcified)

Answer: type I (hypoplastic)

Q12: In Type I dentin dysplasia, roots appear extremely short, and pulps are:

A. normal

B. somewhat smaller

C. extremely large

D. completely obliterated

Answer: completely obliterated


Q13: Where does leukemia form in the body and which type is mostly commonly found in children?

A. lymph nodes, ALL

B. bone marrow, AML

C. lymph nodes, AML

D. bone marrow, ALL

Answer: bone marrow, ALL

Q14: A 48-year-old female patient walks into your office. She states that she has been diagnosed with
some disease, the name of which she can't remember. Her physician wants her to follow up with your
office regularly to watch out for cancer of the tongue and throat. She also has a bald tongue, and states
that her fingernails "look funny." What disease does she have?

A. aplastic anemia

B. Plummer-Vinson syndrome

C. pernicious anemia

D. Cushing syndrome

Answer: Plummer-Vinson syndrome

Q15: All of the following are characteristics of sickle cell anemia EXCEPT one. Which one is the
EXCEPTION?

A. crescent shaped RBC

B. production of hemoglobin S

C. more common in females

D. RBC lifespan of 120 days

E. loss of trabeculae evident in radiographs

F. valine for glutamic acid substitution

G. muscle and joint pain are common

Answer: RBC lifespan of 120 days


Q16: All of the following are true of acute leukemias EXCEPT one. Which one is the EXCEPTION?

A. slow onset and progression

B. characterized by the appearance of immature, abnormal cells in the bone marrow and peripheral
blood and frequently in the liver, spleen, lymph nodes, and other parenchymatous organs

C. marked by the effects of anemia, which are usually severe (fatigue, malaise), an absence of
functioning granulocytes (prone to infection and inflammation), and thrombocytopenia (hemorrhagic
diathesis)

D. moderate enlargement of the spleen, liver, and lymph nodes. Fever and very high ESR

E. leukocyte counts vary greatly from patient to patient

Answer: slow onset and progression

Q17: Which type of purpura is characterized by a low platelet count that is caused by abnormal thrombosis
in terminal arterioles?

A. thrombocytopenic purpura

B. thrombotic thrombocytopenic purpura

C. both of the above

D. neither of the above

Answer: thrombotic thrombocytopenic purpura

Q18: You have a dental patient who mentions during his health history that he has an "overactive thyroid."
You ask him about his medications and he states that he doesn't know what he takes now, but at one time
he took methimazole. He had to stop that because it "really tore up my gums and the roof of my mouth and
I got a lot of infections." Which condition of the blood is most commonly caused as a reaction to medication
that could have caused these symptoms?

A. thrombocytopenic purpura

B. agranulocytosis

C. sickle cell anemia

D. Peutz-Jegher syndrome

Answer: agranulocytosis

Q19: A dentist is often consulted first by a patient with pernicious anemia for relief of:

A. denuded gingiva

B. glossitis

C. edematous buccal mucosa

D. severe gingivitis

Answer: glossitis
Q20: In your office, you see a 6-month-old child whose first teeth are erupting and whose mother is
concerned about the color. The mandibular incisors do show a brownish-blue hue. You are going to ask
the mother about which of the following conditions during her pregnancy:

A. sickle cell anemia

B. erythroblastosis fetalis

C. patent ductus arteriosus

D. low-weight preterm birth

Answer: erythroblastosis fetalis

Q21: Which of the following statements are false regarding chronic leukemias?

A. they have a delayed onset and progression

B. the clinical course is less devastating than that of an acute leukemia

C. they constitute 75% of all leukemias

D. they are characterized by proliferations of lymphoid or hematopoietic cells that are more mature than
those of the acute leukemias

Answer: they constitute 75% of all leukemias

Q22: Leukemias are evenly split between the acute and chronic forms, but among children, one form
accounts for about two-thirds of cases. This one form is:

A. acute lymphocytic leukemia (ALL)

B. acute myeloid leukemia (AML)

C. chronic lymphocytic leukemia (CLL)

D. chronic myeloid leukemia leukemia (CML)

Answer: acute lymphocytic leukemia (ALL)


Q23: You have a new patient in your dental office who has just moved from Denver. He says his doctor
told him that he has some disease caused by living at a high altitude. When conducting an intraoral exam,
you find that his tongue is a deep purple and his gingiva bleed easily. What disease is a likely cause of
these findings?

A. polycythemia vera (primary)

B. secondary polycythemia

C. hemophilia B

D. thalassemia major

E. porphyria

Answer: secondary polycythemia


Q24: The translocation from chromosome 22 to chromosome 9 is a finding of which leukemia?

A. acute lymphocytic leukemia (ALL)

B. chronic myeloid leukemia (CML)

C. acute myeloid leukemia (AML)

D. chronic lymphocytic leukemia (CLL)

Answer: chronic myeloid leukemia (CML)

Q25: Which of the following is an encapsulated mass that presents as an asymptomatic lump?

A. neurofibroma

B. traumatic neuroma

C. neurilemmoma

D. nodular fasciitis

Answer: neurilemmoma

Q26: While in the OR on a general surgery rotation, a 3-month-old is brought in with a large (20 cm) fluid-
filled mass on her neck. The diagnosis is a cystic hygroma. This lesion is under which umbrella of lesions,
which also contains enlarged tissue on the posterior and lateral border of the tongue?

A. angiomas

B. lymphangiomas

C. schwannomas

D. fibrosarcomas

Answer: lymphangiomas

Q27: A 24-year-old female patient comes into your office complaining of 14-mm exophytic, red mass
present on the gingiva between teeth 5 and 6. A health history reveals that she is a smoker and is 3
months pregnant. Her oral hygiene is poor. A likely diagnosis of the mass is:

A. epulis granulomatosa

B. pyogenic granuloma

C. peripheral ossifying fibroma

D. peripheral giant cell granuloma

Answer: pyogenic granuloma


Q28: A patient presents to your clinic with multiple exophytic masses covering the buccal mucosa, tongue,
and lips. A biopsy reveals that these are mucosal neuromas. The most important reason this patient
should be referred to a physician is because of the risk of related:

A. squamous cell carcinoma of the tongue

B. pituitary hyperplasia

C. medullary carcinoma of the thyroid

D. Sipple syndrome

Answer: medullary carcinoma of the thyroid

Q29: Which of the following statements are true regarding neurofibroma? Select all that apply.

A. a benign nerve sheath tumor

B. an encapsulated tumor

C. seen as interlacing bundles of spindle cells

D. can be treated with medication

Answer: a benign nerve sheath tumor, seen as interlacing bundles of spindle cells

Q30: Von Recklinghausen disease displays all of the following characteristics EXCEPT one. Which one is
the EXCEPTION?

A. autosomal dominant

B. axillary freckling

C. café au lait macules

D. iris hamartomas

E. Optic gliomas

F. due to a mutation of the tumor suppressor gene NF3

Answer: due to a mutation of the tumor suppressor gene NF3

Q31: A 55-year-old patient comes into your office for a routine dental cleaning. You see that he has a tooth
fracture (due to decay) of tooth #31. A smooth, firm, asymptomatic lesion is noted on the lateral border of
the tongue adjacent to the sharp enamel of tooth #31. The patient states that the lesion has been there for
years and is annoying because sometimes he will bite it accidentally. Name this most frequently
encountered intraoral benign neoplasm of connective tissue origin.

A. a leiomyosarcoma

B. a fibroma

C. a leiomyoma

D. a rhabdomyoma

Answer: a fibroma
Q32: Systemic sclerosis (scleroderma) is a systemic disease that affects many organ systems. What is
usually the first sign of the disease?

A. lung fibrosis

B. Raynaud's phenomenon

C. microstomia

D. hypertension

Answer: Raynaud's phenomenon

Q33: A newborn girl was delivered via cesarean section due to airway patency concerns. During
ultrasound, there was the discovery of a tumor of the oral cavity. On delivery, the pink, compressible tumor
of the anterior maxilla was deemed to be a congenital epulis of the newborn. This lesion is composed of
cells that are identical to those of the:

A. traumatic neuroma

B. schwannoma

C. granular cell tumor

D. lipoma

Answer: granular cell tumor


Q34: An emergency patient walks into your office with swelling of the left submandibular space. He says
his lower left molar recently "broke down" and has been very painful, especially when something cold hits it
or when he chews on it. What is the most likely etiology of this swelling?

A. orthodontics

B. trauma

C. infection of the pulp of the tooth

D. periodontal disease

Answer: infection of the pulp of the tooth

Q35: A healthy patient comes into your office for an initial exam. On the full mouth series of radiographs,
you see a radiopaque lesion periapical to tooth #19. Tooth #19 has a deep amalgam restoration with
recurrent decay underneath. Though the lesion is not separated from the apex, you can see the entire
outline of the mesial root of tooth #19. The radiopaque lesion does not have a radiolucent rim. What is the
most likely diagnosis of the lesion?

A. condensing osteitis

B. cementoblastoma

C. focal cemento-osseous dysplasia

D. idiopathic osteosclerosis

Answer: condensing osteitis


Q36: All forms of osteomyelitis are much more commonly seen in the mandible.

Unless the inflammatory process has been present for more than 1 week, radiographic evidence of acute
osteomyelitis is usually not present.

A. both statements are true

B. both statements are false

C. the first statement is true, the second is false

D. the first statement is false, the second is true

Answer: both statements are true

Q37: Cleft palate usually occurs starting in the _____ week of embryonic life whereas cleft lip usually
occurs starting in the _____ week.

A. 8th week, 6th week

B. 10th week, 8th week

C. 6th week, 8th week

D. 8th week, 10th week

Answer: 8th week, 6th week

Q38: Achondroplasia is an autosomal dominant disorder caused by a mutation in the __________ gene.

A. TRPS1

B. COL10A1

C. FGFR3

D. FLNB

Answer: FGFR3

Q39: Type I dentinogenesis imperfecta is associated with which genetic disease?

A. osteogenesis imperfecta

B. Marfan syndrome

C. Ehlers-Danlos

D. cystic fibrosis

Answer: osteogenesis imperfect


Q40: Hypophosphatasia is a genetic metabolic disorder of bone mineralization caused by a deficiency in:

A. acid phosphatase

B. vitamin K

C. alkaline phosphatase

D. phosphorus

Answer: alkaline phosphatase

Q41: Pituitary adenoma in a 9-year-old will most likely lead to:

A. gigantism

B. acromegaly

C. achondroplasia

D. dwarfism

Answer: gigantism

Q42: A new 6-year-old pediatric patient walks into your operatory with her mother. Your initial physical
assessment notes a prominent forehead and flattened nose. The patient initially seems to have no
eyebrows but you later realize that the hair is just very fine and sparse. When you shake her hand and she
smiles, you also notice that she is missing teeth and the ones she has are cone shaped. What is her most
likely systemic condition?

A. Pierre Robin syndrome

B. ectodermal dysplasia

C. cleidocranial dysplasia

D. Peutz-Jeghers syndrome

E. osteopetrosis

Answer: ectodermal dysplasia

Q43: A 65-year old Caucasian male presents to your office with ill-fitting dentures. Radiographic and
clinical exam lead you to believe the patient has Paget disease. Which of the following would support this
diagnosis? Select all that apply.

A. hypercementosis

B. enlarged cranium

C. bone of decreased density

D. headaches

E. hearing loss

Answer: hypercementosis, enlarged cranium, headaches, hearing loss


Q44: Cherubism is a benign, autosomal dominant condition exhibiting all of the following characteristics
EXCEPT one. Which one is the EXCEPTION?

A. radiographically shows multilocular radio-opacities

B. histologically close to central giant cell granuloma

C. clinically presents as non-tender swellings

D. treatment is frequent monitoring

E. most cases occur in mandible

F. affects males more than females

Answer: radiographically shows multilocular radio-opacities

Q45: A deficiency of parathyroid hormone can be treated with:

A. vitamin A

B. vitamin C

C. vitamin D

D. vitamin K

Answer: vitamin D

Q46: Graves disease is the most acute and severe form of hyperthyroidism caused by the autoimmune
disease targeting which receptor?

A. MSH receptor

B. TSH receptor

C. GH receptor

D. PTH receptor

Answer: TSH receptor

Q47: Severe hypothyroidism is called _____ in children, whereas it is called _____ in adults.

A. Hashimoto disease, cretinism

B. cretinism, Graves disease

C. myxedema, dwarfism

D. cretinism, myxedema

Answer: cretinism, myxedema


Q48: The clinical features of the primary form of which disease is classically described as "stones, bones,
groans, and moans?"

A. Paget disease

B. hypophosphatasia

C. hyperparathyroidism

D. hyperthyroidism

Answer: hyperparathyroidism

Q49: On a hospital rotation you see an infant who displays bowed legs and muscular weakness. On dental
examination you notice a delayed eruption pattern. The child has rickets, which is a deficiency in which
vitamin?

A. vitamin A

B. vitamin D

C. vitamin C

D. vitamin E

Answer: vitamin D

Q50: Which of the following conditions may be seen in a patient with cerebral palsy?
Select all that apply.

A. difficulty with mastication and swallowing

B. higher incidence of periodontal disease and caries

C. attrition of the teeth

D. multilocular radiolucencies of the jaws

Answer: difficulty with mastication and swallowing, higher incidence of periodontal disease and
caries, attrition of the teeth

Q51: Which of the following diseases is associated with a decrease in caries?

A. Sjögren syndrome

B. cystic fibrosis

C. cerebral palsy

D. Down syndrome

Answer: cystic fibrosis


Q52: You are listening to a story about oyster fishing from a fellow classmate concerning his trip to the
East Coast. He mentions that he got sick and had to be taken to the ER where he was told he had
hepatitis. He is fine now. Which hepatitis is the most likely culprit in your colleague?

A. hepatitis A

B. hepatitis B

C. hepatitis C

D. hepatitis D

Answer: hepatitis A - also called infectious, viral or short-incubation hepatitis

Q53: Which of the following is the most common reason for liver transplant in the United States and has
been linked to a higher incidence of hepatocellular carcinoma?

A. hepatitis A

B. hepatitis B

C. hepatitis C

D. hepatitis D

Answer: hepatitis C

Q54: Which of the following aid in wound healing? Select all that apply.

A. highly vascular areas (i.e., tongue)

B. hyperthermia

C. younger age

D. cortisone

E. hypothermia

Answer: highly vascular areas (i.e., tongue), hyperthermia, younger age

Q55: In which one of the following cases is an incisional biopsy indicated?

A. 3 x 3 mm well-encapsulated fibroma

B. necrotizing sialometaplasia of the hard palate

C. 2 x 2 mm papilloma of left commissure of lips

D. 3 x 2 mm melanotic macule

Answer: necrotizing sialometaplasia of the hard palate


Q56: Epstein-Barr virus (EBV) is associated with all of the following EXCEPT one. Which one is the
EXCEPTION?

A. nasopharyngeal carcinoma

B. oral hairy leukoplakia

C. Burkitt lymphoma

D. Koplik spots

E. infectious mononucleosis

Answer: Koplik spots

Q57: Epithelioid cells and giant cells are derived from macrophages and are important in the development
of:

A. initial inflammation

B. granulomatous inflammation

C. acute inflammation

D. subacute inflammation

Answer: granulomatous inflammation

Q58: A patient you saw yesterday had minor swelling of the submandibular space associated with a
carious #31. You prescribed amoxicillin and sent him home. He called today to say the swelling has gotten
worse. You squeeze him into the schedule and notice that he has trouble breathing. You call an
ambulance to escort him to the ER and tell the paramedics that he has Ludwig's angina. Ludwig angina is
a severe and spreading infection that involves the:

A. submental and sublingual spaces only

B. submandibular, submental, and sublingual spaces unilaterally

C. submandibular and sublingual spaces only

D. submandibular, submental, and sublingual spaces bilaterally

Answer: submandibular, submental, and sublingual spaces bilaterally

Q59: ESR rises with all of the following EXCEPT one. Which one is the EXCEPTION?

A. inflammation

B. administration of hydrocortisone

C. necrosis

D. suppuration

E. pregnancy

Answer: administration of hydrocortisone


Q60: Your 2-year-old nephew tells you he feels sick and then proceeds to vomit on the kitchen floor. You
rush him to the bathroom where you notice toothpaste everywhere and his "SpongeBob" toothpaste tube
completely empty. Your aunt asks you, "What is the estimated toxic dose for fluoride ingestion"? Your
answer to her is:

A. 1-2 mg/kg

B. 5-10 mg/kg

C. 8-10 mg/kg

D. 12-15 mg/kg

Answer: 5-10 mg/kg

Q61: The most common cause of xerostomia is:

A. hereditary

B. medications

C. tooth decay

D. mouth breathing

Answer: medications

Q62: Hereditary gingival fibromatosis is a benign condition affecting both arches.

Microscopically, hereditary gingival fibromatosis resembles fibrous hyperplasia.

A. both statements are true

B. both statements are false

C. the first statement is true, the second is false

D. the first statement is false, the second is true

Answer: both statements are true

Q63: Radiographically, traumatic bone cysts present as a:

A. well-defined unilocular or multilocular radiolucency with scalloping around the roots

B. poorly circumscribed radiopaque lesion which may have a "ground-glass" appearance

C. saucer-shaped radiopaque lesion

D. poorly defined multilocular radiolucency with a "pear-shaped" appearance between the maxillary
central incisors

Answer: well-defined unilocular or multilocular radiolucency with scalloping around the roots
Q64: Where in the oral cavity is malignant melanoma most commonly found?

A. tongue and mandibular alveolar ridge

B. buccal mucosa and pharyngeal pillars

C. palate and maxillary gingiva

D. mucobuccal fold of lower lip

Answer: palate and maxillary gingiva

Q66: A 62-year-old African American patient comes into your office complaining of his loose lower molars.
The health history reveals recent lower back pain. On a hunch, you send him down the hall to the
orthodontist to take a lateral skull radiograph that reveals "punched-out" radiolucencies. You will refer this
patient to the physician suspecting a diagnosis of:

A. non-Hodgkin's lymphoma

B. Hodgkin's lymphoma

C. multiple myeloma

D. Langerhan's cell disease

Answer: multiple myeloma

Q67: The least common type of malignant melanoma is:

A. superficial spreading melanoma

B. lentigo maligna melanoma

C. acral-lentiginous melanoma

D. nodular melanoma

Answer: acral-lentiginous melanoma

Q68: Which of the following grades of oral epithelial dysplasia shows marked pleomorphism and abnormal
proliferation of basal layer cells into the upper third of epithelium?

A. grade-1

B. grade-2

C. grade-3

D. grade-1 and grade-2

Answer: grade-3
Q69: The most common site of squamous cell carcinoma of the tongue is the:

A. dorsum

B. ventral surface

C. tip

D. posterior lateral border

Answer: posterior lateral border

Q70: Of the following types of squamous cell carcinomas, which is the least common?

A. squamous cell carcinoma of the nasopharynx

B. squamous cell carcinoma of the oropharynx

C. squamous cell carcinoma of the maxillary sinus

Answer: squamous cell carcinoma of the nasopharynx

Q71: At which growth stage is metastasis most likely for malignant melanoma?

A. "horizontal" growth phase

B. "circular" growth phase

C. "vertical" growth phase

D. "radial" growth phase

Answer: "vertical" growth phase

Q72: Cancer of which oral cavity structure is most commonly associated with mortality?

A. lip

B. Tongue

C. floor of mouth

D. buccal mucosa

Answer: tongue
Q73: A 47-year-old HIV+ patient is referred from his physician to your office because of an exophytic
growth in the maxillary left mucobuccal fold. Unable to find anodontogenic source, a biopsy was done. A
starry-sky morphology was found and a dismal diagnosis of Burkitt's lymphoma was made. Which virus is
thought to be responsible for this lymphoma?

A. herpes virus

B. Epstein-Barr virus

C. cytomegalovirus

D. human papillomavirus

Answer: Epstein-Barr virus

Q74: Ewing's sarcoma most often presents radiographically as:

A. multiple radiolucent/radiopaque lesions resembling "cotton ball" or "cotton wool" appearance

B. multiple "punched-out" radiolucencies

C. ill-defined lytic lesion with an onionskin parosteal reaction in the long bones

D. ill-defined radiolucent lesion resembling "ground glass"

Answer: ill-defined lytic lesion with an onionskin parosteal reaction in the long bones

Q75: A 15-year-old patient presents to his physician because of localized pain in his right femur and
rapidly enlarging swelling. A radiograph of the area shows a "sun-ray" appearance. Which is a likely
diagnosis for this patient based on incidence?

A. chondrosarcoma

B. osteosarcoma

C. scleroderma

D. chronic osteomyelitis

Answer: osteosarcoma

Q76: While doing a dental mission trip in Ethiopia, you notice purplish-brown nodules on the hard palate of
a 32-year-old female patient. You notice more of these spots on her arms and legs. Given that HIV
infection has an almost 5% prevalence in urban Addis Ababa, for what neoplasm are you suspicious of?

A. nicotinic stomatitis

B. hemangioma

C. Kaposi sarcoma

D. leukemia

Answer: Kaposi sarcoma


Q77: The ___________ presents as a movable, painless submucosal nodule with a yellowish-pink
discoloration most commonly in the floor of the mouth.

A. lipoma

B. ranula

C. lymphoma

D. oral lymphoepithelial cyst

Answer: Oral lymphoepithelial cyst

Q78: Which of the following statements concerning metastatic tumors of the jaws are correct.
Select all that apply.

A. they may be completely asymptomatic

B. the patient is usually aware of slight discomfort or pain

C. the maxilla is affected far more frequently than the mandible

D. the molar region is predominantly involved

Answer: they may be completely asymptomatic, the patient is usually aware of slight discomfort or
pain, the molar region is predominantly involved

Q79: What is the stage for an oral cavity carcinoma with TNM designation T1, N2c, M1?

A. stage I

B. stage III

C. stage IVa

D. stage IVb

E. stage IVc

Answer: stage IVc

Q80: A 65-year-old patient of East Indian origin presents to your clinic with a complaint of a lesion on the
maxillary alveolar tuberosity. The lesion is a thick white, exophytic mass with a cauliflower appearance. A
social history reveals that this woman has been chewing a betel nut concoction from her native India for
over 40 years. What is the likely diagnosis of this lesion?

A. papilloma

B. erythroplakia

C. verrucous carcinoma

D. hyperkeratosis

Answer: verrucous carcinoma


Q81: A 73-year-old patient presents to your office with complaint of a lasting ulceration on the right side of
his tongue. He has a history of hypertension and high cholesterol and a 40-year pack history of smoking.
The patient takes antihypertensives and antilipidemics and has no allergies. You cannot find any sources
of trauma. After 2 weeks, the ulcer has grown in size. What is the likely diagnosis of this most common
malignancy of the oral cavity?

A. adenoid cystic carcinoma

B. mucoepidermoid carcinoma

C. basal cell carcinoma

D. squamous cell carcinoma

Answer: squamous cell carcinoma

Q82: While attempting to give an inferior alveolar nerve block, if you inject the anesthetic solution into the
capsule of the parotid gland, you may cause a Bell's palsy-like feeling for the patient by anesthetizing the:

A. trigeminal nerve

B. glossopharyngeal nerve

C. hypoglossal nerve

D. facial nerve

Answer: facial nerve

Q83: A 25-year-old college student comes into your office complaining that, when she wakes up, she has
trouble opening her mouth. When conducting a TMJ exam, you note tenderness of the right lateral
pterygoid and nonreciprocal clicking of the right TMJ. What is the most likely cause of the patient's
myofacial pain?

A. trauma

B. muscle spasm

C. periodontal disease

D. tumor

Answer: muscle spasm

Q84: A 53-year-old patient comes to your office and notes that, sometimes when he swallows, he gets a
sharp "jolt" on the right side of his throat. He says the pain is severe and he can even feel it in his ear. The
most likely diagnosis is:

A. postherpetic neuralgia

B. orolingual paresthesia

C. Frey syndrome

D. glossopharyngeal neuralgia

Answer: glossopharyngeal neuralgia


Q85: Which of the following is a relatively rare autoimmune disorder of peripheral nerves in which
antibodies form against acetylcholine (ACh) nicotinic post-synaptic receptors at the myoneural junction?

A. myasthenia gravis

B. myelofibrosis

C. multiple sclerosis

D. Graves disease

Answer: myasthenia gravis

Q86: A 34-year-old dentophobe is your patient for the morning. After getting a very hesitant health history,
you decide to begin your oral exam. As you reach toward her face, she immediately flinches and puts her
hands up. She lets you know that if you touch a particular point above her lip, she gets sharp, stabbing
jolts of pain. You let her know that a neurologist can work her up for:

A. glossopharyngeal neuralgia

B. trigeminal neuralgia

C. postherpetic neuralgia

D. diabetic neuralgia

Answer: trigeminal neuralgia

Q87: Which of the following are typically within soft tissue and may mimic inflammatory lesions of
odontogenic origin?

A. median palatal cyst

B. traumatic bone cyst

C. nasolabial cyst

D. nasopalatine cyst

Answer: nasolabial cyst

Q88: Which of the following cysts are congenital? Select all that apply.

A. thyroglossal duct cyst

B. branchial cyst

C. stafne bone cyst

D. dermoid cyst

Answer: thyroglossal duct cyst, branchial cyst, dermoid cyst


Q89: Which of the following cysts are developmental (or fissural)? Select all that apply.

A. nasopalatine duct (canal)

B. nasolabial (nasoalveolar)

C. branchiogenic

D. median palatal

E. median alveolar

Answer: nasopalatine duct (canal), nasolabial (nasoalveolar), median palatal, median alveolar

Q90: A nasopalatine duct cyst is the most common nonodontogenic oral cyst.

A nasopalatine duct cyst is derived from embryonic epithelial remnants of paired nasopalatine ducts.

A. both statements are true

B. both statements are false

C. the first statement is true, the second is false

D. the first statement is false, the second is true

Answer: both statements are true

Q91: The soft tissue, and far less common, variant of the nasopalatine canal cyst is the:

A. median mandibular cyst

B. nasolabial cyst

C. cyst of the incisive papilla

D. aneurysmal bone cyst

Answer: cyst of the incisive papilla

Q92: An 8-year-old girl, who looks like she is 14 years old, comes with her father into your office. Her
father states during the health history that she has McCune-Albright syndrome. Which of the following
would you expect the patient to have? Select all that apply.

A. congenital heart defects

B. polyostotic fibrous dysplasia

C. café au lait spots

D. endocrine dysfunction

Answer: polyostotic fibrous dysplasia, café au lait spots, endocrine dysfunction


Q93: An 8-year-old child has complained of pain on the left side of his head for 5 weeks. There are no
abnormal findings on physical examination. A pan reveals multiple radiolucent lesions on the left side of
the maxilla. The lesions give the appearance of teeth that are "floating in air.''The lesions are sharply
circumscribed with a punched-out appearance. Which of the following is the most likely diagnosis?

A. Langerhans cell disease

B. hyperparathyroidism

C. cherubism

D. Paget disease

Answer: Langerhans cell disease

Q94: Which of the following are true regarding central giant cell granulomas? Select all that apply.

A. appear more frequently than peripheral giant cell granulomas

B. found predominantly in children and young adults

C. affects females more than males

D. present almost exclusively in the small bones of the hands and feet

Answer: found predominantly in children and young adults, affects females more than males

Q95: A 21-year-old male patient is home from college and came to you because his "bite seems off."A
quick physical assessment seems to indicate that his chin is deviated to the right. Taking a panoramic x-
ray and comparing to the previous panoramic radiographs you have in his chart, you notice that the left
condylar neck seems to have elongated. What condition does this patient most likely have?

A. condylar agenesis

B. condylar hyperplasia

C. condylar hypoplasia

D. hemifacial microsomia

Answer: condylar hyperplasia

Q96: A 6-year-old boy is a patient in your practice who has been hospitalized multiple times for broken
bones. During routine lab tests, it was noted that his alkaline phosphatase levels were quite high. If this
patient has a form of fibrous dysplasia, which radiographic feature may you find?

A. the lesions are usually radiolucent, well-circumscribed, and may have a "cotton wool" appearance

B. the lesions are usually a saucer-shaped radiolucency

C. the lesions are usually radiopaque, not well-circumscribed, and may have a "ground-glass"
appearance

D. the lesions are usually well-demarcated unilocular or multilocular radiolucencies

Answer: the lesions are usually radiopaque, not well-circumscribed, and may have a "ground-
glass" appearance
Q97: Mandibular tori most often appear:

A. in the retromolar pad region

B. on the lingual surface of the mandible, most often in the premolar region

C. on the lingual surface of the mandible, inferior to the mylohyoid ridge

D. along the midline of the hard palate

Answer: on the lingual surface of the mandible, most often in the premolar region

Q98: All of the following are features of an ossifying fibroma EXCEPT one. Which one is the EXCEPTION?

A. slow growing expansile lesion

B. more often in maxilla

C. asymptomatic

D. common in young adults around 35 years of age

E. more common in females

Answer: more often in maxilla

Q99: A 17-year-old patient of yours comes in for a routine examination. A head and neck examination
reveals multiple cysts of the skin. Her panoramic exams have always shown multiple impacted teeth.
Today, her panoramic exam shows multiple radiopacities of the jaws, especially at the angle of the
mandible. You suspect Gardner syndrome. What is the most serious complication she should be
concerned with when consulting her physician?

A. odontomas

B. osteomas

C. epidermoid cysts

D. multiple polyps that affect the large intestine

Answer: multiple polyps that affect the large intestine

Q100: A 10-year-old boy comes with his mother to the dental office because of a painless swelling of his
maxilla. Radiographic exam reveals an irregularly shaped radiopaque mass with a ground-glass
appearance. No other bulges have been noted by the mother. A biopsy reveals fibrous tissue in the bone.
What is the most likely diagnosis?

A. monostotic fibrous dysplasia

B. polyostotic fibrous dysplasia

C. Albright syndrome

D. Jaffe syndrome

Answer: monostotic fibrous dysplasia


Q101: A new patient walks into your office. Your initial physical assessment reveals that his eyes are set
wide and that he has multiple lesions of the skin. When shaking his hand, you notice that the skin of his
palm is very thick and has palmer pitting. When doing a health history, he reveals that he sees a
neurologist and that he has some calcified structures "in his brain." A panoramic radiograph may likely
reveal:

A. osteomas

B. keratocystic odontogenic tumors

C. odontomas

D. dentigerous cysts

Answer: keratocystic odontogenic tumors

Q102: A mother brings her 2-year-old boy into the dental office because of a "swelling" on his alveolar
ridge. Your exam reveals a smooth-surfaced bluish lesion with fluctuance where tooth #K will be erupting.
The most likely diagnosis of this is:

A. dentigerous cyst

B. eruption cyst

C. hematoma

D. hemangioma

Answer: eruption cyst

Q103: Upon viewing a panorex of a 14-year-old patient, you see a well-defined radiolucency with
scalloping around the roots on the left side of the mandible apical to the canine and first premolar. No
clinical symptoms are present. Teeth are not carious and respond normally to vitality tests. Medical history
is unremarkable. On opening the area, no fluid or tissue is evident. What is the most probable diagnosis?

A. dentigerous cyst

B. traumatic (simple) bone cyst

C. primordial cyst

D. residual cyst

E. Stafne (static) bone defect

Answer: traumatic (simple) bone cyst


Q104: A healthy 19-year-old patient presents to your office for a routine exam. Taking a panoramic
radiograph, you see a well-corticated, unilocular radiolucency surrounding the crown of impacted tooth
#17. The lesion is asymptomatic. What is the most likely diagnosis?

A. keratocystic odontogenic tumor

B. dentigerous cyst

C. cystic ameloblastoma

D. central ossifying fibroma

Answer: dentigerous cyst - or Follicular cyst

Q105: The odontogenic keratocyst (or keratocystic odontogenic tumor) is derived from which of the
following:

A. Hertwig's epithelial root sheath

B. the reduced enamel epithelium

C. remnants of the dental lamina

D. a preexisting osteoma

Answer: remnants of the dental lamina

Q106: Which of the following most commonly appears radiographically as a well-circumscribed


radiolucency with corticated margins, located laterally to a vital mandibular canine?

A. lateral periodontal cyst

B. dentigerous cyst

C. keratocystic odontogenic tumor

D. all of the above

Answer: lateral periodontal cyst

Q107: A dental granuloma and a radicular cyst can be differentiated:

A. based on symptoms

B. radiographically

C. histologically

D. by an electric pulp tester

Answer: histologically - and only histologically


Q108: A 37-year-old patient comes into your office with the complaint of a slow-growing, painless swelling
of his lower left jaw. A panoramic shows a multilocular radiolucency with well-defined and sclerotic margins
along the left mandibular molar-ramus area. All teeth test as vital. A biopsy showed the lesion to be an
ameloblastoma. It is most likely to be which type of ameloblastoma?

A. solid (multicystic or polycystic)

B. unicystic

C. extraosseous (peripheral)

Answer: solid (multicystic or polycystic)

Q109: A 30-year-old patient comes into your office complaining of a painless swelling of his lower left jaw.
A panoramic radiograph shows a well-circumscribed multilocular radiolucency with a "honeycomb" pattern
at the location of the lower left molars. The teeth have been displaced. The pathology report calls this an
odontogenic myxoma. This tumor:

A. is composed of large polyhedral, neoplastic, epithelial cells

B. is composed of neoplastic epithelium and mesenchyme

C. arises from odontogenic ectomesenchyme

D. is composed of spindle-shaped mesenchymal cells and aggregates of multinucleated giant cells

Answer: arises from odontogenic ectomesenchyme

Q110: The cementoblastoma is more often seen:

A. in the mandible than in the maxilla, and more often in the posterior than in the anterior regions

B. in the mandible than in the maxilla, and more often in the anterior than in the posterior regions

C. in the maxilla than in the mandible, and more often in the posterior than in the anterior regions

D. in the maxilla than in the mandible, and more often in the anterior than in the posterior regions

Answer: in the mandible than in the maxilla, and more often in the posterior than in the anterior
regions

Q111: All of the following are true regarding periapical cemento-osseous dysplasia EXCEPT one. Which
one is the EXCEPTION?

A. appears in middle age

B. mandible is affected more than maxilla

C. a predilection for middle-aged black women

D. no treatment is usually required

E. teeth are non-vital

Answer: teeth are non-vital


Q112: The most common location for a Pindborg tumor (calcifying epithelial odontogenic tumor [CEOT]) is
the:

A. tuberosity area

B. maxillary anterior area

C. mandibular premolar area

D. posterior mandible

Answer: posterior mandible

Q113: The ameloblastic fibroma and ameloblastic fibro-odontoma appear to be variations of the same
process. These neoplasms occur predominantly in:

A. adults with a mean age of 40

B. elderly people with a mean age of 75

C. young adults with a mean age of 25

D. children and young adults

Answer: children and young adults

Q114: Odontoma is a tumor of odontogenic origin with completely differentiated odontogenic epithelium
and ectomesenchyme.

Complex odontomas are morphologically similar to normal teeth.

A. both statements are true

B. both statements are false

C. the first statement is true, the second is false

D. the first statement is false, the second is true

Answer: the first statement is true, the second is false

Q115: A 45-year-old African-American female presents to your office for a routine exam. Periapicals of the
mandibular incisors show multiple radioopacities with radiolucent rims. Teeth #23 through #26 test as vital.
There is no pain on percussion or palpation. Treatment for these lesions should be:

A. do nothing (observe)

B. RCT treatment for teeth #23 through #26

C. surgical excision of lesions

D. none of the above

Answer: do nothing (observe)


Q116: A 5-year-old boy presents with his mother for his first dental exam. Your exam reveals a normally
developing dentition, but you notice multiple "freckles" on his lower lip and on the buccal mucosa. For what
condition should you be concerned?

A. Gorlin-Goltz syndrome

B. Gardner syndrome

C. Peutz-Jeghers syndrome

D. Cleidocranial dysplasia

Answer: Peutz-Jeghers syndrome

Q117: The bluish discoloration of the gingiva, as visible in the picture below, is most likely to be diagnosed
as:

A. melanocytic nevus

B. amalgam tattoo

C. melanotic macule

D. oral melanoacanthoma

Answer: amalgam tattoo

Q118: The most common location for an intraoral congenital nevi (birthmark) is the:

A. buccal mucosa

B. tongue

C. hard palate

D. alveolar mucosa

Answer: hard palate

Q119: Which of the following conditions demonstrate pigmentation of the intraoral mucous membranes.
Select all that apply.

A. Addison disease

B. McCune-Albright syndrome

C. Cushing syndrome

D. Peutz-Jeghers syndrome

Answer: Addison's disease, McCune-Albright syndrome, Peutz-Jeghers syndrome


Q120: A 40-year-old patient presents to his physician with complaints of muscle weakness and loss of
appetite. He has noticed a loss of weight and also that his skin has started to "bronze." His labs show
lowered blood glucose and sodium and increased potassium. One condition likely to be causing this is:

A. Peutz-Jeghers syndrome

B. Cushing syndrome

C. Addison disease

D. Albright syndrome

Answer: Addison disease

Q121: Focal melanosis is a common circumstance in which brownish areas of pigmentation occur in the
oral cavity. Once properly diagnosed:

A. surgical excision is required

B. radiation is required

C. no treatment is necessary

D. antibiotics are required

Answer: no treatment is necessary

Q122: The intraoral nevus is usually a/an:

A. intradermal nevus

B. compound nevus

C. junctional nevus

D. blue nevus

E. intramucosal nevus

Answer: intramucosal nevus

Q123: A patient presents with an asymptomatic, elongated, erythematous patch of atrophic mucosa of the
mid-dorsal surface of the tongue due to a chronic Candida albicans infection. The most likely diagnosis is:

A. thyroglossal duct cyst

B. lymphangioma

C. hemangioma

D. median rhomboid glossitis

Answer: median rhomboid glossitis


Q124: A 13-year-old patient presents with his father to your dental clinic for a routine examination. From
the health history, you gather that the patient has frequent nosebleeds, just like his father. Upon intraoral
examination, you notice multiple red spots on the patient's lower lip and tongue. The father states that he
has these same marks on his tongue, palate, hands, and eyes. What condition do you suspect the child
has?

A. Wegener granulomatosis

B. hereditary hemorrhagic telangiectasia (HHT)

C. Sturge-Weber angiomatosis

D. juvenile nasopharyngeal angiofibroma

Answer: hereditary hemorrhagic telangiectasia (HHT) - aka Rendu-Osler-Weber syndrome

Q125: A clinical term defined as a red patch that cannot be clinically or pathologically diagnosed as any
other condition is called:

A. leukoedema

B. psoriasis

C. erythroplakia

D. white sponge nevus

Answer: erythroplakia

Q126: Which of the following statements is not true about pyogenic granuloma?

A. fast growing, tumor-like growth formed after an exaggerated conditioned response to mild trauma

B. most commonly seen on gingiva, although can be seen anywhere in the mucous membrane

C. lesions bleed easily when probed due to pronounced vascularity

D. older pyogenic granulomas are more vascular than younger lesions

Answer: older pyogenic granulomas are more vascular than younger lesions

Q127: Peripheral giant cell granulomas are seen exclusively in the:

A. buccal mucosa

B. alveolar mucosa

C. bone

D. gingiva

Answer: gingiva
Q128: The picture below shows a benign, soft, moderately well-circumscribed, painless mass which is
deep red or blue-red in coloration. The most likely diagnosis is:

A. lymphangioma

B. hemangioma

C. blue nevus

D. melanotic macule

Answer: hemangioma

Q129: Tumors of the salivary glands are:

A. uncommon and represent 2-4% of head and neck neoplasms

B. common and represent 75-80% of head and neck neoplasms

C. uncommon and represent 25-30% of head and neck neoplasms

D. common and represent 95-98% of head and neck neoplasms

Answer: uncommon and represent 2-4% of head and neck neoplasms

Q130: Which lesion below presents itself as a deep-seated palatal ulcer with clinical and histologic
features mimicking those of a malignant neoplasm?

A. white sponge nevus

B. lichen planus

C. necrotizing sialometaplasia

D. focal hyperkeratosis

Answer: necrotizing sialometaplasia

Q131: In a small Amish community, there is an infectious outbreak. Multiple children are coming down with
symptoms including fever and malaise. Commonly, there is swelling of the parotid glands. Given that this
community does not receive vaccinations, what is a likely diagnosis for the condition?

A. measles

B. mumps

C. rubella

D. chickenpox

Answer: mumps
Q132: The Stafne bone defect is a developmental anomaly represented by a bone concavity usually
containing:

A. parotid gland tissue

B. submandibular gland tissue

C. sublingual gland tissue

D. all of the above

Answer: submandibular gland tissue

Q133: A 53-year-old woman comes into the dental clinic with bilaterally enlarged parotid glands. It was
discovered that she had recently been to the African continent and had contracted tuberculosis. What is
the name of the autoimmune disease associated with enlarged salivary glands in association with a
secondary disease?

A. Sjögren syndrome

B. Mikulicz disease

C. Gorlin-Goltz syndrome

D. Pierre Robin syndrome

E. Apert syndrome

Answer: Mikulicz disease

Q134: A 33-year-old patient comes into your office for a routine maintenance appointment. While doing an
intraoral exam, the hygienist discovers a bluish lesion of the lower lip. The patient relates a history of biting
this area last week when he had a sinus infection. What is the most likely diagnosis of this lesion?

A. ranula

B. infectious sialadenitis

C. maxillary sinus retention cyst

D. mucocele

Answer: mucocele

Q135: What is the most probable diagnosis for a lesion that presents as a translucent, bluish, well-
rounded, smooth-surfaced bulge that protrudes from one side of the floor of the mouth?

A. adenoid carcinoma

B. squamous cell carcinoma

C. a ranula

D. a lymphangioma

Answer: a ranula
Q136: A patient comes to your office complaining of pain when eating and even sometimes when thinking
about food. Your intraoral exam reveals a small, hard swelling in the floor of the mouth. A mandibular
occlusal radiograph shows a pea-sized radiopacity with "onion-skin" thickening lingual to the right
mandibular border. Name the likely diagnosis:

A. sialometaplasia

B. sialadenitis

C. sialolith

D. sialosis

Answer: sialolith

Q137: Of the neoplasms affecting the major or minor glands, the ____________ is the most common.

A. basal cell adenoma

B. sebaceous adenoma

C. pleomorphic adenoma

D. ductal papilloma

Answer: pleomorphic adenoma

Q138: Which of the following disorders should be included in your differential diagnosis of parotid gland
enlargement. Select all that apply.

A. sarcoidosis

B. Mikulicz disease

C. Sjögren syndrome

D. hypothyroidism

E. diabetes mellitus

F. malnutrition/starvation

G. dehydration

H. cystic fibrosis

Answer: sarcoidosis, mikulicz disease, sjögren syndrome, diabetes mellitus,


malnutrition/starvation, dehydration, cystic fibrosis
Q139: The acinic cell carcinoma is derived from serous acinar cells and is found almost exclusively in the:

A. submandibular gland

B. parotid gland

C. minor glands of the palate

D. sublingual gland

Answer: parotid gland

Q140: ________ is the most common salivary gland malignancy and makes up between 5% and 9% of all
salivary gland neoplasms.

A. adenoid cystic carcinoma

B. mucoepidermoid carcinoma

C. acinic cell carcinoma

D. polymorphous low-grade adenocarcinoma

Answer: mucoepidermoid carcinoma

Q141: Which autoimmune disease is associated with the increase in caries?

A. Lupus erythematosus

B. Sjögren syndrome

C. Sarcoidosis

D. Crohn disease

Answer: Sjögren syndrome

Q142: The _____ salivary gland presents with the most number of tumors and _____carcinoma is the
most common malignant salivary gland neoplasm.

A. parotid, acinic cell

B. parotid, mucoepidermoid

C. sublingual, mucoepidermoid

D. sublingual, acinic cell

Answer: parotid, mucoepidermoid


Q143: Oncocytomas are _______ tumors that constitute about _______of benign epithelial salivary gland
neoplasms.

A. common; 50%

B. common; 75%

C. rare; 2%

D. rare; 15%

Answer: rare; 2%

Q144: A 65-year-old patient comes to your office complaining of a slowly growing enlargement of the jaw.
You palpate the angle of his right ramus and find an encapsulated mass that is nontender and firm. Your
oral pathologist defines it as a glandular and cystic tumor lined by a bilayered (inner columnar oncocytic
and outer basal) epithelium with a lymphoid stroma. Name this second most common benign neoplasm of
the parotids.

A. pleomorphic adenoma

B. Warthin tumor

C. fibroadenoma

D. monomorphic adenoma

Answer: Warthin tumor

Q145: A 40-year-old female comes to your clinic with the complaint of a lesion on the lower lip that has
been increasing slowly during the past 2 years. On examining, the subcutaneous growth is nontender, soft,
and mobile. A biopsy taken to confirm the type of lesion reveals a yellow lobulated cut surface.

The lesion being localized and slow growing is suggestive of a malignant tumor.

The cells in benign tumors are always well differentiated, resembling their normal cells of origin.

A. both statements are true

B. both statements are false

C. the first statement is true, the second is false

D. the first statement is false, the second is true

Answer: the first statement is false, the second is true


Q146: An overgrowth of normal tissues in a place where that tissue is not normally found is referred to as
a:

A. teratoma

B. choristoma

C. hamartoma

D. none of the above

Answer: choristoma

Q147: Malignant neoplasms range from well-differentiated to undifferentiated. Malignant neoplasms


composed of undifferentiated cells are said to be:

A. metaplastic

B. hyperplastic

C. anaplastic

D. dysplastic

Answer: anaplastic

Q148: Metaplasia most commonly occurs by replacement of ________ by _______ .

A. cuboidal cells; columnar cells

B. columnar cells; stratified squamous epithelium

C. columnar cells; pseudostratified columnar cells

D. cuboidal cells; stratified squamous epithelium

Answer: columnar cells; stratified squamous epithelium

Q149: A 25-year-old dental student has been cramming for his dental physiology and pharmacology final
exams. He is sleep deprived but otherwise healthy. When flossing, he notices a 3-mm ulceration on the
inner surface of his lower lip. He can find no other instances of this lesion and remembers that he had one
a long time ago in undergraduate clinic, which went away. What is the most likely diagnosis?

A. recurrent aphthous minor

B. recurrent aphthous major

C. recurrent herpetiform aphthous

D. recurrent herpetic stomatitis

Answer: recurrent aphthous minor


Q150: Which of the following make up the triad of Stevens-Johnson syndrome (SJS)?

A. stomatitis

B. lesions of the eye

C. genital lesions

D. maculopapular rash

Answer: stomatitis, lesions of the eye, genital lesions

Q151: The picture below showcases a solitary small whitish sore with a red border measuring 2-5 mm in
diameter. The patient is a 14-year-old female with a history of its recurrent appearance beginning two
months ago. Each time the lesion heals on its own within 10 days. It begins as a reddish area with a
burning or tingling sensation. The most likely diagnosis is:

A. recurrent minor aphthous ulcers

B. recurrent major aphthous ulcers

C. pemphigous

D. recurrent herpetiform ulceration

Answer: recurrent minor aphthous ulcers

Q153: A 43-year-old man presents to the ER with a purple lump of the jaw that is painful. He has an
extraoral sinus tract presenting with yellowish crust. The ER places the individual on a long-term penicillin
regimen. What was the most likely diagnosis?

A. coccidioidomycosis

B. histoplasmosis

C. tuberculosis

D. actinomycosis

E. scarlet fever

Answer: actinomycosis
Q154: Treponema pallidum is the infectious organism of which disease?

A. syphilis

B. gonorrhea

C. chlamydia

D. tuberculosis

Answer: syphilis

Q155: All of the following are the features of Hutchinson triad which are pathognomonic of congenital
syphilis EXCEPT one. Which one is the EXCEPTION?

A. eighth nerve deafness

B. interstitial keratitis

C. screwdriver-shaped central incisor

D. multiple fibroma

Answer: multiple fibroma

Q156: Which of the following fungal infections is most commonly associated with diabetes mellitus?

A. coccidioiodomycosis

B. mucormycosis

C. aspergillosis

D. none of the above

Answer: mucormycosis

Q152: Erythema multiforme (EM) is an acute self-limited eruption characterized by a distinctive clinical
eruption, the hallmark of which is the:

A. chronic desquamative gingivitis

B. petechial hemorrhage

C. iris or target lesion

D. mucocutaneous rash

Answer: iris or target lesion - appears as a central lesion surrounded by concentric rings of pallor
and redness over the dorsal aspect of the hands and forearms
Q157: A 4-year-old patient comes with her mother for a routine appointment. The mother states that her
daughter just started not feeling well and had a mild fever earlier in the day. The daughter has been having
trouble swallowing. An intraoral exam reveals multiple 1-mm to 2- mm vesiculopapular lesions of the
nasopharynx and soft palate. Your working diagnosis is:

A. herpangina

B. hand-Foot-and-Mouth disease

C. herpes simplex infection

D. pemphigus vulgaris

Answer: herpangina

Q158: Which type of herpes virus is associated with the lesion on the lower lip?

A. HSV-1

B. HSV-2

C. HSV-3

D. HSV-4

Answer: HSV-1

Q159: The dormancy state with latency in the trigeminal ganglion is associated with which form of herpes?

A. Cytomegalovirus

B. Epstein-Barr virus

C. Herpes simplex virus type 1

D. Herpes simplex virus type 2

Answer: Herpes simplex virus type 1


Q160: A 65-year-old man presents to your clinic in regards to upper dentures. When getting his health
history, you ask about history of infectious diseases. He mentions that a year ago, his eldest son died and
later he broke out in blisters on only half of his back. He says he doesn't remember what the doctor called
it, but he remembers the physician saying it "stopped at the midline." What is the most likely diagnosis?

A. herpes zoster

B. herpangina

C. recurrent herpes

D. chickenpox

Answer: herpes zoster

Q161: Over 90% of primary herpes simplex viral infections are:

A. manifested as ANUG

B. associated with HIV

C. subclinical

D. characterized by severe lymphadenopathy and acute dermatitis

Answer: subclinical

Q162: After the initial primary attack during the early childhood period, the herpes simplex virus remains
inactive most commonly in the:

A. geniculate ganglion

B. ciliary ganglion

C. trigeminal ganglion

D. pterygopalatine ganglion

Answer: trigeminal ganglion

Q163: A 49-year-old patient of Ashkenazi heritage presents to your office complaining of "blisters in her
mouth." Your intraoral exam shows ulcers present on multiple areas of mucosa. She also related to you
that, while getting out of the car earlier, the skin of her arm rubbed against the car door and tore. Known as
Nikolsky sign, this phenomenon is associated with which disease?

A. herpes zoster

B. lupus erythematosus

C. lichen planus

D. pemphigus

Answer: pemphigus
Q164: The oral lesions of benign mucous membrane pemphigoid most commonly present as a:

A. candidiasis

B. hairy leukoplakia

C. desquamative gingivitis

D. hemorrhagic mass

Answer: desquamative gingivitis

Q165: The common wart or verruca vulgaris is caused by the:

A. human papillomavirus

B. adenovirus

C. Epstein-Barr virus

D. Human parvovirus

Answer: human papillomavirus

Q166: A 45-year-old female walks into your office complaining of a "wart" on her gums that has been there
for years. Your exam reveals an asymptomatic, well circumscribed, slightly raised, papillomatous lesion on
the buccal gingiva of tooth #5. A likely diagnosis of this is:

A. fibrosarcoma

B. neurosarcoma

C. lipoma

D. Verruciform xanthoma

Answer: verruciform xanthoma

Q167: A 63-year-old completely edentulous patient comes into your office because her dentures have
"finally gotten too bothersome to wear." Her health history consists of COPD and cigarette 40-year pack
history. Your intraoral exam reveals a noxious odor and an ill-fitting upper denture. When you remove the
upper denture you note multiple red, papillary projections of the hard palate. Your patient states she does
not remove her dentures at night or between meals. After reviewing denture hygiene instructions, you give
her the diagnosis of:

A. epulis fissuratum

B. inflammatory papillary hyperplasia

C. nicotinic stomatitis

D. Kaposi sarcoma

Answer: inflammatory papillary hyperplasia


Q168: A 54-year-old African-American female presents to your clinic for an initial exam. She has a history
of hypertension controlled with beta-blockers but no other contributory health findings. Your intraoral exam
reveals a bilateral filmy opalescence of the buccal mucosa. When stretching out her cheeks, this white hue
disappears. Your most likely diagnosis is:

A. Squamous cell carcinoma

B. Fordyce granulation

C. Leukoedema

D. Leukoplakia

Answer: Leukoedema

Q169: A 67-year-old Caucasian male comes into your office for a routine check-up. He relates to you that
he just got back from Florida where he goes for the fall and winter months. He enjoys taking his boat out
with his wife. Your extraoral exam shows chapped lips, but his lower lip also presents with grayish-white
plaques. There is a blurring of the vermilion border. Which of the following would you make your diagnosis:

A. actinic keratosis

B. actinic cheilitis

C. actinic dermatitis

D. solar lentigo

Answer: actinic cheilitis

Q170: An incisional biopsy is indicated for which of the following lesions?

A. a 0.2-cm exostosis of the hard palate

B. a 0.2-cm area of Fordyce granules of the cheek

C. a 0.3-cm hemangioma of the tongue

D. a 0.3-cm area of leukoplakia of the soft palate

Answer: a 0.3-cm area of leukoplakia of the soft palate

Q171: You are conducting a routine exam on a 54-year-old patient with diabetes mellitus type 2 and a 20-
year pack history of smoking. You see a white patch on the floor of the mouth. The lesion cannot be wiped
off and the patient denies a history of trauma or allergies. Which of the following would be your diagnosis?

A. squamous cell carcinoma

B. lichen planus

C. erythroplakia

D. leukoplakia

Answer: leukoplakia
Q172: A 75-year-old patient comes to your office wanting a new set of dentures. She hasn't been wearing
her old dentures for about 2 years. She has a collapsed vertical dimension of occlusion and her physician
is concerned about her iron deficiency. The corners of her mouth are fissured, dry, and erythematous.
Which of the following conditions is the likely diagnosis?

A. squamous cell carcinoma

B. angular cheilitis

C. verruca vulgaris

D. stomatitis nicotina

Answer: angular cheilitis - also called perlèche

Q173: A 34-year-old male comes into the clinic for an initial exam. Your health history is noncontributory.
The patient presents with bilateral asymptomatic, white, folded and spongy tissue on the buccal mucosa.
There is no history of cheek biting, and the patient recalls that the lesions have been present as long as he
can remember. Your diagnosis is:

A. hyperkeratosis

B. leukoplakia

C. epidermolysis bullosa

D. white sponge nevus

Answer: white sponge nevus

Q174: Hairy tongue is a condition characterized by hypertrophy of the:

A. filiform papillae

B. fungiform papillae

C. circumvallate papillae

D. foliate papillae

Answer: filiform papillae

Q175: A 62-year-old African-American female patient presents to your clinic for routine dental work. Your
intraoral exam reveals white, lace-like webbing on the buccal mucosa. Your patient has never noticed
these and they have never caused her a problem. You suspect the following:

A. lupus erythematosus

B. erythema multiforme

C. pemphigus vulgaris

D. lichen planus

Answer: lichen planus


Q176: A white patch that, when scraped or removed from the oral mucosa, leaves a raw surface most
likely is:

A. leukoplakia

B. white sponge nevus

C. candidiasis

D. lichen planus

Answer: candidiasis

Q177: Identify the white patch seen below in a 65-year-old male patient, who is a long-time smoker.

A. leukoplakia

B. leukoedema

C. mucosal burn

D. nicotine stomatitis

Answer: leukoplakia

Q178: A 35-year-old healthy female presents to your office for a routine cleaning. While completing the
scaling on the LL quadrant, you notice that her tongue has multiple irregularly shaped red lesions that
have a white border. You make a note in her chart. When she returns 2 weeks later for the restorative work
on the LR quadrant with the dentist, he notes that there are still lesions, but in different locations on the
tongue with different shapes. What is your diagnosis?

A. fissured tongue

B. macroglossia

C. geographic tongue

D. hairy tongue

Answer: geographic tongue


Oral Radiology

Q1: Which type of digital image receptor is most commonly used at this time?

A. CID (charge injection device)

B. CMOS/APS (complementary metal oxide semiconductor/active pixel sensor)

C. CCD (charge-coupled device)

Answer: CCD (charge-coupled device)

Q2: All of the following are advantages of direct digital radiography EXCEPT one. Which one is the
EXCEPTION?

A. superior gray-scale resolution

B. reduced patient exposure to x-radiation

C. increased speed of image viewing

D. lower equipment and film costs

E. sensor size

F. increased efficiency

G. Effective patient education tool

H. enhancement of diagnostic image

Answer: sensor size


Q3: A method of obtaining a digital image where the sensor captures the image and immediately transfers
it to a computer is termed:

A. indirect digital imaging

B. direct digital imaging

C. storage phosphor imaging

Answer: direct digital imaging

Q4: A patient is extremely concerned about radiation exposure. Which of the following is best for limiting
the amount of exposure he will receive during a full mouth series?

A. use of digital imaging

B. use of E-speed films

C. use of F-speed films

D. substitute a panoramic image for the full mouth series

Answer: use of digital imaging

Q5: A radiograph that exhibits areas of black and white is termed high contrast and is said to have a short
contrast scale; a radiograph that exhibits many shades of gray is termed low contrast and is said to have a
long contrast scale.

To limit image magnification, the longest target-receptor distance and shortest object-receptor distance are
used.

A. both statements are true

B. both statements are false

C. the first statement is true, the second is false

D. the first statement is false, the second is true

Answer: both statements are true

Q6: From the list provided below, order the following from LEAST to MOST radiopaque.

• amalgam
• bone
• dentin
• enamel
• maxillary sinus

Answer: maxillary sinus — bone — dentin — enamel — amalgam


Q7: Dental radiographs are the legal property of the:

A. patient

B. dentist

C. state

D. none of the above

Answer: dentist

Q8: A dental hygienist in your practice has an adult recall patient without evidence of caries who states she
needs bite-wing x-rays because it has been 6 months since her last dental images. The hygienist should
tell the patient that:

A. Yes, she is correct, it is time for new x-ray images

B. Bite-wings should be taken only once per year, not twice

C. Images should be taken based on patient need instead of a set time frame

D. None of the above

Answer: Images should be taken based on patient need instead of a set time frame
Q9:

A. Image 1 - Maxillary tuberosity; Image 2 - Hamulus

B. Image 1 - Hamulus; Image 2 - Maxillary tuberosity

C. Image 1 - Coronoid process; Image 2 - Hamulus

D. Image 1 - Hamulus; Image 2 - Coronoid process

Answer: • Image 1 - Hamulus


• Image 2 - Maxillary tuberosity

Q10: The coronoid process often appears on what periapical image?

A. maxillary incisor

B. maxillary molar

C. mandibular incisor

D. mandibular molar

Answer: maxillary molar


Q11: Match the structures labeled 1-8 on the image (bottom left) with their description (bottom right).

• 1.
• 2.
• 3.
• 4.
• 5.
• 6.
• 7.
• 8.
• Anterior wall of the maxillary sinus
• Border of maxillary sinus
• Floor of nasal fossa
• Lamina dura
• Lateral wall of the incisive (nasopalatine) canal
• Nasopalatine fossa
• Periodontal ligament space
• Soft tissue outline of the nose

• Answers 1-8 below


Q12: Match the structures labeled 1-7 on the image (bottom left) with their description (bottom right).

• 1.
• 2.
• 3.
• 4.
• 5.
• 6.
• 7.
• Alveolar crest
• Anterior nasal spine
• Floor of nasal fossa
• Incisive (nasoplatine) foramen
• Lateral wall of nasopalatine canal
• Median palatal suture
• Soft tissue outline of tip of nose

• Answers 1-7 below


Q13: Identify the structures labeled 1- 5 on the image (bottom left) with their description (bottom right).

• 1.
• 2.
• 3.
• 4.
• 5.
• A. Submandibular gland fossa
• B. Nutrient canal
• C. Inferior border of mandible
• D. Bony trabecular plate
• E. Inferior border of mandibular canal

Answer: 1. B, 2. D, 3. E, 4. A, 5. C
Q14: Match the structures labeled 1-8 on the image (bottom left) with their description (bottom right).

• 1.
• 2.
• 3.
• 4.
• 5.
• 6.
• 7.
• 8.
• Anterior wall of maxillary sinus
• Floor of maxillary sinus
• Floor of nasal fossa
• Inferior border of zygoma
• Inferior border of zygomatic process of maxilla
• Inferior nasal conchae
• Mucosa over alveolar bone
• Posterior wall of zygomatic process of maxilla

• Answers 1-8 below


Q15: Match the structures labeled 1-7 on the image (bottom left) with their description (bottom right).

• 1.
• 2.
• 3.
• 4.
• 5.
• 6.
• 7.
• Bony trabeculations
• Film holder
• Genial tubercles
• Lingual cusp of 1st premolar
• Lingual foramen
• Marrow space
• Periodontal ligament space

Answer: Answers 1-7 below


Q16: Match the structures labeled 1-4 on the image (bottom left) with their description (bottom right).

• 1.
• 2.
• 3.
• 4.
• A. Submandibular gland fossa
• B. Film clip mark
• C. Periodontal ligament space
• D. Mental foramen

Answer: 1. C, 2. D, 3. A, 4. B
Q17: Match the structures labeled 1-3 on the image (bottom left) with their description (bottom right).

• 1.
• 2.
• 3.
• A. Submandibular gland fossa
• B. Mental foramen
• C. Cementoenamel junction (CEJ)

Answer: 1. C, 2. B, 3. A
Q18: Match the structures labeled 1-7 on the image (bottom left) with their description (bottom right).

• 1.
• 2.
• 3.
• 4.
• 5.
• 6.
• 7.
• Anterior wall of maxillary sinus
• Floor of maxillary sinus
• Floor of nasal fossa
• Inferior border of the zygomatic process of the maxilla
• Inferior nasal conchae
• Lingual cusp of 1st premolar
• Maxillary sinus

• Answers 1-7 below


Q19: Match the structures labeled 1- 6 on the image (bottom left) with their description (bottom right).

• 1.
• 2.
• 3.
• 4.
• 5.
• 6.
• Ala of nose
• Anterior wall of maxillary sinus
• Floor of nasal fossa
• Lateral wall in incisive canal
• Lingual cusp of 1st premolar
• Maxillary sinus

• Answers 1-6 below


Q20: Match the structures labeled 1-6 on the image (bottom left) with their description (bottom right).

• 1.
• 2.
• 3.
• 4.
• 5.
• 6.
• Dentino-enamel junction (DEJ)
• Film holder
• Lamina dura
• Mucosa over alveolar bone
• Periodontal ligament space
• Periodontal ligament space of palatal root

• Answers 1-6 below


Q21: Match the structures labeled 1- 3 in the image (bottom left) with their description (bottom right).

• 1.
• 2.
• 3.
• A. Lingual foramen
• B. Genial tubercles
• C. Mandibular tori

Answer: 1. C, 2. A, 3. B
Q22: Match the structures labeled 1-4 in the image (bottom left) with their description (bottom right).

• 1.
• 2.
• 3.
• 4.
• A. Trabecular bone
• B. Periodontal ligament space
• C. Alveolar crest of bone
• D. Lamina dura

Answer: 1. C, 2. D, 3. B, 4. A
Q23: Match the structures labeled 1-8 on the image (bottom left) with their description (bottom right).

• 1.
• 2.
• 3.
• 4.
• 5.
• 6.
• 7.
• 8.
• Alveolar crest
• Bony trabecular plate
• Dentin
• Enamel
• Lamina dura
• Marrow space
• Periodontal ligament space
• Pulp canal

• Answers 1-8 below


Q24: Match the structures labeled 1-9 on the image (bottom left) with their description (bottom right).

• 1.
• 2.
• 3.
• 4.
• 5.
• 6.
• 7.
• 8.
• 9.
• Alveolar crest
• Bony trabeculations
• Dentin
• Enamel
• Lamina dura
• Marrow space
• Periodontal ligament space
• Pulp canal
• Pulp chamber

• Answers 1-9 below


Q25: Identify the structure labeled 6 on the image below.

A. bony trabeculations

B. dentino-enamel junction

C. periodontal ligament space

D. marrow space

Answer: periodontal ligament space

Q26: Identify the structure labeled 6 on the image below.

A. amalgam restoration

B. lamina dura

C. film dot

D. plastic bite block

Answer: lamina dura


Q27: Identify the structure labeled 4 on the image below.

A. nasal septum

B. infraorbital rim

C. hyoid bone

D. zygomatic arch

Answer: infraorbital rim


Q28: Identify the structure labeled 10 on the image below.

A. articular eminence

B. mental foramen

C. floor of maxillary sinus

D. air between soft palate & tongue

Answer: mental foramen

Q29: The pattern of stored energy on an exposed film is termed the latent image; this image remains
invisible until it undergoes processing. The function of the developer solution is to chemically reduce the
exposed, energized silver halide crystals to black metallic silver.

A. both statements are true

B. both statements are false

C. the first statement is true, the second is false

D. the first statement is false, the second is true

Answer: both statements are true


Q30: Which ingredient's main function is to remove all unexposed and underdeveloped silver halide
crystals from the emulsion?

A. fixing agent

B. acidifier

C. hardening agent

D. preservative

E. none of the above

Answer: fixing agent

Q31: Your assistant has processed three panoramic films today. She noticed the films are progressively
getting lighter and lighter. What should be done to correct the problem?

A. decrease the temperature of the developer

B. increase the temperature of the fixer

C. replenish the developer

D. process the films a second time

E. decrease the time in the developer

Answer: replenish the developer

Q32: Your assistant has just processed an x-ray film that has a dark spot on it that appears to be caused
due to contact of the film with the tank wall during fixation. Identify the mechanism behind the formation of
this dark spot?

A. contact with the tank wall prevented the fixer from dissolving the unexposed silver bromide crystals

B. contact with the tank wall caused contamination of the film

C. contact with the tank wall caused insufficient washing with water

D. contact with the tank wall caused excessive bending of the film

Answer: contact with the tank wall prevented the fixer from dissolving the unexposed silver
bromide crystals
Q33: Which of the following is the most probable cause of the error on the film below?

A. fixer cut-off

B. developer cut-off

C. fingernail damage

D. static electricity

E. air bubbles

Answer: static electricity

Q34: Dose equivalent is expressed in terms of:

A. Coulombs/kilogram (C/kg)

B. Gray (Gy)

C. Sievert (Sv)

D. Quality factor (QF)

Answer: Sievert (Sv)


Q35: List the following cells from most RADIORESISTANT to most RADIOSENSITIVE.

• 1.
• 2.
• 3.
• 4.
• muscle
• skin
• small lymphocyte
• thyroid gland

Answer: muscle — thyroid gland — skin — small lymphocyte

Q36: After exposure to radiation, symptoms such as hair loss can occur but not until days after the initial
exposure. The time between the initial exposure and onset of symptoms is termed:

A. latent period

B. period of cell injury

C. recovery period

D. cumulative effects period

Answer: latent period

Q37: A patient with a large squamous cell carcinoma of the lateral border of the tongue is scheduled for a
radical neck dissection. Prophylactic extractions of hopeless teeth must be done to prevent which of the
following?

A. osteoradionecrosis

B. bisphosphonate osteoradionecrosis

C. periodontal disease

D. rampant caries

E. none of the above

Answer: osteoradionecrosis
Q38: The most common oral problems that occur following radiation and chemotherapy include mucositis,
infection, pain and bleeding.

The oral cavity is irradiated during the course of treating radiosensitive oral malignancies, usually
squamous cell carcinoma.

A. both statements are true

B. both statements are false

C. the first statement is true, the second is false

D. the first statement is false, the second is true

Answer: both statements are true

Q39: In the dental x-ray tube, the number of electrons flowing per second is measured by:

A. kilovoltage peak (kVp)

B. milliamperage (mA)

C. time (in seconds)

D. none of the above

Answer: milliamperage (mA)

Q40: When the PID length is changed from 8" to 16'', the target-receptor distance is doubled. According to
the Inverse Square Law, the resultant x-ray beam is:

A. 1/4 as intense

B. 1/8 as intense

C. four times more intense

D. eight times more intense

E. none of the above

Answer: 1/4 as intense


Q41: A 6'5'' muscular male with a large mandible requires a complete series of dental images. You plan to
increase the kVp because of his size. Identify each of the following that would change as a result to the
increased kVp:

A. a more penetrating beam

B. a less penetrating beam

C. a reduced subject contrast

D. an increased subject contrast

E. long scale contrast

F. short scale contrast

Answer: • a more penetrating beam


• a reduced subject contrast
• long scale contrast

Q42: All of the following influence the density of an image EXCEPT one. Which one is the EXCEPTION?

A. kVp

B. mA

C. exposure time

D. use of a 2-film packet

Answer: use of a 2-film packet

Q43: Which of the following supplies electrons necessary to generate x-rays?

A. positive anode

B. negative anode

C. positive cathode

D. negative cathode

Answer: negative cathode


Q44: Which of the following focuses the electrons into a narrow beam and directs the beam across the
tube toward the tungsten target of the anode?

A. copper stem

B. tungsten filament

C. insulating oil

D. molybdenum cup

E. lead collimator

Answer: molybdenum cup

Q45: The anode in an x-ray tube consists of a tungsten target. All of the following are the characteristics of
an ideal target material EXCEPT one. Which one is the EXCEPTION?

A. high melting point

B. high thermal conductivity

C. high atomic number

D. high vapor pressure at the working temperature

Answer: high vapor pressure at the working temperature

Q46: Rectification is the conversion of a direct current (DC) to an alternating current (AC).

The dental x-ray tube acts as self-rectifier in that in changes DC to AC while producing x-rays.

A. both statements are true

B. both statements are false

C. the first statement is true, the second is false

D. the first statement is false, the second is true

Answer: both statements are false

Q47: Which of the following occurs only at 70 kVp or higher and accounts for a very small part of the x-
rays produced in the dental x-ray machine?

A. compton scatter

B. coherent scatter

C. characteristic radiation

D. general (Bremsstrahlung) radiation

Answer: characteristic radiation


Q48: All of the following are components of inherent filtration EXCEPT one. Which one is the
EXCEPTION?

A. insulating oil

B. unleaded glass window

C. lead lined PID

D. tubehead seal

Answer: lead lined PID

Q49: Identify each of the following that is recommended for operator protection during exposure.

A. stand 3 feet away from x-ray tubehead

B. stand at a 45-75 degree angle to the beam

C. wear a lead apron

D. stand behind a barrier

E. hold the PID

F. hold the film if the patient cannot stabilize it

Answer: stand behind a barrier

Q50: Prior to x-ray exposure, the proper prescribing of radiographs and the use of proper equipment can
minimize the amount of radiation that a patient receives.

Radiographs must be prescribed by the dentist based on the individual needs of the patient.

A. both statements are true

B. both statements are false

C. the first statement is true, the second is false

D. the first statement is false, the second is true

Answer: both statements are true

Q51: Which of the following is used to restrict the size and shape of the x-ray beam and to reduce patient
exposure?

A. aluminum discs

B. collimation

C. inherent filtration

D. total filtration

Answer: collimation
Q52: If a processed film appears stretched and distorted, which of the following is the likely cause?

A. the film was bent during placement

B. the film was reversed (placed backwards) during exposure

C. the film was exposed twice

D. the patient moved during exposure

Answer: the film was bent during placement

Q53: Of the following factors that influence the geometric characteristics of an image, which one is NOT
able to be changed by the operator?

A. target-receptor distance

B. object-receptor distance

C. film composition

D. focal spot size

E. object-receptor alignment

Answer: focal spot size

Q54: A periapical image shows maxillary central incisors which appear very short. Which of the following is
the likely cause?

A. vertical angulation is excessive/too steep

B. vertical angulation is insufficient/too flat

C. incorrect horizontal angulation

D. none of the above

Answer: vertical angulation is excessive/too steep


Q55: Following the exposure of film #1, the x-ray tubehead was moved and the beam was directed
mesially for exposure of film #2. Given this information and according to the buccal object rule, where is
the round object located?

A. lingual to the first molar

B. buccal to the first molar

C. in soft tissue

D. in bone

Answer: lingual to the first molar

Q56: A periapical image shows overlapped contacts. This error is cause by:

A. vertical angulation is excessive/too steep

B. vertical angulation is insufficient/too flat

C. incorrect horizontal angulation

D. beam not centered over receptor

E. poor receptor placement

Answer: incorrect horizontal angulation


Q57: Identify the cause of this panoramic image error seen below:

A. chin tipped too far upward

B. chin tipped too far downward

C. head tipped to one side

Answer: chin tipped too far downward


Q58: Identify the cause of this distorted periapical film seen below:

A. film bending

B. film creasing

C. phalangioma

D. double exposure

E. movement

Answer: film bending

Q59: Identify all that are advantages of using the paralleling technique.

A. receptor placement

B. comfort

C. accuracy

D. simplicity

E. duplication

Answer: • accuracy
• simplicity
• duplication
Q60: Identify all that are disadvantages of using the bisecting technique.

A. decreased exposure time

B. can be used without a beam alignment device

C. distortion

D. angulation problems

Answer: • distortion
• angulation problems

Q61: Identify which of the following is the correct vertical angulation for an exposure of a bite-wing receptor
using a tab.

A. −10 degrees, PID pointed down

B. −10 degrees, PID pointed up

C. +10 degrees, PID pointed down

D. +10 degrees, PID pointed up

Answer: +10 degrees, PID pointed down

Q62: A bite-wing image includes the crowns of maxillary and mandibular teeth, interproximal areas and
areas of crestal bone, all on the same image.

Bite-wing images are most useful for identifying interproximal at or just below the contact area on
premolars and molars.

A. both statements are true

B. both statements are false

C. the first statement is true, the second is false

D. the first statement is false, the second is true

Answer: both statements are true


Q63: The occlusal image may be used for the evaluation of periodontal disease.

The bisecting technique is the preferred periapical exposure method for the documentation of periodontal
diseases.

A. both statements are true

B. both statements are false

C. the first statement is true, the second is false

D. the first statement is false, the second is true

Answer: both statements are false

Q64: A patient of record has been in a fight where he was punched just below the right eye. A zygomatic
complex fracture is suspected. Which of the following extraoral projections is best for evaluating this type
of injury?

A. Waters projection

B. submentovertex projection

C. reverse Towne projection

D. lateral cephalometric projection

E. posteroanterior projection

Answer: submentovertex projection

Q65: A patient complains of swelling associated with a decayed upper left molar. The patient also
complains of “stuffiness” especially when she bends over to pick up anything. Which of the following is the
best projection for evaluation of the maxillary sinus area?

A. Waters projection

B. submentovertex projection

C. reverse Towne projection

D. lateral cephalometric projection

E. posteroanterior projection

Answer: Waters projection


Q67: A patient sustains an injury to the mandible. Which of the following is the best projection for
evaluation of the condylar neck and ramus area?

A. Waters projection

B. submentovertex projection

C. reverse Towne projection

D. lateral cephalometric projection

E. posteroanterior projection

Answer: reverse Towne projection

Q68: In dental practice, panoramic imaging is used for diagnosing the following problems except one.
Which one is the EXCEPTION?

A. temporomandibular joint pain

B. small carious lesions

C. jaw fractures

D. location of third molars

E. extensive dental or osseous lesions

Answer: small carious lesions

Q69: A panoramic image is viewed as if you are looking at the patient, with structures on the patient's right
side positioned on your left.

In panoramic radiography, when screen film is used as the image receptor, the use of calcium tungsten
screens is preferred over the use of rare earth screens.

A. both statements are true

B. both statements are false

C. the first statement is true, the second is false

D. the first statement is false, the second is true

Answer: the first statement is true, the second is false


Q70: Identify this extraoral image.

A. panaromic x-rays

B. lateral cephalometric projection

C. cone beam computed tomography

Answer: lateral cephalometric projection


Q71: Match the radiolucent areas labeled 1-3 on the image (left) with the description (right).

• 1.
• 2.
• 3.
• A. Nasopharyngeal airspace
• B. Glossopharyngeal airspace
• C. Palatoglossal airspace

Answer: 1C, 2A, 3B


Oral Surgery

Q1: In the normal patient, an increase in stress will lead to a/an _____ in ACTH production and large
doses of circulating systemic steroids will _____this production.

A. decrease, repress

B. increase, stimulate

C. increase, repress

D. decrease, stimulate

Answer: increase, repress

Q2: A person who has been on suppressive doses of steroids will? Select all that apply.

A. may show signs of hyperpigmentation

B. take as long as a month to regain full adrenal cortical function

C. take as long as a year to regain full adrenal cortical function

D. does not require consultation with a physician prior to surgery

Answer: • take as long as a year to regain full adrenal cortical function


• May show signs of hyperpigmentation

Q3: Hypercortisolemia can lead to ______, whereas hypocortisolemia can lead to______.

A. Cushing syndrome, Addison disease

B. Cushing syndrome, ectopic ACTH syndrome

C. Addison disease, Cushing syndrome

D. Addison disease, ectopic ACTH syndrome

Answer: Cushing syndrome, Addison disease

Q4: A 52-year-old woman requests removal of a painful mandibular second molar. She tells you that she
has not rested for 2 days and nights because of the pain. Her medical history is unremarkable, except that
she takes 20 mg of prednisone daily for erythema multiforme. How do you treat this patient?

A. instruct the patient to take 3 grams of amoxicillin 1 hour prior to extraction

B. give steroid supplementation and remove the tooth with local anesthesia and sedation

C. have patient discontinue the prednisone for 2 days prior to the extraction

D. no special treatment is necessary prior to extraction

Answer: give steroid supplementation and remove the tooth with local anesthesia and sedation
Q5: Sensory innervation to the palate is supplied by _____. Motor innervation to tensor veli palatini is
supplied by _____. A greater palatine block can anesthetize the palate as anterior as the maxillary _____.

A. CN V2, CN V3, first premolar

B. CN V3, CN V2, canine

C. CN V2, CN V3, canine

D. CN V3, CN V2, first premolar

Answer: CN V2, CN V3, first premolar

Q6: Please match the following cranial nerves with their appropriate preganglionic parasympathetic fibers.
Remember, postganglionic fibers from these nerves are carried via the opthalmic nerve (V1), maxillary
nerve (V2), and mandibular nerve (V3).

• 1. CN III Oculomotor
• 2. CN VII Facial
• 3. CN IX Glossopharyngeal
• A. Parotid Gland
• B. Pupil Constriction, Accomodation
• C. Lacrimal, Submandibular, Sublingual Gland

Answer: 1:B, 2:C, 3:A

Q7: The retrodiscal tissue of the TMJ is highly vascularized and innervated.

Only the extreme periphery of the articular disc is slightly innervated.

A. both statements are true

B. both statements are false

C. the first statement is true, the second is false

D. the first statement is false, the second is true

Answer: both statements are true

Q8: The average size of the maxillary sinus is 14.75 mL, with a range of 9.5-20 mL. On average, the width
is _____; the height is _____; and the depth is _____.

A. 1.0 cm, 2.75 cm, 2 cm

B. 2.5 cm, 3.75 cm, 3 cm

C. 4.0 cm, 4.75 cm, 4 cm

D. 5.0 cm, 5.75 cm, 3.5 cm

Answer: 2.5 cm, 3.75 cm, 3 cm


Q9: The _________ arises from the anterior surface of the external carotid artery and then passes near
the greater cornu of the hyoid bone.

A. submental artery

B. inferior alveolar artery

C. lingual artery

D. ascending pharyngeal artery

Answer: lingual artery

Q10: The pterygomandibular raphe, an important landmark for the administration of inferior alveolar block,
provides attachment to which of the following muscles? Select all that apply.

A. superior constrictor of the pharynx

B. buccinator

C. masseter

D. levator labii superioris

Answer: superior constrictor of the pharynx, buccinator

Q11: Which of the following are involved in the path for parasympathetic innervation of the parotid gland?
Select all that apply.

A. trigeminal nerve

B. glossopharyneal nerve

C. vagus nerve

D. otic ganglion

E. pterygopalatine ganglion

Answer: glossopharyneal nerve, otic ganglion

Q12: A dentist is performing a routine restoration on the left mandibular first molar. He is giving an inferior
alveolar nerve block injection, where he deposits anesthetic solution right next to the lingula and
mandibular foramen. Which ligament is most likely to get damaged?

A. sphenomandibular ligament

B. temporomandibular ligament

C. stylomandibular ligament

D. none of the above

Answer: sphenomandibular ligament


Q13: Which of the following injuries would cause a patient to deviate toward the side of injury when
protruding? Select all that apply.

A. damage to the lateral pterygoid muscle

B. ankylosis of the condyle

C. condylar hyperplasia

D. unilateral condylar fracture

Answer: • damage to the lateral pterygoid muscle


• ankylosis of the condyle
• unilateral condylar fracture

Q14: Which lymph nodes directly receive lymph from the tip of the tongue?

A. submental lymph nodes

B. submandibular lymph nodes

C. parotid lymph nodes

D. none of the above

Answer: submental lymph nodes

Q15: Which artery descends on the posterior surface of the maxilla and supplies the maxillary sinus and
the maxillary molar and premolar teeth?

A. sphenopalatine artery

B. greater palatine artery

C. posterior superior alveolar artery

D. infraorbital artery

Answer: posterior superior alveolar artery

Q16: The submandibular gland is a _____ gland whose secretomotor innervationcomes from the _____
nerve. The sublingual caruncle marks the opening of its duct into the oral cavity.

A. purely mucous, CN VII

B. mixed, CN IX

C. purely mucous, CN IX

D. mixed, CN VII

Answer: mixed, CN VII


Q17: The TMJ is a _____ joint whose articular disc is composed of _____ tissue. The TMJ is bound
anteriorly by the articular eminence anteriorly and posteriorly by the tympanic part of the temporal bone.

A. ginglymoarthrodial joint, fibrous connective

B. ginglymus joint, fibrocartilaginous

C. ginglymus joint, fibrous connective

D. ginglymoarthrodial joint, fibrocartilaginous connective

Answer: ginglymoarthrodial joint, fibrocartilaginous connective

Q18: When a maxillary third molar is displaced into the infratemporal fossa, it is usually displaced through
the periosteum and located _________ to the lateral pterygoid plate and __________ to the lateral
pterygoid muscle with displacement.

A. medial, inferior

B. medial, superior

C. lateral, inferior

D. lateral, superior

Answer: lateral, inferior

Q19: The carotid sheath contains all of the following EXCEPT one. Which one is the EXCEPTION?

A. carotid artery

B. sympathetic trunk

C. jugular vein

D. vagus nerve

Answer: sympathetic trunk

Q20: Which nerve may, in some cases, also serve as an afferent nerve for the mandibular first molar,
which needs to be considered when there is failure of the inferior alveolar local anesthetic block?

A. posterior superior alveolar nerve

B. glossopharyngeal nerve

C. facial nerve

D. mylohyoid nerve

Answer: mylohyoid nerve


Q21: The mandible is formed by _____ ossification and has synovial TMJ with fibrocartilage articular
surfaces. The _____ artery provides branches for the most direct blood supply of the TMJ.

A. intramembranous, internal carotid

B. intramembranous, external carotid

C. endochondral, internal carotid

D. endochondral, external carotid

Answer: intramembranous, external carotid

Q22: All of the following muscle/nerve combinations are correct EXCEPT one. Which one is the
EXCEPTION?

A. Trapezius muscle / accessory nerve

B. Stylopharyngeus muscle / glossopharyngeal nerve

C. Sternocleidomastoid muscle / vagus nerve

D. Cricothyroid muscle / superior laryngeal nerve

Answer: Sternocleidomastoid muscle / vagus nerve

Q23: After a stroke on the left side of the brain that affects the left upper motor neurons, the tongue
deviates to the:

A. left on protrusion

B. right on protrusion

C. neither of the above, the tongue would not be affected

Answer: right on protrusion — and the right half of the tongue will atrophy

Q24: The sublingual gland is located in the oral cavity between the mucosa of the oral cavity and the:

A. masseter muscle

B. mylohyoid muscle

C. buccinator muscle

D. temporalis muscle

Answer: mylohyoid muscle


Q25: The trigeminal ganglion is located:

A. superior to the deep lobe of the submandibular salivary gland

B. posterior surface of the maxillary tuberosity of the maxilla

C. anterior to the infraorbital foramen of the maxilla

D. at the apex of the petrous part of the temporal bone in the middle cranial fossa

Answer: at the apex of the petrous part of the temporal bone in the middle cranial fossa

Q26: The tongue receives its blood supply from all of the following EXCEPT one. Which one is the
EXCEPTION?

A. tonsillar branch of the facial artery

B. lingual artery

C. vertebral artery

D. ascending pharyngeal artery

Answer: vertebral artery

Q27: Which of the following nerves exits the skull through the foramen rotundum?

A. ophthalmic nerve (CN V1)

B. maxillary nerve (CN V2)

C. facial nerve

D. mandibular nerve (CN V3)

Answer: maxillary nerve (CN V2)

Q28: All of the following muscle/nerve pairings are correct EXCEPT one. Which one is the EXCEPTION?

A. lateral rectus muscle / abducens nerve

B. superior oblique muscle / trochlear nerve

C. medial rectus muscle / abducens nerve

D. inferior rectus muscle / oculomotor nerve

Answer: medial rectus muscle/abducens nerve


Q29: To extract a single mandibular central incisor, which of the following nerve block is considered the
most appropriate?

A. supraperiosteal nerve block

B. none of the above

C. lingual nerve block

D. posterior superior alveolar nerve block

Answer: supraperiosteal nerve block

Q30: The maxillary first molar is innervated by which of the following nerves? Select all that apply.

A. anterior superior alveolar

B. middle superior alveolar

C. posterior superior alveolar

D. greater palatine

E. ascending pharyngeal

Answer: middle superior alveolar, posterior superior alveolar

Q31: Shock is characterized by all of the following EXCEPT one. Which one is the EXCEPTION?

A. increased vascular resistance

B. bradycardia

C. myocardial ischemia

D. mental status changes

E. adrenergic response

F. anxiety, vomiting and diarrhea

G. oliguria

H. cool mottled skin

Answer: bradycardia
Q32: A full E cylinder of oxygen contains approximately:

A. 150 L at a pressure of 2000 psi

B. 300 L at a pressure of 2000 psi

C. 600 L at a pressure of 2000 psi

D. 750 L at a pressure of 2000 psi

Answer: 600 L at a pressure of 2000 psi

Q33: According to Guedel's stages of anesthesia, the proper use of nitrous oxide achieves which level of
anesthesia?

A. stage I

B. stage II

C. stage III

D. stage IV

Answer: stage I

Q34: All of the following drugs help to reduce salivary flow during dental treatment EXCEPT one. Which
one is the EXCEPTION?

A. scopolamine

B. atropine

C. local anesthesia

D. cevimeline HCL

E. benztropine

Answer: cevimeline HCL

Q35: Epinephrine and levonordefrin are added to local anesthetics because of their:

A. ability to increase the potency of the local anesthetic

B. ability to decrease the pain (burning) caused by the injection of the local anesthetic

C. vasoconstrictive properties

D. ability to decrease the possibility of an allergic reaction to the local anesthetic

Answer: vasoconstrictive properties


Q36: After receiving an injection of a local anesthetic containing 2% lidocaine with 1:100,000 epinephrine,
the patient loses consciousness. Which of the following is the most probable cause?

A. acute toxicity

B. allergic response

C. syncope

D. hyperventilation syndrome

Answer: syncope

Q37: Which tooth has a root that is NOT consistently innervated by the PSA nerve?

A. the maxillary first molar

B. the maxillary second molar

C. the maxillary third molar

D. all of the above

Answer: the maxillary first molar

Q38: When a vasoconstrictor is added to a local anesthetic agent, delay and reduction in peak blood levels
of the local anesthetic are observed.

Excessive blood levels of local anesthetics are known to cause systemic toxicity, especially in children.

A. both statements are true

B. both statements are false

C. the first statement is true, the second is false

D. the first statement is false, the second is true

Answer: both statements are true

Q39: Laryngospasm happens when the muscles of the vocal cords seize up, restricting the flow of air into
the lungs. It is a well known, infrequent but serious postsurgical complication. In the operating room, it is
treated by administering:

A. nitrous oxide

B. oxygen

C. epinephrine

D. enflurane

Answer: oxygen
Q40: In regard to nerve fibers, smaller nerve fibers are anesthetized _____ compared to larger nerve
fibers. Comparing nerves of the same size, _____ fibers are blocked first. And finally, nerve fibers with a
high firing rate are blocked _____than slow firing fibers.

A. sooner, myelinated, sooner

B. sooner, myelinated, later

C. later, myelinated, sooner

D. sooner, unmyelinated, later

Answer: sooner, myelinated, sooner

Q41: How will a larger than normal functional residual capacity affect nitrous oxide sedation?

A. nitrous oxide sedation will happen much quicker

B. nitrous oxide sedation will take longer

C. functional residual capacity does not affect nitrous oxide sedation

Answer: nitrous oxide sedation will take longer

Q42: Which of the following correctly describe barbiturates? Select all that apply.

A. not lipid soluble

B. moderately lipid soluble

C. very lipid soluble

D. delayed onset of action

E. rapid onset of action

Answer: very lipid soluble, rapid onset of action

Q43: Which of the following local anesthetics are available in North America? Select all that apply.

A. prilocaine

B. bupivacaine

C. procaine

D. lidocaine

E. tetracaine

F. articaine

Answer: • prilocaine, lidocaine, bupivacaine, articaine


Q44: Syncope is a transient loss of consciousness that most likely occurs in a sedated patient.

Most common incidents of fainting occur on a dental chair, when patient is lying in a supine position.

A. both statements are true

B. both statements are false

C. the first statement is true, the second is false

D. the first statement is false, the second is true

Answer: both statements are false

Q45: For local anesthetics, for every 1% solution there is:

A. 0.10 mg/mL of anesthetic

B. 1 mg/mL of anesthetic

C. 10 mg/mL of anesthetic

D. 100 mg/mL of anesthetic

Answer: 10 mg/mL of anesthetic

Q46: All of the following are needed in combination to produce neurolept-anesthesia EXCEPT one. Which
one is the EXCEPTION?

A. narcotic analgesic

B. benzodiazepine

C. neuroleptic agent

D. nitrous oxide

Answer: benzodiazepine

Q47: The most common cause of loss of consciousness in the dental office is:

A. anaphylaxis

B. syncope

C. heart attack

D. seizure

Answer: syncope — fainting


Q48: Trauma to muscles or blood vessels in the ______________ is the most common etiological factor in
trismus associated with dental injections of local anesthetics.

A. pterygoid fossa

B. temporal fossa

C. submandibular fossa

D. infratemporal fossa

Answer: infratemporal fossa

Q49: There are no contraindications for the use of nitrous oxide sedation in asthmatic patients.

Because anxiety is a stimulus for an asthmatic attack, nitrous oxide sedation is actually beneficial for these
patients.

A. both statements are true

B. both statements are false

C. the first statement is true, the second is false

D. the first statement is false, the second is true

Answer: both statements are true

Q50: A normal platelet count is:

A. 15,000–45,000/mm

B. 75,000–100,000/mm

C. 150,000–450,000/mm

D. 450,000–600,000/mm

Answer: 150,000–450,000/mm

Q51: Which of the following pairings are correct regarding the amount of epinephrine in 1.7 cc of solution?
Select all that apply.

A. 2% lidocaine 1:200,000 / .0085 mg epinephrine

B. 2% lidocaine 1:200,000 / .017 mg epinephrine

C. 2% lidocaine 1:50,000 / .034 mg epinephrine

D. 2% lidocaine 1:50,000 / .017 mg epinephrine

Answer: 2% lidocaine 1:200,000/.0085 mg epinephrine, 2% lidocaine 1:50,000/.034 mg epinephrine


Q52: The primary site of biotransformation of amide drugs is in the _____, whereas the biotransformation
of ester anesthetics is in the _____.

A. kidney, blood plas ma

B. liver, liver

C. blood plasma, liver

D. liver, blood plasma

Answer: liver, blood plasma

Q53: The initial clinical signs and symptoms of CNS toxicity for local anesthetics are usually excitatory in
nature. However, it is also possible that the excitatory phase of the reaction may be extremely brief or may
not occur at all. This is true especially with which local anesthetics? Select all that apply.

A. lidocaine

B. tetracaine

C. etidocaine

D. procaine

E. bupivacaine

Answer: lidocaine, procaine

Q54: The pH of normal tissue is _______ ; the pH of an inflamed area is _______.

A. 9.0; 3 to 4

B. 7.4; 5 to 6

C. 3.6; 8 to 9

D. 8.0; 2 to 3

Answer: 7.4; 5 to 6

Q55: The maximum nitrous oxide to oxygen ratio that can be safely administered to a patient is:

A. 50%, 50%

B. 40%, 60%

C. 60%, 40%

D. 80%, 20%

Answer: 60%, 40%


Q56: The primary action of local anesthetics in producing a conduction block is to decrease the
permeability of the ion channels to:

A. calcium ions

B. chloride ions

C. potassium ions

D. sodium ions

Answer: sodium ions

Q57: Which of the following is the phase of anesthesia that begins with the administration of anesthetic
and continues until the desired level of patient unresponsiveness is reached?

A. induction

B. maintenance

C. recovery

Answer: induction

Q58: Volatile liquids require a vaporizer for inhalational administration. Which one additionally requires a
heating component to allow delivery at room temperature?

A. enflurane

B. halothane

C. sevoflurane

D. desflurane

E. isoflurane

Answer: desflurane

Q59: The optimum site for IV sedation for an outpatient is the:

A. median basilic vein

B. median cephalic vein

C. median antebrachial vein

D. axillary vein

Answer: median cephalic vein


Q60: Ketamine, a phencyclidine (PVP) derivative, is 10 times more lipid soluble than thiopental, enabling it
to cross the blood-brain barrier quickly.

It produces dissociative anesthesia, which can be seen on EEG as dissociation between the thalamus and
limbic system.

A. both statements are true

B. both statements are false

C. the first statement is true, the second is false

D. the first statement is false, the second is true

Answer: both statements are true

Q61: Malignant hyperthermia (MH) is a hypermetabolic state involving skeletal muscle that is precipitated
by certain anesthetic agents in genetically susceptible individuals. The incidence of MH is <0.5% of all
patients who are exposed to anesthetic agents. The major clinical characteristics of MH are all of the
following EXCEPT one. Which one is the EXCEPTION?

A. acidosis

B. rigidity

C. fever

D. myoglobinuria

E. hypermetabolism

F. hypocapnea

Answer: hypocapnea

Q62: The following signs: nausea, pallor, cold perspiration, widely dilated pupils, eyes rolled up, and brief
convulsions are indicative of a patient having a________ reaction.

A. somatogenic

B. psychogenic

C. either of the above

D. none of the above

Answer: psychogenic
Q63: Postoperative hypotension is usually due to the effect of:

A. transfusion reactions

B. a fat embolism

C. the anesthetic or analgesics on the myocardium

D. liver failure

Answer: the anesthetic or analgesics on the myocardium

Q64: Anesthesia performed with general anesthetics occurs in four stages which may or may not be
observable because they can occur very rapidly. Which stage is the one in which blood pressure rises and
becomes irregular, and breathing rate increases?

A. analgesia

B. excitement

C. surgical anesthesia

D. medullary paralysis

Answer: excitement

Q65: All of the following are contraindications for the use of nitrous oxide in a patient EXCEPT one. Which
one is the EXCEPTION?

A. gastrointestinal obstructions

B. middle ear disturbances

C. possibly pneumothorox

D. asthmatic patient

Answer: asthmatic patient

Q66: The propylene glycol in IV valium can cause:

A. cellulitis

B. a unilateral facial paralysis

C. phlebitis

D. syncope

Answer: phlebitis
Q67: When a biopsy is being performed, the incisions should be:

A. oriented perpendicular to lines of muscle tension

B. oriented parallel to lines of muscle tension

C. as deep as possible into muscle fibers beneath the lesion

D. at a 45-degree angle to the long axis of any muscle fibers beneath the lesion

Answer: oriented parallel to lines of muscle tension

Q68: How long should one wait before obtaining a biopsy of an oral ulcer?

A. 4 days

B. 7 days

C. 14 days

D. 30 days

Answer: 14 days — 2 weeks

Q69: An incisional biopsy is indicated for which of the following lesions?

A. 3 x 3 mm well-encapsulated fibroma

B. necrotizing sialometaplasia of the hard palate

C. 2 x 2 mm papilloma of left commissure of lips

D. 3 x 2 mm melanotic macule

Answer: necrotizing sialometaplasia of the hard palate

Q70: All of the following symptoms EXCEPT one suggest that your patient is dehydrated. Which one is the
EXCEPTION?

A. pale or gray skin color

B. dry mouth

C. decreased skin turgor

D. modified state of consciousness

E. high blood pressure

F. rapid pulse

G. reduced urine output

Answer: high blood pressure


Q71: A fasting level above _____ or nonfasting glucose > _____ is indicative of diabetes.

A. 50 mg/dL, 125 mg/dL

B. 75 mg/dL, 150 mg/dL

C. 100 mg/dL, 175 mg/dL

D. 140 mg/dL, 200 mg/dL

Answer: 140 mg/dL, 200 mg/dL

Q72: The most common sign of left-sided heart failure is _____, whereas _____ is the most common sign
of right-sided heart failure.

A. low blood pressure, high blood pressure

B. pulmonary edema, peripheral edema

C. kidney failure, liver failure

D. peripheral edema, pulmonary edema

Answer: pulmonary edema, peripheral edema


Q73: Match the term on the left with the correct meaning on the right.

• Apnea
• Dyspnea
• Hypercapnia
• Hyperpnea
• Hyperventilation
• Hypocapnia
• Hypoventilation
• Respiratory arrest
• A reduced rate and depth of respiration
• An increase in both rate and depth of respiration
• Below normal CO2 in arterial blood
• Excss CO2 in arterial blood
• Increase in depth of respiration
• Permanent cessation of breathing (unless corrected)
• The unpleasant sensation of difficulty in breathing
• Transient cessation or absence of breathing

Answer: • Apnea: Transient cessation or absence of breathing


• Hypercapnia: Excess CO2 in arterial blood
• Hypocapnia: Below normal CO2 in arterial blood
• Dyspnea: The unpleasant sensation of difficulty in breathing
• Hyperpnea: Increase in depth of respiration
• Respiratory arrest: Permanent cessation of breathing (unless corrected)
• Hyperventilation: An increase in both rate and depth of respiration
• Hypoventilation: A reduced rate and depth of respiration

Q74: _______ occurs when air leaks into the pleural space causing the lung to recoil from the chest wall

A. bronchiectasis

B. atelectasis

C. pneumothorax

D. pneumonia

Answer: pneumothorax
Q75: The "blue bloater” suffers from:

A. viral pneumonia

B. chronic bronchitis

C. emphysema

D. asthma

Answer: chronic bronchitis

Q76: Which of the following acid-base abnormalities will develop in a patient with recurrent vomiting of
gastric contents?

A. respiratory acidosis

B. respiratory alkalosis

C. metabolic acidosis

D. metabolic alkalosis

Answer: metabolic alkalosis

Q77: A patient undergoing dental extraction at your office accidently aspirates the extracted tooth. Even
after repeated coughing, the extracted tooth is not expelled out. Provided the patient remains conscious,
what will be the next step you will institute to manage the aspirated tooth?

A. instruct repeated coughing

B. begin abdominal thrusts

C. transport to emergency room

D. perform Heimlich maneuvers until effective

Answer: perform Heimlich maneuvers until effective

Q78: The three basic pathophysiological changes that occur in an asthmatic patient include all of the
following EXCEPT one. Which one is the EXCEPTION?

A. airway inflammation

B. airway obstruction

C. irreversible airway narrowing

D. airway hyperresponsiveness

Answer: irreversible airway narrowing


Q79: Hemophilia A is the most common type of hemophilia present in North America and involves which
coagulation factor?

A. VI

B. VIII

C. XI

D. IX

Answer: VIII

Q80: A history of rheumatic fever, IV drug abuse, or heart murmur should alert the dentist to the possibility
of:

A. diabetes mellitus

B. AIDS

C. valvular disease

D. end-stage renal disease

Answer: valvular disease

Q81: A tall, thin patient presents to your office with shortness of breath. On examination, you note the
patient is breathing through “pursed” lips, his expiratory phase is prolonged, and lung sounds are distant.
Which of the following is the most likely diagnosis?

A. asthma

B. bronchiectasis

C. cystic fibrosis

D. emphysema

Answer: emphysema
Q82: Special considerations must be taken when treating a patient on renal dialysis. Which of the following
should be considered? Select all that apply.

A. treat the day before dialysis

B. treat the day after dialysis

C. NSAIDs are the best analgesic to use

D. morphine is acceptable for use as an analgesic

E. be aware of shunts when taking the patient's blood pressure

F. consider that the patient may be on steroid therapy

Answer: • treat the day after dialysis


• be aware of shunts when taking the patient's blood pressure
• consider that the patient may be on steroid therapy
Q83: Codeine, a widely used analgesic in dentistry:

A. is a natural constituent of opium

B. may be given only by injection

C. has a calming effect on gastric mucosa

D. is stronger than morphine, more addictive, and more constipating

Answer: is a natural constituent of opium

Q84: Which of the following is NOT a correct statement regarding lidocaine?

A. its protein binding capacity is equivalent to bupivacaine

B. it is metabolized in the liver by CYP3A4 enzyme

C. its hepatic extraction ratio is higher than both bupivacaine and ropivacaine

D. it reversibly blocks nerve conduction

Answer: its protein binding capacity is equivalent to bupivacaine

Q85: Which of the following drugs would be BEST given to a patient with a history of gastric ulcers?

A. aspirin

B. ibuprofen

C. acetaminophen

D. naproxen

Answer: acetaminophen

Q86: Which of these barbiturates can be classified as a long-acting compound?

A. amobarbital

B. thiopental

C. phenobarbital

D. pentobarbital

Answer: phenobarbital
Q87: ______ should be used cautiously in the elderly. It should never be given to patients on monoamine
oxidase inhibitors for psychiatric disease and is generally contraindicated in patients receiving phenytoin
(Dilantin) for seizure disorders.

A. ibuprofen

B. acetaminophen

C. meperidine

D. codeine

Answer: meperidine

Q88: Atropine and scopolamine have similar pharmacologic effects. Which of the following actions do they
share?
Select all that apply:

A. reduction of salivation

B. prevention of cardiac slowing during general anesthesia

C. CNS depression

D. mydriasis

E. cycloplegia

Answer: • mydriasis
• cycloplegia
• reduction of salivation
• prevention of cardiac slowing during general anesthesia

Q89: A sedative dose of a barbiturate should be expected to produce:

A. respiratory depression

B. minor analgesia

C. decreased BMR

D. all of the above effects

E. none of the above effects

Answer: none of the above effects


Q90: All of the following are true statements concerning the principles of suturing technique EXCEPT one.
Which one is the EXCEPTION?

A. the needle should be perpendicular when it enters the tissue

B. sutures should be placed at an equal distance from the wound margin (2-3 mm) and at equal depths

C. sutures should be placed from mobile tissue to thick tissue

D. sutures should be placed from thin tissue to thick tissue

E. sutures should not be overtightened

F. tissues should be closed under tension

G. sutures should be 2-3 mm apart

H. the suture knot should be on the side of the wound

Answer: tissues should be closed under tension

Q91: What areas are impacted maxillary third molars occasionally displaced into? Select all that apply?

A. canine space

B. pterygomaxillary space

C. infratemporal space

D. pharyngeal space

E. maxillary sinus

Answer: infratemporal space, maxillary sinus

Q92: For maxillary extractions, the upper jaw of the patient should be:

A. below the height of the operator's shoulder

B. above the height of the operator's shoulder

C. at the same height of the operator's shoulder

D. it makes no difference where the patient's upper jaw is in relation to the operator's shoulder

Answer: at the same height of the operator's shoulder


Q93: Which of the following technique is considered most appropriate for the removal of intraseptal bone
after a dental extraction?

A. Dean’s technique

B. simple alveoplasty

C. ridge split osteoplasty

D. all of the above

Answer: Dean’s technique

Q94: All of the following are ways of eliminating dead space EXCEPT one. Which one is the EXCEPTION?

A. close the wound in layers to minimize the postoperative void

B. apply pressure dressings

C. use drains to remove any bleeding that accumulates

D. allow the void to fill with blood so that a blood clot will form

Answer: allow the void to fill with blood so that a blood clot will form

Q95: In which of the following maxillary teeth during extraction, exerting rotational force is useful in
expanding the tooth socket?

A. canine

B. lateral incisor

C. second premolar

D. all of the above

Answer: canine

Q96: During extraction of a maxillary third molar, you realize the tuberosity has also been extracted. What
is the proper treatment in this case?

A. remove the tuberosity from the tooth and reimplant the tuberosity

B. smooth the sharp edges of the remaining bone and suture the remaining soft tissue

C. no special treatment is necessary

D. none of the above

Answer: smooth the sharp edges of the remaining bone and suture the remaining soft tissue
Q97: Which of the following can be safely excised in preparing the edentulous mandible for dentures?
Select all that apply.

A. labial frenum

B. lingual frenum

C. mylohyoid ridge

D. genial tubercles

E. exostosis

Answer: labial frenum, lingual frenum, mylohyoid ridge, exostosis

Q98: The ideal time to remove impacted third molars is:

A. when the root is fully formed

B. when the root is approximately two-thirds formed

C. makes no difference how much of the root is formed

D. when the root is approximately one-third formed

Answer: when the root is approximately two-thirds formed

Q99: All of the following are normal postextraction procedures EXCEPT one. Which one is the
EXCEPTION?

A. digital compression of alveolus after extraction

B. suture placement when gingival papillae have been excised or there is excessive bleeding

C. patient is instructed to bite down on a pressure dressing for 30-60 minutes

D. printed post-operative instructions are given to the patient

E. patient is told they can return to normal smoking habits immediately following the extractions

Answer: patient is told they can return to normal smoking habits immediately following the
extractions

Q100: The most commonly impacted teeth are the mandibular third molars, maxillary third molars, and the:

A. maxillary canines

B. maxillary lateral incisors

C. mandibular first molars

D. mandibular premolars

Answer: maxillary canines


Q101: All of the following are cardinal signs of a localized osteitis (dry socket) EXCEPT one. Which one is
the EXCEPTION?

A. throbbing pain (often radiating)

B. bilateral lymphadenopathy

C. fetid odor

D. bad taste

E. poorly healed extraction site

Answer: bilateral lymphadenopathy

Q102: Before removing a palatal torus:

A. an intraoral picture should be taken

B. a mandibular torus, if present, should be removed

C. a stent should be fabricated

D. a biopsy should be taken

Answer: a stent should be fabricated

Q103: For impacted mandibular third molars, place the following in their correct order from the least
difficult to most difficult to remove.

• distoangular
• horizontal
• mesioangular
• vertical

Answer: 1. mesioangular - 43% of mandibular impactions


2. horizontal - 3% of mandibular impactions
3. vertical - 38% of mandibular impactions
4. distoangular - 6% of mandibular impactions

Q104: Which two major forces are used for routine tooth extractions?

A. rotation

B. pulling

C. pushing

D. luxation

Answer: rotation, luxation


Q105: The root of which tooth is most often dislodged into the maxillary sinus during an extraction
procedure?

A. palatal root of the maxillary first premolar

B. palatal root of the maxillary first molar

C. palatal root of the maxillary second molar

D. palatal root of the maxillary third molar

Answer: palatal root of the maxillary first molar

Q106: The Caldwell-Luc procedure eliminates blind procedures and facilitates the recovery of large root
tips or entire teeth that have been displaced into the maxillary sinus. When performing this procedure, an
opening is made into the facial wall of the antrum above the:

A. maxillary tuberosity

B. maxillary lateral incisor

C. maxillary premolar roots

D. maxillary third molar

Answer: maxillary premolar roots

Q107: Which suture grading below is the thinnest?

A. 2/0

B. 3/0

C. 4/0

D. 5/0

Answer: 5/0

Q108: Which of the following events are correctly paired with the stages of wound healing? Select all that
apply.

A. fibroblasts lay a bed of collagen / proliferative phase

B. platelet aggregation / inflammatory phase

C. contraction of the wound / remodeling phase

D. thromboplastin makes a clot / inflammatory phase

Answer: • fibroblasts lay a bed of collagen / proliferative phase


• platelet aggregation / inflammatory phase
• thromboplastin makes a clot / inflammatory phase
Q109: Please match the following resorbable suture types with the approximate time period of resorption in
the mouth.

• 1. Plain Catgut
• 2. Chromic Sutures
• 3. Dexon (synthetic)
• A. 30 days
• B. 8 days
• C. 12-15 days

Answer: 1B, 2C, 3A

Q110: Regardless of the flap design used, certain principles should be followed while incising and
reflecting the gingiva. With this in mind, the termination of a
vertical incision at the gingival crest must be:

A. midbuccal of the tooth

B. at the line angle of the tooth

C. midlingual of the tooth

D. beyond the depth of the mucobuccal fold

Answer: at the line angle of the tooth

Q111: While attempting to remove a grossly decayed mandibular molar, the crown fractures. What is the
recommended next step to facilitate the removal of this tooth?

A. use a larger forceps and luxate remaining portion of tooth to the lingual

B. separate the roots

C. irrigate the area and proceed to remove the rest of the tooth

D. place a sedative filling and reschedule patient

Answer: separate the roots

Q112: While extracting a mandibular third molar, you notice that the distal root tip is missing. Where is it
most likely to be found?

A. in the infratemporal fossa

B. in the submandibular space

C. in the mandibular canal

D. in the pterygopalatine fossa

Answer: in the submandibular space


Q113: Arrange the following five phases of healing of an extraction site in their correct order.

• hemorrhage and clot formation


• organization of the clot by granulation tissue
• recontouring of the alveolar bone and bone maturation
• replacement of granulation tissue by connective tissue and epithelialization of the site
• replacement of the connective tissue by fibrillar bone

Answer: 1. hemorrhage and clot formation


2. organization of the clot by granulation tissue
3. replacement of granulation tissue by connective
tissue and epithelialization of the site
4. replacement of the connective tissue by fibrillar bone
5. recontouring of the alveolar bone and bone maturation

Q114: If a subcondylar fracture occurs, which of the following muscles will displace the condyle both
anteriorly and medially?

A. digastric muscle

B. temporalis muscle

C. lateral pterygoid muscle

D. medial pterygoid muscle

Answer: lateral pterygoid muscle

Q115: ___________ are second only to nasal fractures in frequency of involvement.

A. LeFort I

B. LeFort II

C. LeFort III

D. zygomatic fractures

Answer: zygomatic fractures

Q116: The most frequent complication associated with mandibular fracture management is:

A. hematoma

B. wound dehiscence

C. facial or trigeminal nerve injury

D. infection

Answer: infection
Q117: What determines whether muscles will displace fractured segments from their original position?

A. attachment of the muscle

B. type of fracture

C. direction of muscle fibers

D. line of fracture

Answer: line of fracture

Q118: In general, mandibular fractures are less common in children than in adults. When mandibular
fractures occur in children, __________ fractures of the mandible, particularly in the condylar region, are
relatively common.

A. simple

B. greenstick

C. compound

D. comminuted

Answer: greenstick
Q119: Computed tomography (CT) scan is the gold standard for evaluation of which of the following?
Select all that apply.

A. mandibular fractures at the angle

B. fractures of the mandibular condyle

C. LeFort I fractures

D. zygomatic fractures

Answer: • mandibular fractures at the angle


• fractures of the mandibular condyle
• LeFort I fractures
• zygomatic fractures

Q120: Closed reduction is best used in the treatment of:

A. favorable, non-displaced fractures

B. displaced and unstable fractures, with associated midface fractures, and when MMF is
contraindicated

C. either of the above

D. none of the above

Answer: favorable, non-displaced fractures


Q121: The process of fracture healing can occur in:

A. one way: by direct or primary bone healing which occurs without callus formation

B. one way: by indirect or secondary bone healing which occurs with a callus precursor stage

C. two ways: by direct or primary bone healing, which occurs without callus formation, and indirect or
secondary bone healing, which occurs with a callus precursor stage

Answer: two ways: by direct or primary bone healing, which occurs without callus formation, and
indirect or secondary bone healing, which occurs with a callus precursor stage

Q122: The most common pathognomonic sign of a mandibular fracture is:

A. nasal bleeding

B. exophthalmos

C. malocclusion

D. numbness in the infraorbital nerve distribution

Answer: malocclusion

Q123: Which type of LeFort fracture is often referred to as a transmaxillary fracture?

A. LeFort I

B. LeFort II

C. LeFort III

Answer: LeFort I

Q124: A patient presents to your office after a skateboarding accident with complaint of an anterior open
bite and deviation of his mandible to the right side. Based on what you know and anatomic distribution of
mandibular fractures, you suspect the patient has a (an) _____ fracture on the _____ side.

A. condylar, left

B. angle, left

C. angle, right

D. condylar, right

E. symphysis, right

Answer: condylar, right


Q125: Patients with hypocalcemia have an ionized calcium level below 2.0 or serum calcium concentration
lower than 9 mg/dL. Some of the most common causes are:

A. hyperparathyroidism and cancer

B. diabetes and hypothyroidism

C. renal failure and hypoalbuminemia

D. Graves disease and hypopituitarism

Answer: renal failure and hypoalbuminemia

Q126: Whether a bone cyst or other cysts are completely enucleated or treated by marsupialization
depends on the:

A. duration

B. origin

C. color

D. size and location to vital structures

Answer: size and location to vital structures

Q127: At what point should the EMS be activated with adult victims?

A. after 1 minute of CPR

B. after 2 minutes of CPR

C. after 3 minutes of CPR

D. immediately when an adult is found to be unresponsive

Answer: immediately when an adult is found to be unresponsive

Q128: A patient presents to your office with a fasting blood glucose of 150 mg/dL. Based on what you
know, you suspect the patient might have an underlying diabetic condition because the normal range of a
fasting blood glucose is:

A. 70-110 mg/dL

B. 90-125 mg/dL

C. 70-90 mg/dL

D. 110-125 mg/dL

Answer: 70-110 mg/dL


Q 129: Which phase of hemostasis is most affected by patients taking an anticoagulant such as warfarin?

A. vascular phase

B. platelet phase

C. coagulation phase

Answer: coagulation phase

Q130: When a child less than 8 years of age has a pulse but is breathless, what is the recommended rate
of rescue breathing?

A. once every 3 seconds

B. once every 5 seconds

C. once every 8 seconds

D. once every 10 seconds

Answer: once every 3 seconds - (20 breaths/min)

Q131: When a tissue flap is raised, a “buttonhole” near the edge of the flap creates compromised blood
supply on the side of hole away from the flap base.

While incising a tissue layer which is to be suture closed, an oblique incision should be made with the
blade to create squared wound edges.

A. both statements are correct

B. both statements are incorrect

C. the first statement is incorrect, the second is correct

D. the first statement is correct, the second is incorrect

Answer: the first statement is correct, the second is incorrect

Q132: What is the most frequent cause of airway obstruction in an unconscious person?

A. chewing gum

B. cigarette

C. tongue

D. hard candy

Answer: tongue
Q133: Which of the following is a calculated value developed to normalize the reporting of prothrombin
time (PT)?

A. IMR

B. IGR

C. ITR

D. INR

Answer: INR

Q134: Which of the following is the most common error in blood pressure measurement?

A. applying the blood pressure cuff too tightly

B. applying the blood pressure cuff too loosely

C. overinflating the blood pressure cuff

D. underinflating the blood pressure cuff

E. the use of too large or too small cuffs

Answer: the use of too large or too small cuffs

Q135: Which of the following is the gold standard for bone regenerative grafting materials for several
reasons, including the capability to support osteogenesis and having osteoinductive and osteoconductive
properties?

A. xenogenic bone

B. allogeneic bone

C. autogenous bone

D. alloplastic bone

Answer: autogenous bone

Q136: Allogeneic grafts such as freeze-dried bone displays which of the following properties? Select all
that apply.

A. osteoconductive

B. osteogenic

C. osteoinductive

Answer: osteoconductive
Q137: Which of the following refers to a horizontal osteotomy of the anterior mandible?

A. blepharoplasty

B. genioplasty

C. cervicofacial rhytidectomy

D. rhinoplasty

Answer: genioplasty

Q138: Match the term on the left with the correct description on the right.

• Allograft
• Autograft
• Isograft
• Xenograf
• Tissue removed from an animal donor and surgically transplanted to a human
• Tissue surgically removed from one area of a person's body, such as the iliac crest, and transplanted in
another site on the same person
• Tissue surgically transplanted from an individual of the same species
who is genetically related to the recipient
• Tissue surgically transplanted from one individual to a genetically nonidentical individual of the same
species

Answer: • Autograft: Tissue surgically removed from one area of a person's body, such
as the iliac crest, and transplanted in another site on the same person
• Allograft: Tissue surgically transplanted from one individual to a genetically
non-identical individual of the same species
• Xenograft: Tissue removed from an animal donor and surgically transplanted to a human
• Isograft: Tissue surgically transplanted from an individual of the same species
who is genetically related to the recipient

Q139: The term alloplastic is synonymous with:

A. original

B. natural

C. synthetic

D. genuine

Answer: synthetic
Q140: In reference to the bone-implant interface, which of the following yields the most predictable long-
term stability?

A. fibro-osseous integration

B. osseointegration

C. biointegration

Answer: osseointegration

Q141: Which of the following is found between the bone and implant of an endosseous dental implant?

A. periodontal ligament

B. peri-implant ligament

C. epithelial ligament

D. a bone-implant interface

Answer: a bone-implant interface

Q142: Which area of the mouth has a the highest failure rate in osseointegration of dental implants?

A. anterior mandible

B. posterior mandible

C. anterior maxilla

D. posterior maxilla

Answer: posterior maxilla

Q143: Currently, the most popular used implants are:

A. blade form implants

B. subperiosteal implants

C. transosseous implants

D. root form implants

Answer: root form implants


Q144: To prevent infective endocarditis in patients at risk for such infections, the American Heart
Association (AHA) frequently issues guidelines for prophylactic antibiotic coverage during dental
procedures. In accordance with the most recently revised AHA guidelines, which of the following are
acceptable antibiotic options for the prevention of infective endocarditis? Select all that apply.

A. cephalexin

B. amoxicillin

C. clarithromycin

D. erythromycin

E. azithromycin

F. clindamycin

Answer: cephalexin, amoxicillin, clarithromycin, azithromycin, clindamycin

Q145: The roots of the third, second, and first molars are all below the level of the mylohyoid. Infection of
these teeth pass through the root, directly into the____________ and then to the lateral pharyngeal space.

A. buccal space

B. canine space

C. infratemporal space

D. submaxillary space

Answer: submaxillary space

Q146: The mandibular left second molar of a 14-year-old boy is unerupted. Radiographs show a small
dentigerous cyst surrounding the crown. What is the treatment of choice?

A. surgically extract the unerupted second molar

B. uncover the crown and keep it exposed

C. prescribe an antiinflammatory medication and schedule a follow-up appointment in 6 months

D. no treatment is necessary at this time

Answer: uncover the crown and keep it exposed


Q147: All of the following are considered primary fascial spaces EXCEPT one. Which one is the
EXCEPTION?

A. buccal

B. canine

C. submaxillary

D. masticator

E. vestibular

Answer: masticator

Q148: Body temperature can be measured in several different ways, which one is the least accurate?

A. orally

B. axillary

C. rectally

D. aurally

Answer: axillary

Q149: Osteomyelitis usually begins in the medullary space involving the _________.

A. periosteum

B. soft tissues

C. cortical bone

D. cancellous bone

Answer: cancellous bone

Q150: Which conditions would require preoperative antibiotic prophylaxis for the prevention of bacterial
endocarditis? Select all that apply.

A. prosthetic heart valve

B. complex cyanotic congenital heart disease

C. prior coronary artery bypass graft

D. surgically constructed systemic pulmonary shunts or conduits

E. mitral valve prolapse with regurgitation and/or thickened leaflets

Answer: • prosthetic heart valve


• complex cyanotic congenital heart disease
• surgically constructed systemic pulmonary shunts or conduits
Q151: Why is a conventional handpiece that expels forced air contraindicated when performing
dentoalveolar surgery?

A. too much bone will be removed

B. these handpieces can cause tissue emphysema or an air embolus, which can be fatal

C. these handpieces are not high-powered enough to remove bone

D. all of the above

Answer: these handpieces can cause tissue emphysema or an air embolus, which can be fatal

Q152: Which one of the following is an exception to the intracapsular disorders of the temporomandibular
joint?

A. degenerative joint disease

B. anterior disk displacement with reduction

C. anterior disk displacement without reduction

D. pseudoankylosis

Answer: pseudoankylosis

Q153: Incision for drainage (I&D) in an area of acute infection should only be performed after:

A. a culture for antibiotic sensitivity has been performed

B. localization of the infection

C. a sinus tract is formed

D. the patients fever has cleared up

Answer: localization of the infection

Q154: Which one of the following techniques is advantageous in the excision of labial frenal attachments
that have a wide base?

A. Z-plasty

B. localized vestibuloplasty with secondary epithelization

C. simple excision technique

D. Dean’s technique

Answer: localized vestibuloplasty with secondary epithelization


Q155: An orofacial infection can reach the cavernous sinus through two routes: an anterior route via the
______ and ________, and a posterior route via the_________ and the ______.

A. transverse facial vein; pterygoid plexus of veins, angular; inferior ophthalmic veins

B. inferior alveolar, anterior superior alveolar arteries, descending palatine; greater palatine arteries

C. supratrochlear; supraorbital veins, superficial temporal; lingual veins

D. angular; inferior ophthalmic veins, transverse facial vein; pterygoid plexus of veins

Answer: angular; inferior ophthalmic veins, transverse facial vein; pterygoid plexus of veins

Q156: A surgical procedure used to recontour the supporting bone structures in preparation of a complete
or partial denture is called a/an:

A. closed reduction

B. operculectomy

C. alveoloplasty

D. gingivoplasty

Answer: alveoloplasty

Q157: Which of the following can be considered as an alternative treatment option for non-salvageable
impacted maxillary canines?

A. implant-supported crown procedure

B. retention of the primary canine

C. bicuspid substitution

D. all of the above

Answer: all of the above

Q158: All of the following are systemic contraindications to elective surgery EXCEPT one. Which one is
the EXCEPTION?

A. blood dyscrasias (i.e., hemophilia, leukemia)

B. controlled diabetes mellitus

C. Addison disease or any steroid deficiency

D. fever of unexplained origin

E. nephritis

F. any debilitating disease

G. cardiac disease

Answer: controlled diabetes mellitus


Q159: _____ is the most common type of oral cancer and _____ is the most common site.

A. SCC, buccal mucosa

B. SCC, tongue and floor of the mouth

C. acinic cell carcinoma, palate

D. acinic cell carcinoma, attached gingiva

Answer: SCC, tongue and floor of the mouth

Q160: The universal sign of laryngeal obstruction is:

A. mydriasis

B. stridor (crowing sounds)

C. sweating

D. tachycardia

Answer: stridor (crowing sounds)

Q161: Pericoronitis is acute inflammation of the tissue overlying and surrounding a partially erupted or
erupting tooth. The most commonly involved tooth is a:

A. maxillary third molar

B. maxillary second molar

C. mandibular third molar

D. mandibular second molar

Answer: mandibular third molar — See picture below

Q162: Bleeding that consists of pinpoint dots of blood is called ______. Larger flat areas where blood has
collected under the tissue, up to a centimeter in diameter, are called ______. A very large area is called
a/an ______.

A. purpura, petechiae, ecchymosis

B. petechiae, ecchymosis, purpura

C. ecchymosis, purpura, petechiae

D. petechiae, purpura, ecchymosis

Answer: petechiae, purpura, ecchymosis


Q163: Thrombocytopenia (low platelet count) that is less than ______ is an absolute contraindication to
elective surgical procedures because of the possibility of significant bleeding.

A. 50,000/mm3

B. 75,000/mm3

C. 100,000/mm3

D. 125,000/mm3

Answer: 50,000/mm3

Q164: All of the following characteristics raise the suspicion of malignancy EXCEPT one. Which one is the
EXCEPTION?

A. erythroplasia

B. ulceration

C. duration

D. slow growth

E. bleeding

F. induration

G. fixation

Answer: slow growth

Q165: _____ can be used for mandibular advancement or setback, whereas _____ can only be used for
mandibular setback.

A. vertical ramus osteotomy, the step osteotomy

B. sagittal split osteotomy, vertical ramus osteotomy

C. vertical ramus osteotomy, LeFort I osteotomy

D. sagittal split osteotomy, LeFort I osteotomy

Answer: sagittal split osteotomy, vertical ramus osteotomy

Q166: The prototypic neuropathic facial pain is:

A. postherpetic neuralgia

B. burning mouth syndrome

C. trigeminal neuralgia

D. temporal arteritis
Answer: trigeminal neuralgia
Q167: What is the best way to palpate the posterior aspect of the mandibular condyle?

A. intraorally

B. externally over the posterior surface of the condyle with the mouth open

C. through the external auditory meatus

D. any of the above

Answer: externally over the posterior surface of the condyle with the mouth open

Q168: The most common direction in which the articular disc in the TMJ can be displaced is:

A. laterally

B. medially

C. posteriorly

D. anteromedially

Answer: anteromedially

Q169: Which surgical approach is the best to expose the TMJ?

A. preauricular

B. submandibular

C. both are the same

Answer: preauricular

Q170: What clinical sign is considered pathognomonic for the first stage of internal derangement of the
articular disc?

A. ringing in the ears

B. reciprocal clicking

C. muscle inflammation

D. headaches

Answer: reciprocal clicking


Orthodontics

Q1: All of the following are advantages of the indirect method of bonding brackets to a tooth over the direct
method EXCEPT one. Which one is the EXCEPTION?

A. reduced chair-side time

B. more precise location of brackets possible in the laboratory

C. controlled thickness of the resin between the tooth and the bracket interface

D. less technique sensitive

E. easier clean-up during bonding and de-bonding

F. better visualization in lab (especially for lingual brackets)

Answer: less technique sensitive

Q2: Which of the following are considered functional appliances? Select all that apply.

A. Frankel

B. Bionator

C. Clark's twin block

D. Herbst

E. activator

F. quad-helix

Answer: Frankel, Bionator, Clark's twin block, Herbst, activator


Q3: The photograph shows a maxillary fixed bilateral space maintainer. This type of space maintainer also
is known as a:

1. If both primary canines were present, which of the following space maintainer(s) could be used in
place of this appliance that cannot be used in this case?

A. distal shoe

B. Nance appliance

C. lingual holding arch

D. Hawley retainer

E. band and loop (bilateral)

Answer: 1. Nance appliance 2. band and loop (bilateral)

Q4: A headgear appliance is used for:

A. anchorage

B. traction

C. both anchorage and traction

D. neither anchorage nor traction

Answer: both anchorage and traction


Q5: Which of the following are fixed orthodontic appliances? Select all that apply.

A. lingual archwires

B. whip-spring appliances

C. palate-separating devices

D. Frankel's appliances

E. edgewise mechanisms

F. light-wire appliances

Answer: lingual archwires, whip-spring appliances, palate-separating devices, edgewise


mechanisms, light-wire appliances

Q6: When comparing stainless steel versus nickel titanium in orthodontic wires, stainless steel has a
______ modulus of elasticity, and ______ resilience.

A. higher, higher

B. higher, lower

C. lower, higher

D. lower, lower

Answer: higher, lower

Q7: Which appliance is probably the most widely used today by orthodontists?

A. the Begg appliance

B. the edgewise appliance

C. the universal appliance

D. none of the above

Answer: the edgewise appliance


Q8: The Hawley retainer (shown below) is the most common retainer in orthodontics because it can use
the palate for anchorage.

A. both the statement and the reason are correct and related

B. both the statement and the reason are correct but not related

C. the statement is correct, but the reason is not

D. the statement is not correct, but the reason is correct

E. neither the statement nor the reason is correct

Answer: both the statement and the reason are correct but not related

Q9: An active finger spring of a removable appliance usually touches the tooth with a point contact. What
is the most likely type of tooth movement produced in this situation?

A. tipping

B. extrusion

C. intrusion

D. translation

Answer: tipping

Q10: Prior to direct bonding, ________ is used as an etching agent. Prior to placing bands, ________ is
used as an etching agent.

A. nothing; 35-50% unbuffered phosphoric acid

B. 35-50% unbuffered phosphoric acid; nothing

C. nothing; 10-15% unbuffered phosphoric acid

D. 10-15% unbuffered phosphoric acid; nothing

Answer: 35-50% unbuffered phosphoric acid; nothing


Q11: An example of a maxillary permanent central incisor in crossbite is shown. In order to treat this
condition properly, the dentist should do what?

A. do nothing until all permanent teeth have erupted

B. surgically reposition the central incisor

C. correct the condition immediately with a simple appliance

D. place a maxillary expander

Answer: correct the condition immediately with a simple appliance

Q12: Orthodontic forces can be treated mathematically as vectors.

Forces produce either translation (bodily movement), rotation, or a combination of translation and rotation,
depending upon the relationship of the line of action of the force to the center of resistance of the tooth.

A. both statements are true

B. both statements are false

C. the first statement is true, the second is false

D. the first statement is false, the second is true

Answer: both statements are true

Q13: Which of the following may cause extrusion of the maxillary first molars which can cause an open
bite?

A. straight-pull headgear

B. reverse-pull headgear

C. cervical-pull headgear

D. high-pull headgear

Answer: cervical-pull headgear


Q14: Which condition is appropriately treated at an early age?

A. deviated midline in the absence of a functional shift

B. mild crowding of lower permanent incisors

C. two deciduous molars nearly in crossbite

D. posterior crossbite with a functional shift

Answer: posterior crossbite with a functional shift

Q15: Displaced teeth related to functional shifts occur in which of the following situations? Select all that
apply.

A. posterior crossbite after prolonged thumb sucking

B. Class II, division I malocclusion

C. anterior crossbite in mildly prognathic children

D. an anterior open bite after prolonged thumb sucking

Answer: posterior crossbite after prolonged thumb sucking, anterior crossbite in mildly prognathic
children

Q16: Maxillary expansion is often done to correct crossbites.

Tongue thrusting often causes an anterior open bite.

A. both statements are true

B. both statements are false

C. the first statement is true, the statement is false

D. the first statement is false, the second is true

Answer: both statements are true

Q17: Which of the following are not classic symptoms of a sucking habit?

A. anterior open bite

B. crossbite

C. expanded maxillary arch

D. proclination of the maxillary incisors

E. retroclination of the mandibular incisors

F. a Class II malocclusion

Answer: expanded maxillary arch


Q18: An anterior crossbite should be corrected as soon as it is detected, because it is difficult to retain the
corrected occlusion.

A. both the statement and the reason are correct and related

B. both the statement and the reason are correct but not related

C. the statement is correct, but the reason is not

D. the statement is not correct, but the reason is correct

E. neither the statement nor the reason is correct

Answer: the statement is correct, but the reason is not

Q19: A patient's SNA angle is 78°. The SNB angle is 76°. This tells us that the maxilla is ________, the
mandible is ________, and the skeletal profile is ________.

A. prognathic; retrognathic; class II

B. retrognathic; prognathic; class III

C. prognathic; prognathic; class I

D. retrognathic; retrognathic; class I

E. prognathic; prognathic; class II

F. retrognathic; retrognathic; class II

Answer: retrognathic; retrognathic; class I

Q20: A "Poor man's Cephalometric Analysis" is performed via a

A. dental cast analysis

B. facial profile analysis

C. photographic analysis

D. full face analysis

Answer: facial profile analysis

Q21: All of the following correlate with a steep mandibular plane EXCEPT one. Which one is the
EXCEPTION?

A. long anterior facial vertical dimension

B. anterior open bite

C. tendency toward a Class III malocclusion

D. greater maxillary-mandibular plane angle

Answer: tendency toward a Class III malocclusion


Q22: What is needed so that soft tissues are clearly visible on a lateral cephalometric radiograph?

A. adjustment in kilovoltage

B. adjustment in milliamperage

C. a soft tissue shield

D. a hard tissue shield

E. nothing must be done to make soft tissues visible

Answer: a soft tissue shield

Q23: The Frankfort horizontal plane in cephalogram extends:

A. horizontally from the nasion to the porion

B. horizontally from the porion to the orbitale

C. vertically from the orbitale through the maxillary canine

D. horizontally from the orbitale to the sella

Answer: horizontally from the porion to the orbitale

Q24: In predicting the time of the pubertal growth spurt, while treating jaw malrelationships in a growing
child, the orthodontist can get the most valuable information from:

A. wrist-hand radiograph

B. height-weight tables

C. presence of secondary sex characteristics

D. stage of dental development

Answer: wrist-hand radiograph - example shown below

Q25: The term “mesiodens” is related to which of the following dental anomalies?

A. taurodontism

B. supernumerary teeth

C. enamel pearl

D. hypodontia

Answer: supernumerary teeth


Q26: In a serial extraction case, please select the correct order of tooth removal:

A. primary canine, primary first molar, permanent first premolar

B. primary first molar, primary canine, permanent first premolar

C. permanent first premolar, primary canine, primary first molar

D. primary canine, primary first molar, primary second molar

Answer: primary canine, primary first molar, permanent first premolar

Q27: Arrange the following procedures into their proper sequence for molar uprighting of a tooth requiring
both a crown and crown lengthening?

A. band
B. complete crown lengthening procedure
C. complete crown preparation and fabrication
D. separate
E. upright

Answer: 1. separate, 2. band, 3. upright, 4. complete crown lengthening procedure, 5. complete


crown preparation and fabrication

Q28: At age 9, young Jimmy needs his tooth #30 extracted due to caries. What is the proper space
maintenance treatment?

A. distal shoe on "T"

B. band and loop on "T"

C. removable partial denture

D. no space maintenance is needed

Answer: no space maintenance is needed — #31 has not erupted yet and it will tend to erupt close
to the area of where # 30 had been

Q29: ______ describes the movement of the crown in one direction while the root tipis displaced in the
opposite direction. ______ is controlled root movement while the crown is held relatively stable.

A. torque, translation

B. tipping, torque

C. tipping, translation

D. translation, torque

Answer: tipping, torque


Q30: A post-orthodontic circumferential supracrestal fibrotomy is performed to sever collagen fibers, thus
reducing the tendency of a rotated tooth to relapse.

A. both statements are true

B. both statements are false

C. the first statement is true, the second is false

D. the first statement is false, the second is true

Answer: both statements are true

Q31: Which of the following are part of the rationale for retention in orthodontics (accomplished with fixed
or removable retainers)? Select all that apply.

A. allow for reorganization of the gingival and periodontal tissues

B. minimize changes due to growth

C. maintain teeth in unstable conditions

D. permit neuromuscular adaptation to the corrected tooth position

Answer: allow for reorganization of the gingival and periodontal tissues, minimize changes due to
growth, maintain teeth in unstable conditions, permit neuromuscular adaptation to the corrected
tooth position

Q32: The cranial vault is made up of a number of flat bones that are formed by____________,
___________ cartilaginous precursors.

A. endochondral bone formation; with

B. intramembranous bone formation; without

C. both endochondral and intramembranous bone formation; with

D. both endochondral and intramembranous bone formation; without

Answer: intramembranous bone formation; without

Q33: Once bone is formed, it grows by:

A. interstitial growth only

B. appositional growth only

C. both appositional and interstitial growth

D. degenerative changes into bony structures

Answer: appositional growth only


Q34: Cartilage differs from bone in that cartilage can increase in size by:

A. apposition

B. sutural expansion

C. interstitial growth

D. endosteal remodeling

Answer: interstitial growth

Q35: The sole function of the alveolar process is to support the teeth, which is why it resorbs if a
permanent tooth is extracted.

A. both the statement and the reason are correct and related

B. both the statement and the reason are correct but not related

C. the statement is correct, but the reason is not

D. the statement is not correct, but the reason is correct

E. neither the statement nor the reason is correct

Answer: both the statement and the reason are correct and related

Q36: Bone deposition in the __________ region is responsible for the lengthening of the maxillary arch.

A. palate

B. tuberosity

C. incisor

D. zygomatic

Answer: tuberosity

Q37: An 18-year-old patient presents back to you complaining of crowding of his lower anterior incisors.
You explain that this is because of:

A. late mandibular growth

B. pressure from third molars

C. maxillary tooth-size excess

D. trauma

E. an oral habit he must have

Answer: late mandibular growth


Q38: What percentage of 6-year-old children have a median (maxillary) diastema?

A. 78%

B. 98%

C. 49%

D. less than 25%

Answer: 98%

Q39: The length of the mandibular arch is longer than the maxillary arch. The difference is only about 2
mm.

A. both statements are true

B. both statements are false

C. the first statement is true, the second is false

D. the first statement is false, the second is true

Answer: the first statement is false, the second is true

Q40: The most commonly impacted tooth is the mandibular canine.

The longer a tooth has been impacted, the more likely it is to be ankylosed.

A. both statements are true

B. both statements are false

C. the first statement is true, the second is false

D. the first statement is false, the second is true

Answer: the first statement is false, the second is true

Q41: If a permanent maxillary first molar has erupted ectopically against the distal root surface of a primary
second molar, what would be the treatment of choice?

A. disking the distal of the primary first molar

B. an appliance incorporating a finger spring to move the primary second molar mesially

C. a brass wire placed between the primary second molar and permanent first molar

D. extraction of the primary second molar

Answer: a brass wire placed between the primary


second molar and permanent first molar
Q42:

1. The time required to upright a molar can vary from:


2. The time required to stabilize the molar can vary from:

A. 2-3 weeks

B. 1-2 months

C. 2-6 months

D. 6-12 months

E. 2-3 years

Answer: 1. 6-12 months 2. 2-6 months

Q43: Match the dental arch relationships on the left with the correct depiction of the relationship on the
right.

• Class I malocclusion
• Class II malocclusion
• Class III malocclusion
• normal occlusion

Answer: normal occlusion – C, Class I malocclusion – D, Class II malocclusion – A, Class III


malocclusion – B
Q44: Which of the following are signs of incipient malocclusion? Select all that apply.

A. lack of spacing in primary dentition

B. crowding of permanent incisors in mixed dentition

C. premature loss of mandibular primary canines

D. larger than normal primary teeth

Answer: lack of spacing in primary dentition, crowding of permanent incisors in mixed dentition,
premature loss of mandibular primary canines

Q45: All of the following are the features of Angle's class II malocclusion, EXCEPT one. Which one is the
EXCEPTION?

A. excessive maxillary growth resulting in a convex facial profile

B. downward-backward rotation of mandible due to excessive vertical growth of maxilla

C. lingually tipped maxillary incisors resulting in decreased overjet

D. excessive mandibular growth resulting in a concave facial profile

Answer: excessive mandibular growth resulting in a concave facial profile

Q46: A patient presents to your office claiming that they have “Long Face Syndrome” based on what he
learned on WebDDS.com. The man has obvious mouth breathing as noted by your morning patient who
sat next to the man in the waiting room. What malocclusion are you immediately thinking that he has?

A. dental open bite

B. skeletal open bite

C. dental crossbite

D. skeletal crossbite

Answer: skeletal open bite—sometimes called the "Long Face Syndrome"

Q47: Angle's Class I occlusion represents ________ % of the population. Angle's Class II occlusion
represents ________%. While Angle's Class III occlusion represents the remainder.

A. 40; 55

B. 50; 45

C. 60; 35

D. 70; 25

Answer: 70; 25
Q48: What cephalometric analysis measurement is characteristic in Class I malocclusions?

A. SNA angle of >84°

B. SNB angle of <78°

C. ANB angle of <4°

D. none of the above

Answer: ANB angle of <4°

Q49: A concave profile is associated with a Class III occlusion.

It is also termed a retrognathic profile.

A. both statements are true

B. both statements are false

C. the first statement is true, the second is false

D. the first statement is false, the second is true

Answer: the first statement is true, the second is false

Q50: Which of the following terms can be used to describe a Class II malocclusion?

A. retrognathism only

B. overbite only

C. underbite only

D. overbite or retrognathism

E. underbite or prognathism

Answer: overbite or retrognathism

Q51: Which of the following is the least common?

A. Class I malocclusion

B. Class II malocclusion

C. Class III malocclusion

D. normal occlusion

Answer: Class III malocclusion


Q52: Which Angle Classification is shown in this set of study
casts?

A. Class I

B. Class II, Division 1

C. Class II, Division 2

D. Class III

Answer: Class II, Division 1

Q53: The existence of a forward shift of the mandible during closure to avoid incisor interference is found
in:

A. "true" class III malocclusions

B. "pseudo" class III malocclusions

C. "Sunday bite"

D. all of the above

Answer: "pseudo" class III malocclusions

Q54: The most common cause of Class I malocclusion is:

A. an abnormal frenum

B. uneven growth of the arches

C. mandibular incisor crowding

D. discrepancy between tooth size and supporting bone

Answer: discrepancy between tooth size and supporting bone

Q55: Which of the following is/are generalized causes of failure of tooth eruption or delayed tooth
eruption? Select all that apply.

A. hereditary gingival fibromatosis

B. Down syndrome

C. rickets

D. hyperparathyroidism

Answer: hereditary gingival fibromatosis, Down syndrome, rickets


Q56: Match the following age/age groups with their corresponding stages of emotional development in a
growing child as proposed by Erikson.

• 1. Birth to 18 months
• 2. 18 months to 3 years
• 3. 3 to 6 years
• 4. 7 to 11 years
• 5. 12 to 17 years
• 7. Adult
• 8. Late adult
• A. Development of autonomy
• B. Development of initiative
• C. Development of basic trust
• D. Mastery of skills
• F. Development of personal identity
• G. Attainment of integrity
• H. Guidance of the next generation

Answer: 1. C, 2. A, 3. B, 4. D, 5. F, 6. E, 7. H, 8. G1. C, 2. A, 3. B, 4. D, 5. F, 6. E, 7. H, 8. G

Q57: Which of the following statements are true? Select all that apply.

A. in the maxillary arch, the primate space is located between the central incisors and lateral incisors

B. in the mandibular arch, the primate space is located between the lateral incisors and canines

C. in the mandibular arch, the primate space is located between the canines and first molars

D. in the maxillary arch, the primate space is located between the lateral incisors and canines

Answer: in the maxillary arch, the primate space is located between the lateral incisors and
canines, in the mandibular arch, the primate space is located between the canines and first molars

Q58: Which of the following primary molar relationships could you expect to result in a Class II relationship
in the permanent dentition?

A. distal step

B. flush terminal plane

C. mesial step

D. terminal shift

Answer: distal step


Q59: A periapical radiograph of primary tooth M shows tooth #22 overlapping half of the root. The patient
is not in the chair, so you cannot palpate to determine on which side tooth #22 is erupting. What would you
presume?

A. tooth #22 is erupting lingually

B. tooth #22 is erupting mesially

C. tooth #22 is erupting distally

D. tooth #22 is erupting facially

Answer: tooth #22 is erupting facially

Q60: Leeway space is a calculated difference between primary and permanent tooth size.

There is typically more leeway space in the maxillary arch.

A. the first statement is true, the second is false

B. both statements are false

C. both statements are true

D. the first statement is false, the second is true

Answer: the first statement is true, the second is false

Q61: A 6-year-old patient and her mother present to your office. Her mom's chief complaint is "My kid's
overbite makes her look like Bugs Bunny, her front teeth hide her lower lip." What is wrong with the mom's
statement?

A. she is mixing up overbite and negative overjet

B. she is mixing up overbite and open bite

C. she is mixing up overbite and overjet

D. she is mixing up her cartoon characters

Answer: she is mixing up overbite and overjet

Q62: Which of the following teeth are required for a dentist to perform a Moyers' mixed dentition analysis?

A. mandibular incisors

B. maxillary first molars

C. mandibular first molars

D. maxillary incisors

Answer: mandibular incisors


Q63: Which of the following is false concerning a mixed dentition analysis?

A. the analysis is based on a correlation of tooth size

B. it determines space available vs. space required

C. it is used to predict the amount of crowding after the permanent teeth erupt

D. analysis is done for each quadrant

E. it is performed with a Boley gauge, study models and a prediction table

F. it is performed during the mixed dentition

Answer: analysis is done for each quadrant


Clinical Dentistry OSCE Questions
Department of Orthodontics

A fit and healthy 14-year-old patient attends your practice for a routine
checkup. A clinical photograph of their mouth is shown below.

1. The patient appears to have an anterior open bite. What are the possible
causes of such a condition?

 Habit (digit sucking): associated with asymmetrical anterior open bite ±


posterior crossbite.
 Soft tissue (action of the tongue): tongue thrust often causes symmetrical
anterior open bite.
 Skeletal pattern: associated with increased lower face high and Frankfort-
mandibular plane angle.
 Localized failure of alveolar development: e.g., cleft lip and palate/can be
spontaneous.
2. Based on the clinical photograph, what is the most likely cause of the
anterior open bite in this case?

 Habit/digit sucking is the likely etiology due to the following features:


asymmetrical anterior open bite with unilateral crossbite and non-coincident
centerline.
An 18-year-old male patient presents at your clinic with the dentition shown in
the clinical photograph below.

1. What appearance is shown in the above photograph?

 The patient has missing upper lateral incisors and a non-coincident dental
centerline. They have a reduced overbite and reduced overjet, and features of
Class III in the buccal segments.

2. What are the possible causes of the missing lateral incisors?

 Developmentally absent.
 Previously extracted.
 Avulsed (unlikely as bilateral).
 Dilacerated/displaced due to trauma (unlikely as bilateral).
 Supernumerary teeth preventing eruption.
 Crowding – insufficient space.
 Presence of a pathological lesion.
This is an intra-oral photograph of a removable orthodontic appliance.

1. What is the name of the active component in this appliance?

 T-spring.

2. Based on the photograph above, what orthodontic problem is the


appliance aiming to correct?

 Anterior crossbite (tooth #21 and #22).


A 15-year-old boy presents to you in general dental practice requesting
closure of the spaces between his upper front teeth. His permanent teeth erupted at
a normal age with large spaces between them. The primary predecessors had all
been present and were exfoliated normally. None of the permanent teeth has been
extracted.

1. What is the cause of the spacing between the patient’s upper anterior
teeth?

 Congenitally missing permanent upper lateral incisors.

2. How can this case be managed?

 Closing the space: this case demands fixed orthodontic appliance treatment to
accurately position the central incisors and to rotate the canines.
 Replacing the missing laterals: rotated canines and spacing in the upper
anterior teeth both demand permanent orthodontic retention which is the main
reason an implant-retained solution is not appropriate. Considering the age of
the patient, a resin-bonded bridge should be used as it is more conservative
than a conventional bridge. A degree of orthodontic retention must be
designed into the prosthesis and only a fixed–fixed bridge extending from
canine to canine is suitable. Regular recalls are essential to detect possible
debonding of the wings of the resin-bonded bridge. If debonding is a repeated
problem, a conventional bridge design may have to be considered.
Examine the orthodontic appliance in the photograph below.

1. What is this appliance called?

 Frankel III appliance.

2. When is this appliance used?

 Class III malocclusion.

3. How does this appliance work?

 The labial pads and vestibular shield are to counteract the forces of the
surrounding musculature that tend to restrict forward maxillary skeletal
development. The vestibular shields need to be positioned away from the
alveolar process of the maxilla, but must fit closely to the tissue of the
mandible. This results in stimulation of maxillary alveolar development and
restriction of mandibular alveolar development.
Examine the following photograph of a dental appliance.

1. What is this appliance called?

 Space regainer.

2. When is this appliance used?

 When there is a space loss caused by the mesial drift of the permanent first
molar.

3. In which quadrant will this appliance be placed?

 The upper left quadrant.


Examine the following photograph of a dental appliance.

1. What is this appliance called?

 Fixed habit-breaking appliance.

2. What is the purpose of this appliance?

 To correct a thumb-sucking habit.


You are a dentist in general practice. A mother has brought her 9-year-old
daughter in to see you for her first dental appointment. The mother was unsure
whether her daughter needs orthodontic treatment to correct what appears to be a
prognathic mandible, upon clinical examination you found that:
SNA= 72 SNB = 80 ANB = -8

1. Describe the patient profile in the photograph.

 Concave profile.

2. What will be your advice to the mother and what will be your interceptive
treatment plan?

 The mother will be informed that her child has a Skeletal Class III
malocclusion and the best time to start treatment is between the age of 8 to 10
years, and the treatment plan would be an extra-oral appliance (Face mask)
combined with a Hyrax expander.
You are a dentist in general practice. A mother has brought her 11-year-old
son in to see you for her first dental appointment. The mother was unsure whether
her son needs orthodontic treatment to correct what appears to be a retrognathic
mandible, upon clinical examination you found that:
SNA= 82 SNB = 75 ANB = 7

1. Describe the lip form shown in the photograph.

 Incompetent lips.

2. What will be your advice to the mother and what will be your interceptive
treatment plan?

 The mother will be informed that


o The best time to treat her child is during the growth spurt which will be
evaluated using the CVMI index.
o 2- The treatment plan is a functional orthopedic appliance to enhance
mandibular growth and an example of such a device is: bionator
appliance.
Clinical Dentistry OSCE Questions
Department of Pedodontics

A fit and healthy 13-year-old boy attends your practice after a football tackle
leaves him with an injury to the upper front teeth.

1. Explain how you would manage this case.

 Establish the circumstances surrounding the injury and how it occurred.


 Ask if there was any loss of consciousness.
 Check the patient’s tetanus status.
 Examine the patient: determine which teeth are mobile; exclude any alveolar
fractures.
 A radiograph should be taken to visualize any fractures and/or displacements
that might have occurred which will decide the appropriate treatment.

2. You took an IOPA radiograph of this patient as shown below. What will be
your treatment?

 The radiograph shows tooth #11 with a


root fracture in the apical 1/3rd of the
root. This would require no treatment,
unless:
o If the tooth is mobile, it will require
the use of a flexible splint for 4
weeks.
o If the coronal fragment is displaced,
it should be repositioned and then a
flexible splint is used for 4 weeks.
o If pulp necrosis develops then a
root canal treatment (RCT) of the
coronal pulp to the level of the root
fracture is necessary.
A 7-year-old boy presents with first permanent molar teeth which his parents say
have decayed rapidly, starting immediately on eruption. The child complains of pain
from his back teeth on both sides. The pain is worse with sweet foods and cold liquids
and persists for several minutes after stimuli. Toothbrushing with cold water is also
painful but the teeth do not cause pain on biting. Clinical photographs of the upper left
first permanent molar and upper central incisors are shown below.

1. What is the cause of the child’s pain?


 Molar–incisor hypomineralization.

2. What is the treatment for the molars?


 The appropriate treatment for grossly hypoplastic first permanent molar teeth is
extraction, particularly when caries are present, as in this case.

3. When should the molar teeth be extracted?


 This depends on:
o Stage of dental development: Second permanent molars are most likely to
erupt passively into a favorable position when there is radiographic evidence of
calcification inter-radicular dentine (mineralizing of the furcation). This is the
ideal time to extract the first molar and is generally between 8.5 and 10.5 years
of age.
o Presences of third molars: This must be assessed radiographically. Hard tissue
formation should be visible at age 9–10 years. The crypt may be visible as
early as 7 years. If no third molars are present, the need for orthodontic
treatment may be critical in deciding whether or not to extract the first molars.

4. How can the molar teeth be preserved until the patient is at the optimal age
for their extraction?
 By using stainless steel crowns.
Examine the clinical photograph below.

1. What happened to this patient?

 The patient had a traumatic injury to his upper central incisors that resulted in
a crown fracture with the tooth fragment being embedded in their lower lip.
Examine the following clinical photograph of a 5-year-old child.

1. What is this condition called?

 Early childhood caries.

2. What is your initial treatment for this condition?

 Caries removal and restoring the teeth with fluoride-releasing glass ionomer
restorative material.
A 7-year-old child has sustained a trauma to his upper central incisor 36 hours
ago that resulted in a complicated crown fracture of tooth #21 with pulpal exposure
of 1-2 mm. There was no swelling or mobility. Radiographic findings showed normal
periapical tissues.

1. What is your management for this case?

 Partial pulpotomy (Cvek’s pulpotomy).

2. Why did you choose this type of treatment?

 Because of the age of the patient, he would have immature roots. So, pulp
vitality should be maintained to allow root closure to continue. Based on the
time elapsed since the pulpal exposure and the size of the pulpal exposure, a
partial pulpotomy is appropriate.
Examine the following picture.

1. The crown identifying marking is located on which surface?

 On the buccal surface of the crown.

2. This specific crown corresponds to which tooth in the FDI tooth numbering
system?

 #74.
Examine the following picture.

1. What is this instrument called?

 Band pusher/seater.

2. When is this instrument used?

 It is used to seat orthodontic bands.


Examine the following radiograph.

1. What is this appliance called?

 Distal shoe space maintainer.

2. What is the error shown in the radiograph above?

 Obstruction of the second premolar from eruption by the intra-alveolar


extension.
Examine the following clinical photograph of a 16-month-old child.

1. What is this condition called?

 Eruption cyst.

2. What is your treatment for this condition?

 Incision.
Examine the following clinical photograph.

1. What is this type of restoration called?

 Anterior strip/celluloid crown.

2. What type of finish line does this restoration require?

 Feather edge.

3. What are the disadvantages of this type of restoration?

 It is extremely technique sensitive.


 It is not as durable or retentive as stainless steel/open-faced crowns, pre-
veneered crowns, or polycarbonate crowns.
 Not recommended on patients with a bruxism habit or a deep bite.
 Adequate moisture control might be difficult on an uncooperative patient.
A nine year old reports to your dental office with past dental treatment from
another clinic and has pain on biting on the tooth with stainless steel. A periapical
radiograph was taken as shown below.

1. Mention the abnormality in the eruption sequence in the above radiograph.

 The second premolar is erupting before the first premolar.

2. What is the reason for the abnormal eruption sequence and what is its
management?

 Previous pulpal treatment has resulted in a faster root resorption of its primary
predecessor, hence early eruption.
 Management: Extraction with NO space maintainer.
If you are performing a pulpotomy and clinically there is still a profuse bleeding
even after placement of ferric sulphate medicament.

1. Give two reasons and management for both scenarios.

 Radicular pulp is irreversibly inflamed. Management - Proceed for pulpectomy


 Pulpal remnants are still present due to improper deroofing of pulp chamber.
Management - Ensure complete pulp removal
A seven year old child reported to your dental office with history of pain at
night. The following radiograph was taken.

1. Enumerate important parameters that you should look for in a periapical


radiograph for a correct diagnosis.

 Root Resorption; Bone Resorption; Furcation area involvement, presence of


permanent tooth; lamina dura continuity.

2. What is your treatment plan for painful tooth?

 Extraction and band & loop space maintainer.


A six year old child comes to you in pain with previous history of dental
treatment carried out one year ago in another clinic. The following periapical
radiograph was taken.

1. Mention any two faulty / wrong treatments performed by the first dentist as
visible from the radiograph.

 Overhang distal filling on D.


 Stainless steel crown was not placed after pulpotomy
 Zinc oxide dressing was not condensed properly. It should contact radicular
pulp stumps otherwise this space can act as an area for bacterial growth
leading to necrosis of radicular pulp.
An eight year old cooperative child with the following carious lesions on adjacent
primary teeth.

1. Which tooth-colored restorative material will you prefer for these class II
lesions?

 Composite.

2. What is the reason for your selection of the restorative material in the
above situation?

 Since the cavities are small in size, isolation is good, child is cooperative and
compressive strength is excellent among other tooth-colored restorations.
Examine the following photograph.

1. What is this instrument called?

 Crown contouring or crimping plier.

2. What is this instrument used for?

 It is used to contour/crimp the gingival margins of stainless-steel crowns


thereby increasing their retention.
Examine the following clinical photograph.

1. Explain the possible consequences that can result with faulty design of the
following appliance.

 Lingual arch will push the mandibular incisors more labially and 32/42 will
erupt lingually to the lingual arch.

2. Describe the long-term clinical management in the following situation.

 Place bilateral band & loop space maintainers till the eruption of 32/42 and
once they have fully erupted, replace band and loop with lingual arch to allow
normal distal movement of primary mandibular canines which will further allow
natural increase in inter-canine width.
Clinical Dentistry OSCE Questions
Department of Periodontics

Below is a picture of a periodontal instrument.

1. What is the instrument shown above called?

 The World Health Organization (WHO) periodontal probe, ball-ended, with a


diameter of 0.5 mm.

2. At what level is the black-colored band that is closest to the tip of the
instrument located?

 The colored black band lies 3.5-5.5 mm from the tip of the probe.
A 35-year-old man has noticed a gap appearing between two incisor teeth. He
noticed the gap about 9 months ago and feels that it has enlarged, that the teeth have
drifted forwards, and that the crowns are now loose. He has had no symptoms from
these or any other teeth. He has mild asthma controlled with an inhaled steroid and
salbutamol. He does not smoke.
1. Based on the clinical presentation
and the history of the complaint,
what do you think is the cause of
these gaps?

 Localized aggressive periodontitis


caused by traumatic occlusion
from over-contoured upper central
incisors PFM crowns. The
periodontitis has reduced the
bone support for the upper and
lower right anterior teeth and they
have drifted labially.

2. How should this case be


managed?

 The ideal treatment plan would


include:
o Patient education.
o Instruction in plaque control,
tooth brushing, and interdental
cleaning.
o Subgingival scaling.
o Root planing all severely affected
teeth under local analgesia.
o Extraction of the teeth that lack
bone support based on
radiographic assessment and
probing depths recording.
o Immediate replacement prosthesis (Acrylic RPD).
o A maintenance phase to assess the response.
o Continued care of the remaining dentition including caries prevention.
o Extraction of further teeth as required.
o Assessment for permanent prosthetic replacement (Cr-Co RPD).
Examine the following clinical photograph

1. What is this material?

 Periodontal surgical dressing.

2. When is this material used?

 After periodontal surgery.

3. How is this material applied clinically?

 The base and catalyst are mixed together thoroughly on a glass slab.
petroleum jelly is rubbed on the hands of the operator so the dressing does
not stick to their gloves. The operator will then roll the mixed dressing into a
ball/roll and then adapt it gently towards the teeth while also manipulating the
lip/cheek of the patient to prevent overextension of the dressing into the
vestibule. The dressing is retained in the patient’s mouth through mechanical
interlocking in the gingival embrasures.
After extracting a lower molar, you noticed a black-colored substance on the
distal root of the extracted tooth as shown in the photograph below.

1. Based on the photograph above, what do you think was the cause of
extraction for this tooth?

 Lack of bone support.

2. What is the black-colored substance on the distal root of the tooth?

 Sub-gingival calculus.
Examine the following photographs.

1. What type of bone loss is shown in (A)?

 Horizontal bone loss.

2. What type of bone loss is shown in (B)?

 Vertical (angular) bone loss.


Examine the following clinical photographs.

1. According to Glickman’s classification of furcation involvement, what


grade of furcation involvement does the lower first molar have?

 Grade IV furcation.

2. Although the conventional straight probe can be used to measure furcation


involvement, a superior option is:

 Nabers probe.
Examine the following clinical photograph.

1. What is this procedure called?

 Root resection.

2. What are the main contraindications for this procedure?

 The remaining Periodontal Support is inadequate.


 The tooth cannot be treated endodontically.
 Restoration of the remaining tooth, including splinting cannot be performed.
Examine the following clinical photographs.

1. What is this surgical procedure called?

 Surgical crown lengthening.

2. How long do you have to wait after performing this surgical procedure
before taking a final impression and placing a permanent restoration?

 3 to 6 months.
Clinical Dentistry OSCE Questions
Department of Endodontics

1. According to the above radiograph, what complication has occurred?

 There is radio-opaque matter in the inferior dental canal beneath the lower
second molar. This tooth also has some radio-opaque material in the distal
root canal. A root canal filling was being carried out and material has extruded
through the apex of the tooth and has ended up in the inferior dental canal.

2. What symptom might the patient be experiencing?

 If material is in the inferior dental canal, the patient will likely complain of
altered sensation in the distribution of the inferior dental nerve, i.e., the lower
lip. The altered sensation may be numbness (anesthesia) or tingling
(paresthesia) and in some cases pain.

3. What are other causes of these symptoms?

 Iatrogenic – trauma following surgery, e.g., surgical removal of wisdom teeth,


lower premolars.
 Infection – osteomyelitis.
 Degenerative – multiple sclerosis.
 Metabolic – tetany, diabetic neuropathy.
 Neoplastic – space-occupying lesion.
A 60-year-old female patient presents complaining of discomfort from a lower
left back tooth. The filling was lost from the tooth about 4 months ago. She has
suffered intermittent problems since the tooth was root canal-treated 2 years ago.
Pain is triggered by biting on the tooth and changes in temperature have no effect.
The symptoms have remained similar in intensity ever since the root canal treatment
was completed. Below is a parallax view, taken from the mesial.

1. This case requires endodontic retreatment. Mention the factors that were
taken into account to arrive at this conclusion.

 Pain on biting and well-localized pain indicate periapical periodontitis and the
cause is almost certainly the failure of the root canal treatment.
 There is a fractured instrument. This probably means that no apical seal would
have been achieved in that canal, reducing the chances of successful
treatment.
 The provisional restoration was lost 4 months ago. This indicates a complete
loss of coronal seal to the root canal system that would allow microleakage of
bacteria and their toxins along the length of the root filling.

2. In which canal is the fractured instrument located?

 The mesiolingual canal.


A 9-year-old male child presented for treatment 1 day after sustaining trauma
to the maxillary right lateral incisor. The intraoral clinical examination showed no soft
tissue lacerations, no crown or root fracture, and no mobility. A radiograph (A) was
taken. Since the tooth did not respond to cold and hot stimuli, the patient was
scheduled for reevaluation within 2 weeks, but he did not show up. The patient
returned 3 months later with a fistula associated with the apical region of the
traumatized tooth. A new periapical radiograph was taken (B).

1. Why did the tooth not respond to cold or hot stimuli during the initial visit?

 Immediately following traumatic injury, teeth often fail to respond to


conventional pulp testing methods. This temporary loss of response is caused
by injury, inflammation, pressure, or tension to apical nerve fibers.

2. What is your treatment for this tooth?

 Apexification.
1. What is this instrument called?

 MAP (Micro Apical Placement) system / Messing gun.

2. What is this instrument used for?

 Placing endodontic materials by orthograde or retrograde approaches.


A 17-year-old college student presents at your general dental practice with
several carious lesions, one of which is very large. He complains that a filling has
fallen out of a tooth on the lower right side and has left a sharp edge that irritates his
tongue. He is otherwise asymptomatic. The filling was placed about a year ago at a
casual visit to the dentist precipitated by acute toothache triggered by hot and cold
food and drink. He did not return to complete a course of treatment.

1. Based on the history of the patient and the clinical photograph above, what
is the diagnosis for this patient?

 Chronic periapical abscess.

2. Based on the clinical photograph above, do you need to check the vitality
of the teeth in this area? And why?

 Yes. Even though the first molar is the most likely cause, the adjacent teeth
should be tested because more than one tooth might be nonvital. The results
should be compared with those of the teeth on the opposite side. Both hot/cold
methods and electric pulp testing could be used because extensive
reactionary dentine may moderate the response.
The picture below shows a cross-section of an endodontic file.

1. What is this endodontic file called?

 Wave One Gold reciprocal file.

2. What is the equivalent ISO tip size for this endodontic file?

 #45.
The picture below shows a set of drills.

1. What is the name of these drills?

 Gates Glidden.

2. These drills come in multiple sizes, the diameter of size 3 of this


instrument is:

 0.90 mm
Tooth #32 is not responding to sensibility tests and there is a periapical
radiolucency in the radiograph. The tooth responded negatively to percussion
testing and there is no extra-oral swelling. There was an intra-oral fistula.

1. What is the diagnosis for tooth #32?

 Necrotic pulp with chronic apical abscess.

2. What is your management for tooth #32?

 Drainage of the abscess and then root canal treatment.


Tooth #36 was diagnosed with necrotic pulp with asymptomatic apical
periodontitis. The tooth was previously accessed and a temporary filling was placed.
The patient was referred to you to continue the root canal treatment for this tooth.
The patient tells you that the temporary filling has fallen off and this was confirmed
during the clinical examination. You took a radiograph as shown below. After local
anesthetic administration and rubber dam placement, you irrigated with sodium
hypochlorite and you located all of the canal orifices.

1. Based on the radiograph above, what can happen if you try to measure the
working length and reach the apex with an electronic apex locator now?

 Formation of a ledge.

2. Why can this happen?

 The presence of dentine triangles/interferences over the canal orifices causes


bending of the instrument while inserting in the canal leading to instrumental
errors (no straight-line access).
Tooth # 11 is not responding to sensibility tests; a fluctuant swelling is
palpable on the buccal vestibule.

1. Write the diagnosis for tooth#11

 Pulp necrosis with acute apical abscess.

2. What is your immediate management?

 Drainage of the tooth (apical trephination).


 Soft tissue incision.
The x- ray below illustrating treatment delivered to a 9-year-old male patient
with a diagnosis of pulpal necrosis secondary to trauma, with a class 3 fracture in
tooth #11 and a class 2 fracture in tooth #21

1. Name the procedure that been used to treat the above case.

 Apexification.

2. Name the materials that can used for this procedure.

 MTA.
 Ca(OH)2.
 Tricalcium phosphate.
Fig. A illustrate a J-shaped “halo” radiolucency associated with the second lower
premolar.
Fig. B illustrate an extensive bone damage associated in a second right maxillary
premolar.

1. What is in your opinion the most possible cause of failure of both teeth in
the above x-Rays?

 Post.
 Vertical root fracture.

2. What is the management?

 Extraction.
Examine the following photograph.

1. Name the main compositions of the above solution?

 Tetracycline.
 Citric acid.
 Detergent.

2. What it is used for?

 Irrigation.
Patient visited your clinic seeking a treatment for the discoloration of his teeth as
shown in the picture.

1. identify type and cause of this discoloration.

 Dental fluorosis.

2. Name five causes of intrinsic discoloration.

 Drug related cause (tetracycline).


 Pulp necrosis.
 Intrapulpal hemorrhage.
 Developmental defect.
 Restorative materials.
A 19-year-old patient presented with cold sensitivity from his upper right
second molar (17) which had recently fractured. There were no symptoms prior to
the tooth fracturing 3 weeks ago. Since the fracture, the tooth was sensitive to cold
liquids and symptoms lasted for 5–10 seconds only. The pain did not linger 4 years
previously.

1. What did the clinical picture and radiograph reveal about tooth #17?

 Deep caries lesion.


 Caries 1mm close to the pulp.
 No P.A lesion.

2. What is the diagnosis and treatment planning?

 Reversible pulpitis.
 Remove the caries fully, Indirect pulp cap.
The radiograph shown belongs to a patient that had a history of luxation injury in his
incisor.

1. Name the required treatment.

 Apexification.

2. Mention three materials that can be used in your treatment.

 MTA.
 Ca(OH)2.
 Tricalcium phosphate.
Examine the following picture.

1. Mention the composition of MTA.

 Calcium sulfate.
 Aluminum.
 Tricalcium silicate.
 Silicon bismuth oxide.

2. Mention three uses of MTA.

 Direct pulp capping.


 Perforation repair.
 Apexification.
Examine the following clinical photograph.

1. How you can differentiate between the bone and apex of the root during
surgical endodontic procedure?

 The bone is white in color and bleeds more.


 The apex of the root is yellowish in color and does not bleed.
Clinical Dentistry OSCE Questions
Department of Operative Dentistry

This is a bitewing radiograph of a 32-year-old patient that was taken during a


routine check-up visit. During clinical examination, there was no cavitation present.

1. What treatment do you propose?

 Detailed diet history; assessment of patient motivation to follow a rigorous


preventive regimen involving dietary change, meticulous plaque control and,
home fluoride application until stability achieved. Regular review required.

2. When is it necessary to drill these carious lesions?

 Operative intervention is necessary only in the presence of cavitation and/or


when the lesion is reaching the dentine.
A 32-year-old man presents at your general dental practice with intermittent
pain on biting. He complains of pain on biting which is unpredictable, extremely
painful, and sharp but poorly localized. It originates in the lower right quadrant and
lasts a very short time, only as long as the teeth are in contact, and is so painful that
he has become accustomed to eating on the left. The pain only arises on biting hard
foods or deliberately clenching his teeth. Apart from these sharp electric shock-like
pains he has no other symptoms. The pain is a recent phenomenon, having been
first noticed a month or two ago. At first, it was frequent but it has become less of a
problem now that he has learned to avoid triggering the pain. He has not noticed the
pain being provoked by hot or cold. Upon performing clinical tests, you discover that
all the teeth in the quadrant are vital. Biting on cotton wool on the lower second
molar provokes pain that the patient identifies as the same as that on biting. The
patient has no occluding restorations of dissimilar metals.

1. What is the most likely cause of


the patient’s pain?

 A crack in the tooth.

2. How can you confirm your


diagnosis?

 The path of the crack must be


defined as far as possible
because this will determine
treatment options. The
restoration(s) in the tooth
should be removed and a
further attempt is made to find the crack using transillumination and dye.

3. After confirming your diagnosis, what is your treatment of choice for this
tooth?

 If the crack appears to enter the pulp or be directed towards it, root treatment
will be required. Otherwise, a full coverage indirect restoration is the treatment
of choice. Full occlusal coverage is needed to protect the tooth from further
damage and a casting can provide some splinting, reducing the potential for
further cracks.
During routine caries removal for proximal carious lesions, you stopped to
assess your cavity design and the caries removal. Examine the clinical photograph
of the resultant cavity preparations.

1. If you were to consider this as your final cavity preparation and proceed
with the placement of the permanent restoration, what fault(s) would be
present in your cavity preparation?

 Leaving caries at the dentino-enamel junction (DEJ).


 Leaving demineralized enamel at the gingival margin.

2. What is the significance of such fault(s)?

 Caries at the DEJ must be removed completely. If incompletely removed and


sealed under a permanent restoration, its lateral spread will not stop and will
result in recurrent caries and a failed restoration.
 Demineralized enamel could affect the marginal integrity of the permanent
restoration and might lead to leakage.
The following photograph shows a self-adhesive resin cement product.

1. What does the phrase “self-adhesive resin cement” mean?

 It means that this resin cement does not require any surface treatment for
neither the tooth surface nor the indirect restoration or post.

2. When comparing a self-adhesive resin cement to a resin cement with a


self-etching primer component, which resin cement has the stronger bond
strength?

 A resin cement with a self-etching primer has a stronger bond strength.


Examine the following photograph of a dental instrument.

1. What are these instruments called?

 Tofflemire retainers.

2. What is the main indication of using the second instrument when


compared to the first one?

 The contra-angled head of the second instrument allows it to be placed from


the lingual side instead of the buccal.
 When the buccal wall of a tooth is lost and a cervical seal is required, the
retainer is placed from the lingual side so that the gap formed between the
matrix and the retainer will be on the lingual surface and the buccal surface
will be completely sealed by the matrix.
Examine the following picture.

1. What is the instrument in (A) called?

 Tofflemire matrix band with gingival extension.

2. When this instrument is used, sometimes the step shown in (B) is


performed. What is the significance of this clinical step?

 The matrix band may be trimmed for the unaffected proximal surface of the
tooth (DO or MO) or for the shallower gingival margin (MOD), permitting the
matrix to seat farther gingivally for the deeper gingival margin on the other
proximal surface and not be impeded by the interdental papilla of the
unaffected proximal side / shallower gingiva margin.
Examine the following radiograph of the 2 class II permanent restorations that
have been done on the lower first and second molars.

1. What was done poorly during the restoration of these teeth?

 The radiograph shows that the wedge deformed the matrix band towards the
inside of the cavity preparation because of improper placement of the wedge
which resulted in a poor contour of the proximal surface.

2. Why is this considered significant?

 The poor contour of the proximal surface will result in food packing and plaque
accumulation.
Examine the following picture of different brands of universal bonding agents.

1. What is usually meant by a “universal” bonding agent?

 That it can be used with any etching technique: total-etch, self-etch, or


selective-etch modes.

2. Regarding the bonding strength to enamel: the weakest bond strength to


enamel is attributed to which enamel surface treatment?

 Self-etching the enamel with a self-etching bonding agent or a universal


bonding agent instead of etching it with phosphoric acid will provide a weaker
bond to the enamel.
Examine the following picture of a self-etching, dual-cure resin cement.

1. What will happen if you etch the dentin with phosphoric acid before
applying the self-etching primer component of such resin cement?

 The phosphoric acid will remove the smear layer present on the dentine
surface which will compromise the bond strength to dentine as the self-etching
primer relies on the smear layer present on the dentine to penetrate into the
dentinal tubules and form resin tags when thoroughly scrubbed into dentine.

2. The manufacturer prohibits the use of lentulo spirals for loading the dual-
cure resin cement into the root canal during the cementation of posts. Why
is that?

 Using a lentulo spiral to place the resin cement into the root canal will greatly
accelerate the polymerization of the dual-cure resin cement due to the rapid
mixing of the paste while in the canal which can lead to a complete setting of
the cement in the canal before even placing the post.
Examine the following picture.

1. The colored ring on the shank of the burs indicates the roughness of the
bur. What type of roughness does the green ring indicate?

 Coarse bur.

2. What is a bur with a green-colored ring used for?

 Fast reduction of enamel.


Examine the following photographs.

1. Which type of permeant restoration is shown in the photograph?

 Glass-ceramic endocrown.

2. What are the advantages of utilizing this type of permanent restoration?

 Using this type of restoration preserves more tooth structure resulting in


increased longevity.
 It will preserve more enamel, thus enhancing the adhesion.
 Does not require post placement that could weaken the roots.
A 30-year-old female patient presented to the dental office complaining of pain
form her maxillary upper second molar. The pain increased when consuming cold
food or drink and also sharp pain when biting, as the applied occlusal force
increased and there was pain on release of pressure when eating fibrous foods. The
radiograph did not show an inflammatory lesion and the cold vitality test showed a
positive reaction of the pulp.

1. What is your diagnosis for this tooth?

 Cracked tooth syndrome (incomplete tooth fracture).

2. What is the treatment to solve the tooth problem?

 Cusp capping restoration (only is mainly preferred, overlay or crown can also
be used according to the extent of the crack).
A 42-year-old male patient, presented to your dental clinic for dental treatment.
You identified the presence of multiple cervical lesions, located on the buccal of
lower left posterior teeth, The patient had complaint of dental hypersensitivity with
cold in that area, the medical history revealed the existence of the phenomenon of
bruxism. Clinical examination revealed that the source of sensitivity was the
presence of exposed dentin cervically, and areas of oclussal wear and enamel
fissures which are specific to hyperoccluded teeth..

1. What is your clinical diagnosis for these defects?


 Abfraction (occlusal trauma and heavy force in eccentric occlusion causes
stress concentration at the cervical portion of the tooth, resulting in loosening
and gradual loss of enamel rods).

2. What are the two main aspects of treatment?


 Occlusal adjustment to heavy contacts and removing premature contact.
(occlusal guard should prove to be a good treatment strategy).
 Restorative treatment is required to eliminate hypersensitivity areas prevent
dental fractures and pulp complications.

3. Do you prefer to restore these lesions with a restoration that has high or
low modulus of elasticity and why?
 Restoration with law modulus of elasticity, because it will flex with the tooth
and will not compromise the retention of the restoration.
A patient visited your dental clinic complaining of pain in a mandibular molar
into which a DO amalgam filing was placed few days previously. He described the
pain as sharp and of short duration whenever the opposing teeth are in contact.
Upon clinical examination, an intact Class II dental amalgam restoration is present
which is not in hyperocclusion. The opposite posterior teeth are restored with gold
inlays.

1. What is your diagnosis for the tooth pain?

 Galvanic shock.

2. What is your treatment plan to relief the pain?

 Reassure the patient that the pain will be reduced within time.
 If the patient cannot tolerate the pain, replace the DO amalgam restoration
with a non-metallic restoration.
A 16-year-old boy was treated for the fracture of a maxillary central incisor he
sustained. After endodontic treatment, the tooth was restored with an adhesive
composite resin. During the course of 3 months, the restoration fractured and was
repeatedly replaced. The third time the restoration failed, a thorough examination
and radiographs revealed successful endodontic treatment. The tooth was negative
to percussion and palpation.

1. What is your suggested treatment plan for this case?

 Immediate post retained composite crown build up.


 An all-ceramic crown after 2 years.
A teenage girl visited your dental clinic with her parents complaining of white
spots on her maxillary central incisors since their eruption. On clinical examination
the facial surface of these incisor teeth were intact and smooth.

1. What is your suggested treatment plan to improve the teeth color?

 Microabrasion (or resin infiltration technique ICON).

2. Name the material to be used during treatment.

 Microabrasion: 18% hydrochloric acid with pumice.


 ICON technique:
o ICON - etch: 15% hydrochloric acid.
o ICON - dry: ethanol.
o ICON - infiltration: low viscosity resin.
Examine the following photographs.

1. What is the best indicated luting agent (cementing medium) used to bond
those feldspathic porcelain veneers?

 Composite resin cement (with more opaque shade).

2. What surface treatment we can do to the inner surface of the veneers to


enhance their retention?

 Etching with hydrofluoric acid, followed by treatment with silane coupling


agent.
Examine the following clinical photograph.

1. To restore the upper first and second premolars, is it better to go for open
or closed sandwich technique and why?

 Open sandwich technique is better because the erosive lesion is extended to


the exposed root surface (non-enamel area) where GIC has better adhesion
and sealing to the cavity margin at the area.
Examine the following clinical photograph.

1. Name two common material indicated to seal this accidental pulp


exposure.

 Calcium hydroxide.
 Mineral trioxide aggregate (MTA).

2. Briefly explain the mechanism of action of these materials.

 those two materials have alkaline PH, so necrotic zone is formed adjacent to
those materials, the dentin bridge then forms between this layer and the
underlying vital pulp. The necrotic tissue eventually degenerates, and
disappear, leaving an gap between the capping material and the ridge.
A 30-year-old patient presented with large carious lesions in most of the upper
teeth. He reported that the teeth are sensitive to cold, but otherwise asymptomatic.
A cold test did verify the transient sensitivity to cold, which dissipated at
approximately 30 seconds. A periapical radiograph confirmed that the caries was in
close proximity to the pulp with no periapical or periodontal changes.

1. What is your emergency treatment for this patient?

 Caries control restorative treatment or step-wise (partial) caries removal.

2. Name two main restorative material that can be used in this initial
emergency visit.

 Glass ionomer (conventional or RMGI).


 IRM.
Clinical Dentistry OSCE Questions
Department of Prosthodontics

Below is a photograph of a four-unit fixed-fixed metal-ceramic bridge framework.

1. What are the mistakes that you can see?

 Inadequate occlusal clearance on 21, 11, and 12 for ceramic on the palatal
surface.
 The Connector mesial to 13 is not wide enough occlusogingivally.
 The Connector mesial to 13 is not wide enough buccolingually.
 13 retainer margin does not have space for an adequate ceramic shoulder.
 21 pontic is too buccally placed for correct esthetics of ceramics build-up.
You are taking a final impression for an upper completely edentulous arch with
a custom impression tray. Examine the clinical photograph below.

1. You are planning to do border molding with putty polyvinyl siloxane


impression material. How can you border mold the area indicated in the
photograph?

 After the custom tray with the putty material is inserted into the patient’s
mouth, the patient should close halfway. The putty material then should be
pushed into the notch by the operator’s finger. The patient then will open wide.
A vertical notch of pterygomandibular raphe should be noted in the putty.

2. What other types of materials that could be used to border mold this area?

 Green stick impression compound.


The following is a photograph of a mandibular custom tray.

1. There was a fault in the making of this custom tray. What is it?

 The angulation of the handle of the custom tray is incorrect.

2. What is the significance of this fault?

 The handle will almost certainly get in the way of the lower lip, risking
displacement during use and distortion of the sulcus affecting the peripheral
seal and the retention of the complete denture.
Examine the below clinical photograph of a patient with a completely
edentulous mandibular arch.

1. What is the whitish-colored tissue in the patient’s mouth?

 Mucosal graft.

2. Why does this patient have such an appearance?

 The patient had a vestibuloplasty.

3. Why is this appearance rarely seen nowadays?

 While this approach yields a larger available area for acrylic resin coverage, it
also risks a so-called “witch’s chin” appearance as a result of localized altered
morphology and muscle function. This procedure is being used less and less
nowadays due to routine implant placement and advances in bone grafting
combined with implant prescriptions.
During a try-in visit for a patient with completely edentulous upper and lower
arches, you noticed that the patient can only speak with the teeth clenched together
and the teeth were making clicking sounds when the patient was pronouncing the
letter “m”.

1. What is the cause of this situation?

 Increased vertical dimension of occlusion / reduced interocclusal space.

2. How would you go about correcting this matter?

 The teeth will have to be removed from one of the record bases and then
replaced with a wax rim and then a new proper jaw relation record can be
made.
The following is a photograph of a polyvinyl siloxane impression material.

1. What is the consistency of this material?

 Medium body PVS impression material.

2. Why is this specific material called “monophase”?

 Because it can be used as a single mix. Part of it will fill the impression tray,
and the other part will be used as a syringe material to inject on and around
the abutments (in the case of a Cr-Co RPD). In the case of a complete
denture, it is also considered as a single mix impression technique.
Examine the following picture.

1. The length of the portion of the post that is inside the root canal can be
measured as being:

 2/3 the root length.


 1/2 the distance between the crestal bone and root apex.
 Equal to the clinical crown.

2. Ideally, the best length measurement of the post portion that is inside the
root canal is:

 Equal to the clinical crown.


Examine the following radiograph of a patient with a chief complaint of missing
teeth in the lower back areas.

1. What is the Kennedy classification of the lower arch?

 Class I.

2. If you were to construct a Cr-Co removable partial denture for the lower
arch, what is the main issue with this prosthesis choice in this case?

 The upper posterior teeth are supra-erupted so the space in the lower
posterior areas will be reduced and will not be enough for a Cr-Co RPD.
Examine the following photograph of a metal-ceramic bridge.

1. During tooth preparation for a metal-ceramic bridge, what type of finish


line will you place on the buccal and the lingual surface, and at what depth
will you place the finish line?

 Buccal surface: shoulder finish line, 1.0 mm depth.


 Lingual surface: chamfer finish line, 0.5 mm depth.

2. Regarding axial reduction, what is the minimum thickness required for the
metal in case of an axial wall to be restored with metal only and in case of
an axial wall to be restored with metal veneered with porcelain to resist
deformation, bending, and fracture?

 Axial wall to be restored with metal only: 0.5-1.0 mm


 Axial wall to be restored with metal veneered with porcelain: 0.3 mm.
Examine the following photograph.

1. Give 2 indications for tissue conditioner in complete denture.

 Knife edge ridge.


 Treatment of abuse or traumatize tissue.
 Functional impression.
Examine the following clinical photograph.

1. What could be the possible causes for above porcelain chipping?

 Flexible metal or wrong type of metal substructure.


 Thick porcelain.
 Improper metal surface treatment.
Examine the following photograph.

1. Identify this lab procedure.

 Beading and boxing.

2. What is the purpose of this procedure?

 Preservation of the functional depth and width of the sulcus of the recorded
final impression.
Examine the following clinical photograph.

1. Identify this maxillary landmark.

 Incisive papilla.

2. Mention one fact about this landmark.

 Denture should be relived over incisive papilla.


 Useful guide for setting up maxillary teeth.
Examine the following clinical photograph.

1. Identify the maxillary relieving land mark?

 Mid palatine raphe.

2. List TWO consequences occur in case of failure to relieve this area.

 Soreness of mucosa.
 Acts as fulcrum rocking.
 Midline fracture of the denture.
Examine the following photograph.

1. Identify this clinical procedure.

 Border molding.

2. List TWO materials that can be used for this procedure?

 Putty elastomeric impression material.


 Greenstick compound.
Examine the following clinical photograph.

1. Identify marked landmark.

 Posterior palatal seal.

2. Write TWO advantages of it?

 Retention.
 Reduce tendency of gag reflex.
 Prevent forming of the gap between soft palate and the denture (no food
impaction).
 Compensate for polymerization shrinkage during curing.
Examine the following photograph.

1. What is the use of window in the tray?

 Flabby ridge.

2. If this tray was spaced, which materials would be used with it?

 PVS.
In this clinical photograph, teeth number #11, #13, and #15 are abutment teeth.

1. What is tooth number 13 called?

 Pier abutment.

2. What is the best fixed partial denture type which could be used to replace
the missing teeth?

 Fixed movable bridge.


Examine the following photograph.

1. What type of major connector is used in this Cr-Co RPD?

 Linguoplate mandibular major connector.

2. Mention the indications for the use of this major connector.

 When the lingual frenum is high or the space available for a lingual bar is
limited.
 In Class I situations in which the residual ridges have undergone excessive
vertical resorption.
 For stabilizing periodontally weakened teeth, splinting with a linguoplate can
be of some value when used with definite rests on sound adjacent teeth.
 When the future replacement of one or more incisor teeth will be facilitated by
the addition of retention loops to an existing linguoplate.
Examine the following photograph.

1. What is the type of the clasp assembly on tooth #34 and tooth #47?

 Tooth #34: RPA clasp assembly.


 Tooth #47: circumferential clasp assembly.
Examine the following photograph.

1. What is the name of this bur?

 Torpedo cylindrical carbide bur.

2. What type of finish line does this bur produce?

 Chamfer.
Examine the following picture.

1. What is the Kennedy classification shown in (A)?

 Class III modification 1.

2. What is the Kennedy classification shown in (B)?

 Class II.
Examine the following photograph.

1. Identify the marked structure.

 Buccal shelf area.

2. What is the significance of this structure?

 It lies at a right angle to the vertical occlusal force. This makes it suitable as a
primary stress bearing area for lower denture.
Examine the following photographs.

1. Identify the marked structure.

 Retromolar pad.

2. What is the significance of this structure?

 The bone beneath it does not resorb secondary to the pressure associated
with denture use making it one of the primary support areas and it will prevent
distal denture displacement.
Clinical Dentistry OSCE Questions
Department of Oral Surgery

A 36-year-old lady presents with severe pain a few days after tooth extraction.
She complains of a distressingly severe pain from an extraction socket on the left
side of her mandible. The pain is localized to the extraction socket and is not
sensitive to hot or cold. It is a constant, dull, boring pain unrelieved by aspirin or
paracetamol preparations. It prevents the patient performing any normal activity and
kept her awake last night. The patient underwent surgical removal of the lower left
third molar tooth at her dentist 4 days ago. The extraction had proved more difficult
than expected and involved repeated attempted elevation and a small amount of
bone removal using a bur. Following the extraction, bleeding stopped normally. The
extraction site had been tender but apparently was healing slowly until the pain
started yesterday. Since then, she has also noticed halitosis and a bad taste.

1. What are the causes of


post-extraction pain?

 Pain from surgical trauma.


 Osteomyelitis.
 Dry socket.
 Fractured mandible.
 Retained root fragments.

2. Based on the history and


the clinical photograph,
what is the most likely
cause of the pain?

 Dry socket.
A 48-year-old man presents to you in general dental practice with a gingival
swelling. The patient complains of a lump on the gum at the front of his mouth on the
left side. It sometimes bleeds, usually after brushing or eating hard food but it is not
painful. The swelling has been present for 4 months and has grown slowly during
this period. It was never painful but now looks unsightly. The patient gives no history
of other mucosal or skin lesions. The patient has hypertension, controlled with
atenolol 50 mg daily. The patient is partially dentate and has relatively few and
extensively restored teeth. He wears an upper partial denture. The root of the upper
lateral incisor is present and its carious surface lies at the level of the alveolar ridge.
The teeth on each side of the lesion are restored with metal-ceramic crowns. There
is a mild degree of marginal gingivitis. Most of the interdental papillae are rounded
and marginal inflammation is present around crowns. Flecks of subgingival calculus
are visible.

1. From the information in the


history and examination so far,
what is your differential
diagnosis?

 Pyogenic granuloma.
 Fibrous epulis.
 Peripheral giant cell
granuloma.
 Papilloma.
 Benign hamartoma or neoplasm.
 Malignant neoplasm.

2. What is the most likely diagnosis?

 Pyogenic granuloma.

3. What is your treatment plan?

 Excision biopsy.
 Removal of causative factors, i.e., plaque and calculus.
 Provide treatment for the generalized gingivitis.
 Extract or restore the lateral incisor root.
Examine the following clinical photograph.

1. What is this surgical procedure called?

 Alveoloplasty.

2. What is the name of the instrument that is being used in the photograph?

 Bone file.
Examine the following clinical photographs of a surgical procedure.

1. What is this surgical procedure called?

 Palatal rotational flap.

2. When is this surgical procedure used?

 This surgical procedure is used in the closure of oroantral communication /


fistula.
Examine the following radiograph.

1. What is the angulation of impaction for tooth #38?

 Mesioangular.

2. Which type of mandibular third molar impaction is the most difficult to


extract and which one is the easiest?

 The most difficult type to extract is distoangular lower third molar impaction,
while the easiest type to extract is mesioangular lower third molar impaction.
A 22-year-old male patient reported to your clinic with a complaint of swelling
present over the left side of his face for 2 months. Swelling had no specific onset
and had gradually increased in size. There was no history of pain or discomfort
associated with the swelling. Medical history was non-contributory. There was a
history of unsuccessful minor surgical intervention performed 5 days before with an
attempt to extract an impacted wisdom tooth from the lower left back tooth region.
Extraoral examination revealed a diffuse swelling present over the left lower side of
the face extending from the angle of the mandible to the left temple region and was
around 5 cm × 3 cm in dimensions. On palpation, the swelling was hard. Aspiration
was performed, which revealed a viscous, straw-colored fluid. An
orthopantomograph was taken as shown below.

1. Based on the radiograph, this lesion is showing a:

 Unilocular radiolucency.

2. Based on the provided information, what is your diagnosis for this case?

 Dentigerous cyst.
The following is a photograph of the topical anesthetic gel used in your clinics.

1. What is the active anesthetic agent in this type of anesthetic gel?

 Benzocaine.

2. What is its concentration percentage?

 20%.
Examine the surgical instrument shown in the following photographs.

1. Identify the instrument.

 Maxillary right molar forceps.

2. What is the essential information that must be obtained from a radiograph


before extracting the tooth for which this instrument is used?

 The proximity of the molar roots to the floor of the maxillary sinus.
 Configuration of the roots - the number, curvature, divergence, and size.
A 55-year-old male presents to the dental clinic with a chief complaint of 'pain
in the right lower back teeth.' Clinical examination shows grossly decayed mandibular
right 1st and 2nd molars, and 2nd premolar as shown in the below clinical photograph.
The patient was admitted 2 years ago in the hospital and had a prosthetic cardiac
valve placed. He says he has been taking medications since then to keep his blood
thin as advised by his physician but does not know the name of the medications. He
does remember that the same medicine was given to him even before the valve
replacement surgery to manage minor aches and pains. He gives no other medical
history of significance except
that it was found that during the
last hospital visit he is allergic
to penicillin. The patient says
he is fit and well and he walks
regularly and is not breathless
while doing routine work. Blood
pressure is measured and
found to be 130/80 mm Hg;
pulse rate: 70/min. A surgical
extraction is planned for the two
mandibular molars and the
mandibular 2nd premolar.

1. What medication must this patient be given before surgery and why?
 The patient must be given antibiotics for prophylaxis because he is at risk of
infective endocarditis.
 Clindamycin 600 mg tablet is given orally one hour before the procedure.

2. What type of medications is the patient taking to keep his blood thin?
Name two of the most commonly used drugs in this category.
 Antiplatelet medications (e.g., Aspirin, Clopidogrel).

3. Should these medications be stopped before or after surgery and why?


 These medications should not be stopped by the dentist before dental surgery
procedures such as extraction, because stopping antiplatelet therapy
increases the risk of myocardial infarction. The patient is at greater risk of
permanent disability or death if antiplatelet medication is stopped prior to a
dental procedure than if continued.
 Postoperative bleeding after dental procedures can be controlled using local
hemostatic measures in patients taking oral antiplatelet medication. The
patient can be taken up for extraction provided his INR (International
Normalized Ratio) is within normal range.
A patient with prosthetic heart valve attended your dental surgery, and requires
extraction of a lower second molar.

1. What preoperative investigations do you need to obtain from him for an


uneventful surgery?

 INR; the patient will be on anticoagulants.

2. Explain to him what the concerns are with respect having dental
extraction?

 Severe Hemorrhage if INR levels are high.

3. How would you manage the above-mentioned concerns?

 If within limits below 3.5, and can be treated without hospitalization minor oral
surgery can be done; the use of local hemostatic agents (foamed gelatin,
collagen sponge) with primary closure can control bleeding and achieve
hemostasis.

A patient with chronic renal failure attended your dental surgery, and requires
extraction of maxillary third molar.

1. What further investigations do you need to obtain from him?

 The patient will be on anticoagulants (Renal Dialysis); Screening for bleeding


disorders (bleeding time, platelet count, hemoglobin, INR).

2. Explain to him what the concerns are with respect to dental extraction.

 Due to low platelet count, and prolonged bleeding time, hemostatic measures
must be taken after extraction.
Examine the following picture of the mandible.

1. Identify the remaining unlabeled nerves.


Examine the following picture.

1. Label the nerves in the picture.

2. What are the areas they supply?


Examine the following photograph.

1. Identify the foramen marked by the wire.

 Posterior alveolar nerve foramen.

2. Mention the name of the nerve that passes through this foramen.

 Posterior alveolar nerve of the maxillary branch of the trigeminal nerve (V).
Examine the following photograph.

1. Identify the remaining unlabeled lymph nodes


Examine the following photograph.

1. The dentist is in the operating room in the photograph above. Are there
any violations and why?

 Violation of infection control protocol.


 The dentist is holding the file with contaminated gloves.
Examine the following photograph.

1. Mention 3 items that can be disposed of in this container.

 Scalpel blades.
 Anesthetic needles.
 Suture needles
 Endodontic files.
Examine the following photograph.

1. How much adrenaline (in milligrams per milliliters) is present in this


cartridge?

 0.01mg/ml x 1.8 = 0.018mg/ml epinephrine.

2. What is the chemical classification of local anesthetic drug present in this


solution?

 Amide (Lidocaine).

3. How many milligrams of the local anesthetic agent are contained within
this cartridge?

 2%= 20mg/ml x 1.8 = 36mg/ml.

You are planning to extract a badly destructed lower right second molar.

1. Mention the most common injection technique for extracting the lower
right second molar

 Inferior alveolar nerve block.


 Long buccal nerve block.

2. What other nerves could get anesthetized with this injection technique.

 Incisive, mental, lingual.


Examine the following photographs.

1. Identify the surgical instruments and their uses in (A), (B), and (C).

 A: Curved Hemostat, control bleeding, crush and open fibrous septa in


abscess drainage.
 B: Winter Cross bar, used to elevate mandibular molar roots.
 C: Upper third molar forceps, used to extract upper third molars.
Examine the following photograph.

1. Identify the injection technique.

 Gow-Gates mandibular nerve block.

2. What is the main landmark that can guide the operator in this injection?

 Height of injection established by placement of the needle tip just below the
mesiolingual (mesiopalatal) cusp of the maxillary second molar.
Examine the following radiograph.

1. Mention the most common complication that can develop after the
extraction of the maxillary first molar on the shown in the periapical
radiograph above?

 Oro-antral communication.

2. How can you manage this condition?

 >2 mm: no further treatment.


 2-6 mm: figure of 8 suture.
 <7 mm: buccal advancement flap, palatal rotation flap, or pedicle flap to close
the socket.
Examine the following photograph of a common material that can be used in
the oral surgical procedure.

1. Mention the name of this material?

 Gelatin Sponge (Gelfoam).

2. What is its main indication?

 Hemostatic agent after extraction.

3. What is the mechanism of its action?

 When applied directly to bleeding surfaces, arrest bleeding by the formation of


an artificial clot and by producing a mechanical matrix that facilitates clotting.
Examine the following picture.

1. Mention the name of this suturing technique.

 Vertical mattress suture.

2. Mention one main indication for using this suturing technique.

 Adapt the tissue flaps tightly to underlying structures when a bone graft or
regenerative membrane is used, or during dental implant surgery.
Examine the following photograph.

1. What is the compression-to-breath ratio for this patient?

 30:2

2. When should you stop doing CPR on this patient?

 When the person begins to move.


 When emergency help arrives.
 When you are exhausted and unable to continue.
Examine the following clinical photograph of a surgical attempt at removal of a
root from the maxillary sinus.

1. What is this procedure called?

 Caldwell-Luc surgery.

2. Which tooth root is most likely to be at risk for such a complication?

 Palatal root of the maxillary first molar.


Examine the following photograph of the spread of a dental infection.

1. Identify the facial space infection.

 Buccal space infection.

2. Which teeth can likely cause this?

 Maxillary premolars, molars, and mandibular premolars.

3. Write an appropriate prescription for the patient’s pain and infection?

 Amoxicillin/clavulanic acid, 1.2g IV q 6-8 hrs for 3 days, 625 mg.


 Amoxicillin/clavulanic acid every 8 hrs for 5 days.
The radiograph below shows a broken instrument left behind after the surgical
removal of an impacted lower third molar..

1. What potential complications can the patient have?

 Post-extraction granuloma may lead to osteomyelitis?

2. How can this be managed?

 Removal, curettage of granuloma.


Examine the following photograph.

1. Mention the indication for the following medication.

 Dextrose Ampule→ Hypoglycemia.

2. Mention the dosage and route of administration.

 Venous access should be gained and a 50 ml ampule of 50% dextrose in


water should be administered IV over 2-3 minutes.
Examine the following pictures.

1. Identify the suturing technique shown in the pictures.

 Locking continuous suture.

2. Give an example each of a synthetic absorbable and non-absorbable


suture material.

 Absorbable: polyglycolic acid.


 Non-absorbable: silk.
Pediatric Dentistry

Q1: Radiographs of a preschool child with _____________will show obliteration of the pulp chambers with
secondary dentin, a characteristic finding.

A. amelogenesis imperfecta

B. dentinogenesis imperfecta

C. fluorosis

D. enamel hypoplasia

Answer: dentinogenesis imperfect

Q2: ____________ is a condition produced when two tooth buds are joined together during development
and appear as a macrodont.

A. concrescence

B. gemination

C. fusion

D. dens in dente

Answer: fusion

Q3: What condition is depicted below?

A. enamel hypoplasia

B. erythroblastosis fetalis

C. nursing bottle caries

D. dentinal dysplasia

Answer: enamel hypoplasia

Q4: Excessive fluoride levels in drinking water are associated with fluorosis. Fluoride levels in excess of
____________ begin to pose a risk for fluorosis.

A. one part per million

B. two parts per million

C. three parts per million

D. four parts per million

Answer: three parts per million


Q5: ________________ is the bedrock strategy on which all of pediatric dental behavior management
rests.

A. Tell-Show-Do (TSD)

B. positive reinforcement

C. distraction

D. nonverbal communication

Answer: Tell-Show-Do (TSD)

Q6: When treating a child who is obviously afraid, the dentist should:

A. use restraint

B. use the hand-over-mouth technique (HOME)

C. permit the child to express his fear

D. avoid all reference to the child's fear


Answer: permit the child to express his fear — identify the fear

Q7: The process of shaping a patient's behavior through appropriately timed feedback is called:

A. Tell-show-do

B. voice control

C. positive reinforcement

D. distraction

E. nonverbal communication

Answer: positive reinforcement

Q8: All of the following procedures have proved beneficial in treating an intellectually disabled child
EXCEPT one. Which one is the EXCEPTION?

A. speak slowly and in very simple terms

B. listen carefully to the patient

C. schedule long appointments

D. ask the patient if there are any questions about anything you will be doing

Answer: schedule long appointments


Q9: The management of a child who must undergo dental extractions is based on which of the following
factors?

A. the age and maturity of the child

B. the past medical and dental experiences that might influence the behavior of the child

C. the physical status of the child

D. the length of time and amount of manipulation necessary to accomplish the surgery

E. all of the above

Answer: all of the above

Q10: All of the following instances may make the use of a rubber dam impractical EXCEPT one. Which
one is the EXCEPTION?

A. the presence of fixed orthodontic appliances

B. a patient with congested nasal passages or other nasal obstruction

C. a very nervous or anxious patient

D. a recently erupted tooth that will not retain a clamp

Answer: a very nervous or anxious patient

Q11: Which of the following is often caused by a strain of coxsackie A virus?

A. herpangina

B. scarlet fever

C. diphtheria

D. mumps

Answer: herpangina

Q12: Both necrotizing ulcerative gingivitis (NUG) and diphtheria have a viral etiology.

Both NUG and diphtheria are characterized by pseudomembrane formation.

A. the first statement is true, the second is false

B. the first statement is false, the second is true

C. both statements are true

D. both statements are false

Answer: the first statement is false, the second is true


Q13: A Class II cleft palate involves what structures?

A. hard and soft palates

B. soft palate only

C. alveolar process only

D. hard palate only

Answer: hard and soft palates

Q14: Ectodermal dysplasia is characterized by: Select all that apply.

A. sparse hair

B. lack of sweat glands

C. oversized crowns

D. elongated roots

E. normal mental status

F. an enlarged mandible

G. absence of teeth

Answer: sparse hair, lack of sweat glands, normal mental status, absence of teeth

Q15: All of the following are characteristics of primary herpetic gingivostomatitis as seen in the picture
below EXCEPT one. Which one is the EXCEPTION?

A. caused by HPV-2

B. disease is self-limited lasting 10-14 days

C. generally affects children under the age of three with prodromal


symptoms

D. can occur on keratinized and unkeratinized mucosa

Answer: caused by HPV-2


Q16: Cellulitis in a child is easier to treat than in adults.

The most common causative organisms of cellulitis are group A streptococci and Staphylococcus aureus.

A. both statements are true

B. both statements are false

C. the first statement is true, the second is false

D. the first statement is false, the second is true

Answer: the first statement is false, the second statement is true

Q17: Historically, the incidence of dental decay in individuals with Down syndrome has been reported to be
____________ . The rate of periodontal disease in these persons has been reported to be
_______________ .

A. extremely high, extremely low

B. relatively the same as the general population, extremely high

C. extremely low, relatively the same as the general population

D. extremely low, extremely high

Answer: extremely low, extremely high

Q18: Type 1 diabetes is the most common form seen and children and exhibits all of the following
subjective characteristics EXCEPT one. Which one is the EXCEPTION?

A. polyuria

B. polyphagia

C. polydipsia

D. polymyalgia

Answer: polymyalgia

Q19: The hemangioma is usually treated by conservative surgical excision. Hemangiomas are more
common in boys than girls.

A. both statements are true

B. both statements are false

C. the first statement is true, the second is false

D. the first statement is false, the second is true

Answer: the first statement is true, the second is false


Q20: Which of the following is very common in a child with achondroplasia?

A. Class I malocclusion

B. Class II malocclusion

C. Class III malocclusion

D. none of the above

Answer: Class III malocclusion

Q21: Craniosynostosis is the premature closure of one or more sutures of the skull, resulting in an
abnormal head shape. Syndactyly is the soft tissue or bony fusion of the fingers and toes.

The most common craniosynostotic syndrome occurring without syndactyly is Apert syndrome.

A. both statements are false

B. the first statement is true, the second is false

C. the first statement is false, the second is true

D. both statements are true

Answer: the first statement is true, the second is false

Q22: Which of the following are true concerning a young epileptic who has a grand mal seizure in the
dental office? Select all that apply.

A. it is generally fatal

B. it is best treated by injecting insulin

C. they generally recover if restrained from self-injury and oxygen is maintained

D. it can be prevented with antibiotics

Answer: they generally recover if restrained from self-injury and oxygen is maintained

Q23: The most common of the craniofacial malformations is:

A. bifid tongue

B. macroglossia

C. cleft palate and cleft lip

D. anodontia

Answer: cleft palate and cleft lip


Q24: Which type of leukemia is referred to as the "leukemia of childhood"?

A. acute myeloid leukemia

B. chronic myelocytic leukemia

C. acute lymphocytic leukemia

D. chronic lymphocytic leukemia

Answer: acute lymphocytic leukemia

Q25: An important diagnostic finding in congenital porphyria is the presence of red-brown teeth in both the
deciduous and permanent dentition.

The oral mucosa is rarely affected in porphyrias.

A. both statements are true

B. both statements are false

C. the first statement is true, the second is false

D. the first statement is false, the second is true

Answer: both statements are true

Q26: In baby bottle decay, the teeth typically are decayed in a specific order. Place the teeth in the correct
order.

• mandibular anterior teeth


• mandibular posterior teeth
• maxillary anterior teeth
• maxillary posterior teeth

Answer: 1. maxillary anterior teeth, 2. maxillary posterior teeth, 3. mandibular posterior teeth, 4.
mandibular anterior teeth

Q27: All of the following statements concerning recurrent aphthous ulcers (canker sores) are true EXCEPT
one. Which one is the EXCEPTION?

A. they occur in women more than men

B. they may occur at any age, but usually first appear between the ages of 10 and 40 years

C. the cause is a Coxsackie virus

D. they appear to be associated with stress

E. they usually appear on nonkeratinized oral mucosa, including the inner surface of the cheeks and lips,
tongue, soft palate, and the base of the gingiva
Answer: the cause is a Coxsackie virus
Q28: Cretinism is a deficiency disease caused by the congenital absence of:

A. insulin

B. thyroxine

C. calcitonin

D. epinephrine

Answer: thyroxine

29: In children with cystic fibrosis, halitosis is common.

In children with cystic fibrosis, dental development is normal but eruption is delayed.

A. both statements are true

B. both statements are false

C. the first statement is true, the second is false

D. the first statement is false, the second is true

Answer: the first statement is true, the second is false

Q30: The oral lesions are pathognomonic of:

A. smallpox (variola)

B. German measles (rubella)

C. mumps

D. measles (rubeola)

Answer: measles (rubeola)

Q31: The principal characteristics of attention-deficit/hyperactivity disorder are all of the following EXCEPT
one. Which one is the EXCEPTION?

A. inattention

B. intellectually disabled

C. hyperactivity

D. impulsivity

Answer: intellectually disabled


Q32: A 10-year-old girl with a history of congenital heart disease visits a dental clinic. Which one of the
following dental procedures necessitates antibiotic prophylaxis to prevent infective endocarditis?

A. placement of orthodontic appliances

B. administration of routine local anesthetics

C. dental radiography

D. gingival pocket surgery

Answer: gingival pocket surgery

Q33: A pediatric patient presents to your office for a restoration on #T MO. The patient weighs 40 kg. What
is the maximum amount of 2% lidocaine 1:100,000 that you can safely administer to this patient?

A. 3.8 carpules

B. 4.2 carpules

C. 4.8 carpules

D. 5.2 carpules

Answer: 4.8 carpules

Q34: ___________, alone or in combination with other drugs, is the most common sedative agent used in
pediatric dentistry.

A. pentobarbital

B. secobarbital

C. paraldehyde

D. chloral hydrate

Answer: chloral hydrate

Q35: A 15-year-old female has lived in a nonfluoridated area all of her life. Which of the following is most
likely to occur in this young lady when she moves to a community where the drinking water naturally
contains 6 ppm of fluoride?

A. 50% reduction in dental caries

B. moderate dental fluorosis

C. an increase in the amount of fluoride stored in her bones

D. gastrointestinal problems

Answer: an increase in the amount of fluoride stored in her bones


Q36: Fluoridation has several mechanisms for caries inhibition.

Included are enhancement of remineralization of enamel, inhibition of glycolysis, and the incorporation of
fluoride into the enamel hydroxyapatite crystal.

A. both statements are true

B. both statements are false

C. the first statement is true, the second is false

D. the first statement is false, the second is true

Answer: both statements are true

Q37: Which of the following fluoride therapies should be recommended to a 13-year-old child who is prone
to decay and lives in a community where the water is fluoridated at an appropriate level? Select all that
apply.

A. professionally applied fluoride every 6 months

B. fluoride toothpaste

C. dietary fluoride supplements

D. a low concentration fluoride mouth rinse

E. a high concentration fluoride mouth rinse

Answer: • professionally applied fluoride every 6 months


• fluoride toothpaste
• a low concentration fluoride mouth rinse

Q38: All of the acidulated phosphate fluoride products should be applied for_____________ in order to
achieve the best results.

A. 1 minute

B. 2 minutes

C. 3 minutes

D. 4 minutes

Answer: 4 minutes
Q39: Before fluoride applications:

A. vaseline is applied to protect any teeth with sealants

B. the teeth should be dry to prevent dilution of the fluoride concentration

C. all bacterial plaque must be removed to prevent interference with fluoride uptake by the enamel
surface

D. patients should be placed in a semi-supine position

Answer: the teeth should be dry to prevent dilution of the fluoride concentration

Q40: You examine a 10-year-old boy in your practice and determine that he has multiple carious lesions.
The family resides in a rural area and drinks well water. What is your advice regarding fluoride
supplementation?

A. prescribe fluoride tablets for the patient immediately

B. arrange for a sample of the patient's well water to be sent to a laboratory to assess the amount of
naturally occurring fluoride in the water. Then prescribe the appropriate dose of fluoride
supplementation in lieu of the fluoride that is occurring in the water, if any.

C. the child is too old for fluoride supplementation to be of benefit, so you do not recommend it

D. none of the above


Answer: arrange for a sample of the patient's well water to be sent to a laboratory to assess the
amount of naturally occurring fluoride in the water. Then prescribe the appropriate dose of fluoride
supplementation in lieu of the fluoride that is occurring in the water, if any.

Q41: Clinical studies demonstrate that acidulated phosphate fluoride is most effective at what pH?

A. 1.0

B. 2.5

C. 3.2

D. 5.5

Answer: 3.2

Q42: The lethal dose of fluoride for a typical 3-year-old child is approximately:

A. 100 mg

B. 200 mg

C. 350 mg

D. 500 mg

Answer: 500 mg
Q43: What is the most effective method of reducing the dental caries problem in the general population?

A. school water fluoridation

B. fluoridation of the communal water supply

C. fluoride rinses at home

D. frequent dental visits

Answer: fluoridation of the communal water supply

Q44: The most common congenitally missing primary tooth is the:

A. primary mandibular canine

B. primary maxillary lateral incisor

C. primary maxillary canine

D. primary mandibular first molar

Answer: primary maxillary lateral incisor — Note: This is rare

Q45: A 15-month-old child would normally have all of the following teeth erupted EXCEPT one. Which one
is the EXCEPTION?

A. primary lateral incisors and canines

B. primary canines and first molars

C. primary canines and second molars

D. primary central and lateral incisors

E. primary first and second molars

Answer: primary canines and second molars

Q46: All of the following are true when comparing the normal child periodontium to the normal adult
periodontium EXCEPT one. Which one is the EXCEPTION?

A. there is greater blood and lymph supply

B. the alveolar crest is flatter

C. the cementum is thinner and less dense than that of the adult

D. gingival pocket depths are larger

E. attached gingiva is much wider

Answer: attached gingiva is much wider


Q47: The permanent maxillary lateral incisor typically erupts when a child is about:

A. 5-6 years old

B. 8-9 years old

C. 11-12 years old

D. 13-14 years old

Answer: 8-9 years old

Q48: The crowns of all 20 primary teeth begin to calcify between:

A. 1.5 to 2 months in utero

B. 3.5 to 6 months in utero

C. 7.5 to 9 months in utero

D. 10 to 12 months in utero

Answer: 3.5 to 6 months in utero - (14-24 weeks in utero)

Q49: Nonsuccedaneous teeth include all of the following EXCEPT one. Which one is the EXCEPTION?

A. the permanent maxillary and mandibular premolars

B. the permanent maxillary and mandibular first molars

C. the permanent maxillary and mandibular second molars

D. the permanent maxillary and mandibular third molars

Answer: the permanent maxillary and mandibular premolars

Q50: All of the following syndromes demonstrate microdontia EXCEPT one. Which one is the
EXCEPTION?

A. Ectodermal dysplasia (hypohidrotic type)

B. Hemifacial microsomia

C. Down syndrome

D. Otodental syndrome

Answer: Otodental syndrome

Q51: The dental formula for the primary teeth in humans is:
A.

B.

C.

D.

Answer:

Q52: The dental formula for the permanent teeth in humans is:

A.

B.

C.

D.

Answer:
Q53: When do the permanent teeth begin to calcify?

A. At birth

B. 1 month

C. 4 months

D. 1 year

Answer: At birth

Q54: The primary maxillary first molars are usually exfoliated when a child is about:

A. 6-7 years old

B. 7-8 years old

C. 9-11 years old

D. 12-14 years old

Answer: 9-11 years old

Q55: All of the following statements are true EXCEPT one. Which one is the EXCEPTION?

A. the primary teeth are darker in color than the permanent teeth

B. for primary teeth, the interproximal contacts are broader and flatter than permanent teeth

C. the pulp cavities are proportionately larger in the primary teeth

D. in general, the crowns of primary teeth are more bulbous and constricted than their permanent
counterpart

E. the pulp horns of primary teeth are closer to the surface of the tooth

F. the crown surfaces of all primary teeth are much smoother than the permanent teeth (in other words,
there is less evidence of pits and grooves)

G. primary teeth have thinner enamel

Answer: the primary teeth are darker in color than the permanent teeth

Q56: The primary mandibular canine typically erupts when a child is about:

A. 6-10 months old

B. 10-16 months old

C. 17-23 months old

D. 23-31 months old

Answer: 17-23 months old


Q57: The sum of the mesiodistal widths of the primary molars in any one quadrant is:

A. 5-10 mm greater than the permanent teeth that succeed them - premolars

B. 2-5 mm less than the permanent teeth that succeed them - premolars

C. 2-5 mm greater than the permanent teeth that succeed them - premolars

D. 5-10 mm less than the permanent teeth that succeed them - premolars

Answer: 2-5 mm greater than the permanent teeth that succeed them - premolars

Q58: The most frequently taken radiographic views in pediatric dentistry are:

A. molar bitewing radiographs

B. mandibular molar periapical radiographs

C. mandibular anterior periapical radiographs

D. maxillary molar periapical radiographs

Answer: molar bitewing radiographs

Q59: Which of the following is the most common primary tooth to be retained due to a congenitally missing
permanent successor?

A. maxillary second molars

B. maxillary first molars

C. mandibular second molars

D. mandibular first molars

Answer: mandibular second molars

Q60: All of the following are criteria for determining an ideal pattern of the dental arch for a 7-year-old child
EXCEPT one. Which one is the EXCEPTION?

A. tight proximal contacts

B. no tooth rotation

C. flat occlusal plane

D. no leeway space

Answer: no leeway space


Q61: Please order the following primary dentition in the correct sequence of eruption.

• canine
• central incisor
• first molar
• lateral incisor
• second molar

Answer: central incisor, lateral incisor, first molar, canine, second molar

Q62: When attempting a MO Class II amalgam preparation and filling on a primary tooth, you encounter a
very large mesial marginal ridge that resembles a cusp. You also notice a transverse ridge from
mesiolingual to mesiobuccal cusp that is rather large. This tooth proves difficult to restore, which tooth is
it?

A. mandibular first molar

B. maxillary first molar

C. mandibular second molar

D. maxillary second molar

Answer: mandibular first molar

Q63: Which of the following tooth is depicted by the occlusal picture


given below?

A. primary mandibular right first molar

B. primary mandibular right second molar

C. primary maxillary right first molar

D. primary maxillary right second molar

Answer: primary mandibular right second molar

Q64: The morphology of the primary mandibular second molar closely resembles that of the permanent
mandibular first molar. All of the following statements reflect the differences between the two types of teeth
EXCEPT one. Which one is the EXCEPTION?

A. there is a relative size difference in the distal cusps of the two teeth

B. there is variation in the mesiodistal width at the cervical portion of the two teeth

C. the roots of the primary molars are slender and longer

D. the primary mandibular molar has five cusps

Answer: the primary mandibular molar has five cusps


Q65: Which tooth is the only anterior tooth in either dentition to have a shorter incisocervical height than
the mesiodistal width?

A. the primary mandibular central incisor

B. the primary mandibular lateral incisor

C. the primary maxillary lateral incisor

D. the primary maxillary central incisor

Answer: the primary maxillary central incisor

Q66: Morphologically, the primary maxillary second molar strikingly resembles the:

A. permanent maxillary third molar

B. permanent maxillary second molar

C. permanent maxillary first molar

D. permanent mandibular second molar

Answer: permanent maxillary first molar - but they are smaller

Q67: A 10-1/2-year-old patient comes into your office. You are not sure whether his maxillary canines are
permanent or primary. Which of the following statements will help you determine whether they are
permanent or primary canines?

A. the cusp of the primary maxillary canine is much shorter than the cusp of the permanent maxillary
canine

B. the mesial cusp ridge on the primary maxillary canine is shorter than the distal cusp ridge; this is
opposite of all other canines

C. the cusp on the primary maxillary canine is much longer and sharper than the cusp on the permanent
maxillary canine

D. the primary maxillary canine is much narrower and longer than the permanent maxillary canine

Answer: the cusp on the primary maxillary canine is much longer and sharper than the cusp on the
permanent maxillary canine
Q68: The occlusal form of the ___________________ varies from that of any tooth in the permanent
dentition.

A. the primary mandibular first molar

B. the primary maxillary first molar

C. the primary mandibular second molar

D. the primary maxillary second molar

Answer: the primary maxillary first molar

Q69: An 11-year-old child traumatized a permanent maxillary central incisor some time ago. The tooth has
never been restored. It is now painful and there is evidence of swelling. A periapical x-ray discloses a
pathosis associated with the apex. The suggested treatment is:

A. pulpotomy

B. extraction

C. pulpectomy

D. observation

Answer: pulpectomy - root canal therapy

Q70: Indirect pulp treatment is a procedure performed in a tooth with:

A. a necrotic pulp

B. a deep carious lesion adjacent to the pulp

C. aperiapical radiolucency

D. pulp tissue that is irreversibly infected due to caries or trauma

Answer: a deep carious lesion adjacent to the pulp

Q71: A 4-year-old child presents with acute pain associated with a primary mandibular second molar that
has a large carious lesion with pulpal involvement. Radiographically, there is periapical pathology on the
distal root. The child is very cooperative and is able to tolerate long appointments. What is the preferred
choice of therapy for the primary mandibular second molar?

A. incision and drainage

B. pulpotomy

C. primary tooth endodontics (pulpectomy)

D. extraction

Answer: primary tooth endodontics (pulpectomy)


Q72: Which treatment is the proper one for a Class II fracture of a permanent tooth with an immature
apex?

A. pulpectomy

B. apply calcium hydroxide to exposed dentin and restore tooth with a permanent restoration

C. pulpotomy

D. observe

Answer: apply calcium hydroxide to exposed dentin and restore tooth with a permanent restoration

Q73: The first indication for a pulpotomy is carious invasion deep enough to cause mechanical exposure of
the pulp or inflammation of the coronal pulp.

Inflammation or infection of pulp tissue beyond the coronal pulp contraindicates a pulpotomy.

A. both statements are true

B. both statements are false

C. the first statement is true, the second is false

D. the first statement is false, the second is true

Answer: both statements are true

Q74: Direct pulp caps (DPC) involve direct placement of the capping material on the pulp. __________is
the agent that is most frequently used.

A. cavity varnish

B. glass ionomer

C. ZOE

D. calcium hydroxide

Answer: calcium hydroxide (CaOH)

Q75: One alternative to the traditional full-strength formocresol pulpotomy is the formocresol pulpotomy
using a diluted solution of formocresol. A _______ dilution has been recommended and has been shown
to produce good long-term therapeutic results.

A. one-third

B. one-quarter

C. one-fifth

D. three-fifths

Answer: one-fifth - 20%


Q76: All of the following statements are true EXCEPT one. Which one is the EXCEPTION?

A. the occlusal anatomy of primary teeth is not as defined as that of permanent teeth; therefore,
amalgam preps can be more conservative

B. enamel and dentin are thicker in primary teeth; therefore, amalgam preps are deeper

C. the pulpal horns of primary teeth are longer and pointed; therefore, amalgam preps must be
conservative to avoid a pulpal exposure

D. primary molars have an exaggerated cervical bulge that makes matrix adaptation much more difficult

E. the occlusal table is narrower on primary molars

Answer: enamel and dentin are thicker in primary teeth; therefore, amalgam preps are deeper

Q77: The success rates for mandibular nerve blocks are higher in children than in adults because of the
anatomy of less-developed mandibles. The anterioposterior position of the mandibular foramen is about
the same or slightly more mesial in children than in adults.

A. both statements are true

B. both statements are false

C. the first statement is true, the second is false

D. the first statement is false, the second is true

Answer: the first statement is true, the second is false

Q78: The bulbous, conically shaped primary teeth also affect the amount of extension of the occlusal
outline of the preparation. The general rule is that the occlusal outline is about __________ of the
intercuspal distance, between the buccal and lingual cusps, on the occlusal surface of primary molars.

A. one-half

B. one-third

C. two-thirds

D. three-quarters

Answer: one-third
Q79: Depth cuts can be used as a gauge to help establish the depth of the occlusal reduction when
preparing a primary tooth for a stainless steel crown. Approximately ________ of the occlusal surface
should be removed.

A. 1 to 1.5 mm

B. 3 to 3.5 mm

C. 4 to 4.5 mm

D. 5 to 5.5 mm

Answer: 1 to 1.5 mm

Q80: All of the following statements are true EXCEPT one. Which one is the EXCEPTION?

A. dental decay in primary teeth is an infectious process that can be very painful and can spread and
affect the development of the adult teeth

B. dental decay in primary teeth most often means there will be dental decay in the adult teeth

C. radiographically, the appearance of primary teeth differs from that of permanent teeth because of the
lower organic content found in primary teeth

D. dental decay in primary teeth tends to progress more rapidly from initial surface demineralization to
involvement of the dentin

E. the enamel layer of primary teeth is thinner in all dimensions as compared to permanent teeth

Answer: radiographically, the appearance of primary teeth differs from that of permanent teeth because of
the lower organic content found in primary teeth

Q81: Mandibular second premolars are different than other premolars in the mouth because they have
_____ developmental lobes.

A. two

B. three

C. four

D. five

Answer: five
Q82: Match the tooth structure-related terms on the left side with their appropriate description on the right
side.

• 1. von Korff’s fibers


• 2. von Ebner lines
• 3. Plexus of Raschkow
• 4. Reparative dentin
• A. Subodontoblastic nerves
• B. Tertiary dentin
• C. Incremental dentin growth
• D. Formation of dentin matrix

Answer: 1. D, 2. C, 3. A, 4. B

Q83: Enamel dysplasia, enamel hypoplasia, concrescence, and the formation of enamel pearls occur
during which stage in the life cycle of a tooth?

A. initiation

B. bud stage

C. cap stage

D. bell stage

E. apposition

F. calcification

G. eruption

H. attrition

Answer: apposition

Q84: Which structure functions to shape the root (or roots) and induce dentin formation in the root area so
that it is continuous with the coronal dentin?

A. dental papilla

B. dental lamina

C. dental sac

D. Hertwig sheath

Answer: Hertwig sheath


Q85: A 3-year-old patient reports to your office with an intrusion injury on teeth #E and #F (see
photograph). You inform the child's parents about the current standard of care regarding intruded teeth.
Which of the following statements best describes the current understanding regarding intruded primary
teeth?

A. the intruded teeth should be extracted

B. the intruded teeth should be left to reerupt

C. the therapeutic approach to intrusion injuries in


primary teeth is controversial. Some authors in the
field advocate extraction and some advocate
leaving the tooth to reerupt.

D. the intruded teeth should be gently moved into


position with gauze and stabilized by splinting

Answer: the therapeutic approach to intrusion injuries in primary teeth is controversial. Some authors in the
field advocate extraction and some advocate leaving the tooth to reerupt.

Q86: Discolored primary teeth that are symptom-free and show no radiographic changes are best treated
by:

A. no treatment

B. extirpation of the pulp tissue followed by the placement of ZOE paste in the root canal space

C. extraction

D. pulpotomy

Answer: no treatment

Q87: An 8-year-old patient presents to your office with a small pulp exposure on the permanent maxillary
left central incisor, resulting from a fracture of the tooth. The injury is about 1-hour old. Your clinical and
radiographic examinations show there are no other injuries. What is the indicated course of therapy at the
time of the emergency?

A. place a direct pulp cap and proceed with a glass ionomer Band-Aid restoration

B. begin partial pulpotomy therapy immediately

C. begin endodontic therapy immediately

D. schedule the patient for endodontic therapy as soon as possible, once the initial anxiety from the
traumatic episode has abated

Answer: place a direct pulp cap and proceed with a glass ionomer Band-Aid restoration
Q88: A 9-year-old patient has fractured the root of the permanent maxillary right lateral incisor. There is no
other identifiable injury. The fracture occurred around the middle of the root. What is the indicated course
of therapy at this time?

A. begin endodontic therapy immediately

B. extract the tooth and the root remnant if possible

C. do nothing if the tooth seems fairly stable

D. splint the tooth to the adjacent two or three teeth

Answer: splint the tooth to the adjacent two or three teeth

Q89: What is the main cause of failure of replantation of permanent teeth?

A. external root resorption

B. internal root resorption

C. irreversible pulpitis

D. reversible pulpitis

Answer: external root resorption

Q90: The patient below is a 5-year-old child with acute pain associated with tooth #K. If tooth #K was
extracted, what type of space maintainer would be needed?

A. band and loop space maintainer

B. distal shoe space maintainer (fixed)

C. distal shoe space maintainer (removable)

D. crown and loop space maintainer

Answer: distal shoe space maintainer (fixed)


Q91: What cement is the best choice for cementing a lower fixed bilateral holding arch in place?

A. zinc phosphate cement

B. zinc oxide eugenol cement

C. IRM

D. glass ionomer cement

Answer: glass ionomer cement

Q92: A mother of a 6-year-old female reports that her daughter has complained of a severe spontaneous
pain on the upper right side of her mouth. Your examination indicates a large lesion on the distal aspect of
the primary maxillary right first molar which extends to the pulp. All other maxillary teeth are present and
are noncarious. You decide that extraction of the tooth is warranted. What type of space maintainer will
you advise for the patient?

A. maxillary right removable unilateral appliance

B. maxillary removable bilateral appliance

C. maxillary right band and loop appliance

D. distal shoe space maintainer

Answer: maxillary right band and loop appliance

Q93: The photograph shows a maxillary fixed bilateral space maintainer. This type of space maintainer
also is known as a:

A. Frankel appliance

B. Nance appliance

C. Herbst appliance

D. Ricketts appliance

Answer: Nance appliance


Periodontics

Q1: All of the following statements concerning an autogenous free gingival graft are true EXCEPT one.
Which one is the EXCEPTION?

A. it can be placed to prevent further recession

B. it can be used to effectively widen the attached gingiva

C. it retains its own blood supply and is not dependent on the bed of recipient blood vessels

D. the greatest amount of shrinkage occurs within the first 6 weeks

E. it is also useful for covering nonpathologic dehiscences and fenestrations

Answer: it retains its own blood supply and is not dependent on the bed of recipient blood vessels

Q2: Most root amputations involve the:

A. mandibular first and second premolars

B. maxillary first and second molars

C. maxillary canines

D. mandibular molars with buccal and lingual class II or III furcation involvements

Answer: maxillary first and second molars

Q3: The main goal of osseous recontouring (surgery) is:

A. to cure periodontal disease

B. to eliminate the existing microflora

C. to eliminate periodontal pockets

D. to change the existing microflora

Answer: eliminate periodontal pockets – to reshape the marginal bone to resemble that of the
alveolar process undamaged by periodontal disease
Q4: All of the following statements regarding periodontal flaps are true EXCEPT one. Which one is the
EXCEPTION?

A. full-thickness periodontal flaps involve reflecting all of the soft tissue, including the periosteum, to
expose the underlying bone

B. the partial-thickness periodontal flap includes only the epithelium and a layer of the underlying
connective tissue

C. both full-thickness and partial-thickness periodontal flaps can be displaced

D. flaps from the palate are considered easier to be displaced than those from any other region

E. flaps should be uniformly thin and pliable

Answer: flaps from the palate are considered easier to be displaced than those from any other
region

Q5: There are various distal flap approaches used for retromolar reduction. The simplest is the:

A. gingivectomy

B. apically positioned flap

C. distal wedge

D. laterally positioned flap

Answer: distal wedge

Q6: Contraindications to gingivectomy include all of the following EXCEPT one. Which one is the
EXCEPTION?

A. the need for bone surgery or examination of the bone shape and morphology

B. situations in which the bottom of the pocket is apical to the mucogingival junction

C. esthetic considerations, particularly in the anterior maxilla

D. elimination of gingival enlargements

Answer: elimination of gingival enlargements


Q7: The primary objective and advantage of surgical flap procedures in the treatment of periodontal
disease is:

A. reduce or eliminate periodontal pockets

B. to provide access to root surfaces for debridement

C. regrowth of alveolar bone

D. maintenance of biological width

E. establishment of adequate soft tissue contours

Answer: to provide access to root surfaces for debridement – all the other choices are goals but
not a primary objective

Q8: The modified Widman flap (MWF) facilitates instrumentation and attempts to reduce pocket depth.

The apically displaced flap improves accessibility and eliminates the pocket, but it does the latter by
apically positioning the soft tissue wall of the pocket.

A. both statements are true

B. both statements are false

C. the first statement is true, the second is false

D. the first statement is false, the second is true

Answer: the first statement is false, the second is true

Q9: A soft tissue graft that is rotated or otherwise repositioned to correct an adjacent defect is called a:

A. free gingival graft

B. pedicle graft

C. connective tissue graft

D. frenectomy

Answer: pedicle graft

Q10: What is guided tissue regeneration?

A. a soft tissue graft used to correct mucogingival junction involvement

B. placement of nonresorbable barriers or resorbable membranes and barriers over a bony defect

C. a free gingival graft used to increase the amount of attached gingiva

D. placement of an autograft to treat a bony defect

Answer: placement of nonresorbable barriers or resorbable membranes and barriers over a bony
defect
Q11: The primary reason for the failure of a free gingival autograft is:

A. infection

B. edema

C. disruption of the vascular supply before engraftment

D. the formation of scar tissue

Answer: disruption of the vascular supply before engraftment

Q12: A variation of the laterally positioned flap is called:

A. a coronally positioned flap

B. a modified Widman flap

C. a double papilla flap

D. a free gingival flap

Answer: a double papilla flap

Q13: Which of the following statements is NOT true regarding the conventional flap design?

A. the entire papilla is incorporated in one of the flaps

B. used in cases where the flap needs to be displaced

C. used in instances of too narrow interdental spaces

D. none of the above

Answer: the entire papilla is incorporated in one of the flaps

Q14: Which of the following has a relatively high degree of predictability and is a "workhorse" of
periodontal therapy?

A. coronally displaced (positioned) flap

B. laterally displaced (positioned) flap

C. double papilla flap

D. apically displaced (positioned) flap

Answer: apically displaced (positioned) flap


Q15: A free mucosal autograft (subepithelial connective tissue graft) differs from a free gingival graft in that
the transplant in a free mucosal graft is:

A. connective tissue with an epithelial covering

B. connective tissue without an epithelial covering

C. epithelial tissue with its own blood supply

D. epithelial tissue without its own blood supply

Answer: connective tissue without an epithelial covering

Q16: Which of the following mucogingival surgical techniques is indicated in areas of narrow gingival
recession adjacent to a wide band of attached gingiva that can be used as a donor site?

A. free gingival autograft

B. double papilla flap

C. modified Widman flap

D. laterally displaced (positioned) flap

Answer: laterally displaced (positioned) flap

Q17: Positive and negative architecture refer to the relative position of interdental bone to radicular bone.
The architecture is said to be "positive" if the radicular bone (facial and lingual) is ______ to the interdental
bone.

A. apical

B. coronal

C. the same height

D. none of the above

Answer: apical

Q18: The most common osseous defects are:

A. one-wall intrabony defects

B. two-wall intrabony defects (osseous craters)

C. three-wall intrabony defects

D. through-and-through furcation defects

Answer: two-wall intrabony defects (osseous craters)


Q19: All of the following microorganisms are the primary colonizers in dental plaque EXCEPT one. Which
one is the EXCEPTION?

A. Streptococcus mitis

B. Actinomyces odontolyticus

C. Treponema denticola

D. Capnocytophaga gingivalis

Answer: Treponema denticola

Q20: Smokers demonstrate more orange and red microbial complexes. Smokers have an increase
in Tannerella forsythia levels.

A. both statements are true

B. both statements are false

C. the first statement is true, the second is false

D. the first statement is false, the second is true

Answer: both statements are true

Q21: Mediators produced as a part of the host response that contribute to tissue destruction include all of
the following EXCEPT one. Which one is the EXCEPTION?

A. Free radicals

B. Proteinases

C. Prostaglandins

D. Cytokines

Answer: Free radicals

Q22: Inflammation, bleeding on probing, and pocket depths are the most important diagnostic aids or signs
of gingival or periodontal disease.

The presence or absence of stippling is not diagnostic.

A. Both statements are true

B. Both statements are false

C. The first statement is true, the second is false

D. The first statement is false, the second is true

Answer: Both statements are true


Q23: In a clinically healthy periodontium, the microbial flora is largely composed of:

A. gram-negative obligate microorganisms

B. gram-negative facultative microorganisms

C. gram-positive obligate microorganisms

D. gram-positive facultative microorganisms

Answer: gram-positive facultative microorganisms

Q24: Endotoxins are the lipopolysaccharide component of the cell wall of:

A. gram-positive bacteria

B. gram-negative bacteria

C. both gram-positive and gram-negative bacteria

D. neither gram-positive and gram-negative bacteria

Answer: gram-negative bacteria

Q25: In a healthy sulcus, which of the bacteria below are most abundant?

A. Actinobacillus actinomycetemcomitans and Tannerella forsythus

B. Streptococcus and Actinomyces species

C. Treponema and Capnocytophaga species

D. Prevotella intermedia and Porphyromonas gingivalis

Answer: Streptococcus and Actinomyces species

Q26: Which of the following forms of periodontitis has been excluded from the classification of periodontal
disease at the 2017 World Workshop on the Classification of Periodontal and Peri-Implant Diseases and
Conditions?

A. necrotizing periodontal disease

B. periodontitis as a direct manifestation of systemic diseases

C. localized aggressive periodontitis

D. periodontitis

Answer: localized aggressive periodontitis


Q27: Diseases that present clinically as desquamative gingivitis include all of the following EXCEPT one.
Which one is the EXCEPTION?

A. lichen planus

B. pemphigoid

C. Pemphigus vulgaris

D. leukemia

E. chronic ulcerative stomatitis

F. lupus erythematosus

Answer: leukemia

Q28: The lateral periodontal cyst is most commonly located in the midline between the maxillary central
incisors.

The lateral periodontal cyst is not the result of an inflammatory reaction.

A. both statements are true

B. both statements are false

C. the first statement is true, the second is false

D. the first statement is false, the second is true

Answer: the first statement is false, the second is true

Q29: All of the following are the characteristic features of periodontitis EXCEPT one. Which one is
the EXCEPTION?
alveolar bone loss

A. periodontal pocket

B. clinical attachment loss

C. hypertrophy of gingiva

Answer: hypertrophy of gingiva

Q30: Which of the following is most significant in regard to the prognosis of a periodontally involved tooth?

A. pocket depth

B. attachment loss

C. anatomical crown length

D. bleeding on probing

Answer: attachment loss


Q31: Gingivitis is most often caused by:

A. a hormonal imbalance

B. inadequate oral hygiene

C. occlusal trauma

D. a vitamin deficiency

E. aging

Answer: inadequate oral hygiene – this leads to the formation of bacterial plaque
and its products, which are the primary etiologic factors in gingivitis

Q32: _________ needs to be evident in making a diagnosis of periodontitis?

A. bleeding

B. pocket depths of 5 mm or more

C. radiographic evidence of bone loss

D. a change in tissue color and tone

Answer: radiographic evidence of bone loss

Q33: The degree of gingival enlargement can be scored as follows: Grade 0, Grade I, Grade II, and Grade
III. Enlargement that involves the papilla and marginal gingiva would be scored as:

A. grade 0

B. grade I

C. grade II

D. grade III

Answer: grade II

Q34: The most important plaque retentive factor is:

A. subgingival and/or overhanging margins of restorations

B. carious lesions that extend subgingivally

C. calculus

D. furcations exposed by loss of attachment and bone

E. crowded and malaligned teeth

F. root grooves and concavities

Answer: calculus
Q35: During pregnancy there is an increase in levels of both progesterone and estrogen.

The so-called pregnancy tumor should be removed as it is a true neoplasm.

A. both statements are true

B. both statements are false

C. the first statement is true, the second is false

D. the first statement is false, the second is true

Answer: the first statement is true, the second is false

Q36: Plasma cells are the predominant immune cells in which stage of gingivitis?

A. I

B. II

C. III

Answer: III

Q37: Medical conditions in which the influences of periodontal infection are documented include all of the
following EXCEPT one. Which one is the EXCEPTION?

A. coronary heart disease

B. atherosclerosis

C. stroke

D. cirrhosis of the liver

E. diabetes mellitus

F. low-birth-weight delivery

Answer: cirrhosis of the liver

Q38: The frequency of maintenance visits for a patient who has had previous periodontal treatment should
depend on which two factors?

A. on whether or not the patient feels that frequent visits will help maintain his/her periodontium

B. on the appearance and clinical condition of the gingival tissues

C. on the amount of attachment loss prior to the periodontal treatment

D. on the patient's ability to perform the proper home-care

Answer: on the appearance and clinical condition of the gingival tissues – this will determine if the
patient is maintaining adequate plaque control, on the patient's ability to perform the proper home-
care – this will determine the effectiveness of the patient's oral hygiene
Q39: If one of the steps in the fabrication of your implant restoration is attaching an analogue to the
impression post and then seating them both into the set impression material, you have made a/an:

A. open-tray impression

B. closed-tray impression

C. either of the above

D. neither of the above

Answer: closed-tray impression

Q40: An "implant level impression" means that:

A. the impression coping (or impression post) was attached to the implant

B. the impression coping was attached to the abutment

C. the impression captured or recorded the actual abutment attached to the implant

D. none of the above

Answer: the impression coping (or impression post) was attached to the implant

Q41: Which of the following is NOT an advantage of retaining an implant-supported crown to an abutment
with a screw (rather than with cement)?

A. the crown is retrievable

B. the residual cement excess in the sulcus is avoided

C. the occlusal surface of the crown is left intact

D. a shorter abutment may be used

Answer: the occlusal surface of the crown is left intact

Q42: The purpose of the second-stage surgery in the creation of an implant restoration is to:

A. check on healing progress

B. remove the implant for cleaning

C. uncover the implant and place an abutment

D. determine which components will fit

E. all of the above

Answer: uncover the implant and place an abutment


Q43: In implantology, "countersinking" refers to the process of:

A. flaring or enlarging the coronal end of the osteotomy

B. reversing the engine to remove the implant

C. placing the implant in a counterclockwise rotation

D. torquing the abutment to place

Answer: flaring or enlarging the coronal end of the osteotomy

Q44: The "macro design" of implants describes:

A. screw or cylinder shape

B. surface texture

C. alloy composition

D. surface coatings

E. none of the above

Answer: screw or cylinder shape

Q45: The soft tissue interface between the oral tissue and titanium can be? Select all that apply.

A. established by keratinized mucosa

B. established by nonkeratinized mucosa

C. dependent on hemidesmosomes

D. comparable with the interface on teeth

Answer: • established by keratinized mucosa


• established by nonkeratinized mucosa
• dependent on hemidesmosomes
• comparable with the interface on teeth

Q46: The advantage of using a screw-shaped implant instead of a straight cylinder implant without threads
is:

A. easier osteotomy preparation

B. increased initial stability

C. less force required to insert the implant

D. no antirotational components are necessary

Answer: increased initial stability


Q47: When considering probing depths around implants and the associated attachment level, what
landmark is used to determine the "clinical" attachment level?

A. CEJ

B. apical end of the implant

C. ball tip of the probe

D. implant shoulder or other permanent physical structure

E. radiograph

Answer: implant shoulder or other permanent physical structure

Q48: If the implant-to-abutment interface consists of a permanent extension on the top of an implant and a
receptacle inside the abutment, the implant connection is categorized as:

A. an internal connection

B. an external connection

C. a cone in a socket connection

D. a nonengaging connection

E. a rotational element

Answer: an external connection

Q49: Which of the following occlusal conditions would exert the lowest amount of biting force under normal
conditions?

A. complete dentures resting on soft tissue

B. complete dentures supported by implants

C. fully dentate occlusion

D. maxillary complete denture opposing natural teeth

Answer: complete dentures resting on soft tissue

Q50: Osseointegration can fail due to:

A. occlusal overload

B. bacterial plaque

C. micromotion during healing

D. excessive cantilevering of the prosthesis

E. failure to achieve primary stability of the implant

F. all of the above


Answer: all of the above
Q51: Which of the following is/are necessary for osseointegration to be successful?

A. primary stability of the implant

B. limited micromotion during healing

C. minimization of surgical trauma

D. uncontaminated implant surfaces

E. all of the above

Answer: all of the above

Q52: "Site development" for implants includes the following techniques. Select all that apply.

A. guided tissue regeneration

B. socket grafting

C. bone grafting with cow bone

D. bisphosphonate supplementation for bone density

E. orthodontic tooth movement

Answer: • guided tissue regeneration


• socket grafting
• bone grafting with cow bone
• orthodontic tooth movement

Q53: "Antirotational" element is added to an implant to:

A. prevent rotation of multiunit restorations

B. prevent rotation of the implant

C. prevent rotation of the abutment in the implant

D. prevent overtorquing of the abutment screw

E. all of the above

Answer: prevent rotation of the abutment in the implant


Q54: A radiograph should be exposed and interpreted at which of the following points during implant
placement and restoration:

A. at the end of the surgical procedure

B. following the final torque application to an abutment

C. anytime the fit or seating of a component must be verified

D. intraoperatively to verify implant position

E. all of the above

Answer: all of the above

Q55: The hole that is surgically created in the bone to receive the implant body is called the:

A. osteotomy

B. bone channel

C. smokestack

D. callus core

E. chimney

Answer: osteotomy

Q56: Which of the following was/were included in the 1986 Albrektsson et al criteria for functional implant
success?

A. implant is immobile and has no periimplant radiolucency

B. bone loss is less than 0.02 mm/year after the first year of service

C. absence of persistent and/or irreversible pain, infection, paresthesia or violation of the mandibular
canal

D. success rates of at least 85% at year 5 and 80% at year 10

E. all of the above

Answer: all of the above


Q57: The most common source of patient dissatisfaction with implants is:

A. pain

B. appearance

C. mobility

D. loss of osseointegration

Answer: appearance

Q58: "Direct structural and functional connection between ordered, living bone and the surface of a load-
carrying implant" is the definition of:

A. a cold weld

B. ankylosis

C. osseointegration

D. metal callous formation

Answer: osseointegration

Q59: Implants should not be treatment planned in young individuals until skeletal growth has been
completed.

A. true

B. false

Answer: true

Q60: The highest rate of implant failure occurs in

A. type 1 bone

B. type 2 bone

C. type 3 bone

D. type 4 bone

Answer: type 4 bone


Q61: In patients with normal bone and normal healing capabilities, one should anticipate dental implant
success rates of:

A. 35%

B. 50% to 60%

C. 74%

D. 90% to 95%

E. 100%

Answer: 90% to 95%

Q62: The greatest esthetic challenge for the restorative dentist occurs in the patient having a:

A. high smile line, thick periodontal biotype

B. low smile line, thin periodontal biotype

C. low smile line, thick periodontal biotype

D. high smile line, thin periodontal biotype

Answer: high smile line, thin periodontal biotype

Q63: Implants are useful for orthodontic anchorage because implants:

A. do not move in response to orthodontic forces

B. do not have a periodontal ligament

C. can be placed and later removed

D. can be located to gain force advantage

E. all of the above

Answer: all of the above

Q64: Assessment of the potential implant patient should include:

A. direct palpation of the bony ridges

B. determination of any limitations of mouth opening

C. cross-sectional imaging

D. all of the above

Answer: all of the above


Q65: The "anterior loop" is descriptive of the:

A. circular course of the internal maxillary artery

B. maxillary sinus extension toward the canine tooth

C. course of the inferior alveolar nerve anterior to the mental foramen

D. necessity for angled abutments

E. none of the above

Answer: course of the inferior alveolar nerve anterior to the mental foramen

Q66: At the conclusion of surgical placement of a titanium implant, complete soft tissue coverage of the
implant is required for successful osseointegration to occur.

A. true

B. false

Answer: false

Q67: Where should the healing abutment be placed at the stage II (uncovering) surgery for a two-piece
implant system?

A. approximately 1 to 2 mm "taller" than the height of the tissue

B. out of occlusion

C. radiographically confirmed as seated

D. designed to shape or mold the tissue

E. all of the above

Answer: all of the above

Q68: Cross-sectional imaging is produced by all of the following EXCEPT one. Which one is the
EXCEPTION?

A. cone beam computed tomography

B. panoramic radiography

C. conventional linear tomography

D. computed tomography scans

Answer: panoramic radiography


Q69: The clinical examination of osseointegrated dental implants should include:

A. soft tissue observation

B. radiographic assessment

C. probing

D. visual inspection of implant components

E. all of the above

Answer: all of the above

Q70: Which of the following dimensions is not part of routinely recommended space requirements for root
form implants?

A. a minimum of 3 mm of space between adjacent implants

B. at least 2 mm of bone remaining between the implant and vital structures

C. at least 1 mm of bone surrounding the implant in all dimensions

D. a minimum of 16 mm of implant length for stability

E. placement of the restorative platform apical to the CEJ's of adjacent teeth

Answer: a minimum of 16 mm of implant length for stability

Q71: Of the following factors, which determine(s) the abutment that the restorative dentist should select?

A. soft tissue (mucosal or gingival) height

B. interarch space

C. implant angulation

D. esthetic requirements

E. all of the above

Answer: all of the above

Q72: A "Morse taper" is one of many designs for:

A. the abutment to implant connection

B. the shape of the abutment's restorative part

C. press-fit implants

D. threads on a screw implant

E. an implant milling machine

Answer: the abutment to implant connection


Q73: A "polished collar," or the smooth polished exterior surface of the implant closest to or in the oral
cavity, is designed to:

A. provide a location for epithelial attachment

B. make the exposed surface cleansable

C. keep plaque from gaining access to deeper structures

D. inhibit biofilm formation

E. all of the above

Answer: all of the above

Q74: If the space available for the abutment and the crown is limited vertically, i.e., a small interarch
space, which of the following abutment combinations would be least advantageous:

A. cemented crown to the abutment

B. screw-retained crown on the abutment

C. restoration that fits to the implant with no abutment

D. abutment with a ball extension for a removable prosthesis

Answer: cemented crown to the abutment

Q75: Which of the following is NOT acceptable for cleaning of titanium surfaces, either by the patient or the
dental clinician?

A. powered toothbrushes

B. end-tufted brushes

C. plastic curettes

D. conventional ultrasonic tips

E. floss, especially multifilament varieties

Answer: conventional ultrasonic tips

Q76: Which of the following conditions is NOT a contraindication to dental implant placement?

A. current chemotherapy for the treatment of cancer

B. history of radiation therapy to the maxillofacial complex

C. dysmorphophobia

D. advanced patient age

Answer: advanced patient age


Q77: "Platform switching" refers to:

A. replacing the implant to get a better abutment selection

B. changing to a wider abutment for molar contours

C. changing to a shorter abutment to hide margins

D. matching a smaller diameter abutment with a wider diameter implant

E. converting a round abutment to triangular

Answer: matching a smaller diameter abutment with a wider diameter implant

Q78: If two adjacent implants are surgically placed too close together:

A. the patient will have difficulty cleaning the area

B. bone loss will probably occur between the implants

C. the presence of a normal papilla is unlikely

D. optimum esthetic contours will be prevented

E. all of the above

Answer: all of the above

Q79: Which cells control all three stages of inflammation?

A. plasma cells

B. red blood cells

C. leukocytes

D. Sertoli cells

Answer: leukocytes

Q80: Initially, the first group of cells to arrive at the site of injury are neutrophils. Later, _________ become
more numerous. In certain parasitic infections, ________ predominate. In viral infection, ________ rather
than neutrophils usually predominate.

A. plasma cells, basophils, eosinophils

B. macrophages, eosinophils, lymphocytes

C. macrophages, lymphocytes, basophils

D. lymphocytes, macrophages, eosinophils

Answer: macrophages, eosinophils, lymphocytes


Q81: Which cells of the immune system possess receptors for the complement component (C3a and C5a)
as well as receptors for the Fc portion of the antibody molecules IgE and IgG?

A. peripheral dendritic cells

B. dermal dendrocytes (histiocytes)

C. mast cells

D. natural killer (NK) cells

Answer: mast cells

Q82: The main cells involved in chronic infection are lymphocytes and:

A. plasma cells

B. mast cells

C. neutrophils

D. macrophages

Answer: macrophages

Q83: The _________ has emerged as a unique immune cell that could be activated by many nonimmune
processes, including acute stress, and could participate in a variety of inflammatory diseases in the
nervous system, skin, joints, as well as cardiopulmonary, intestinal, and urinary systems.

A. neutrophil

B. epithelioid cell

C. mast cell

D. eosinophil

Answer: mast cell

Q84: The purposes of periodontal dressings (packs) include all of the following EXCEPT one. Which one is
the EXCEPTION?

A. stop persistent bleeding

B. maintain the sutured position of the flaps

C. improve patient comfort

D. prevent mechanical injury to healing tissues

Answer: stop persistent bleeding (all bleeding should be controlled


before the periodontal dressing is placed)
Q85: Which one of the following will increase the abrasive action of a polishing agent?
Select all that apply.

A. using an agent with nice large particles

B. using an agent containing particles that are dull and round

C. using a thick, viscous mixture of polishing paste

D. applying the polishing agent with firm pressure and increasing to a heavy constant pressure

E. polishing at a low speed with light pressure

Answer: • using an agent with nice large particles


• using a thick, viscous mixture of polishing paste
• applying the polishing agent with firm pressure
and increasing to a heavy constant pressure

Q86: Gingival fibers consist of:

A. type I collagen

B. type II collagen

C. type III collagen

D. type IV collagen

Answer: type I collagen

Q87: Microorganisms that colonize the periodontal abscess have been reported to be primarily:

A. gram-positive aerobic cocci

B. gram-negative aerobic rods

C. gram-negative anaerobic rods

D. gram-positive anaerobic cocci

Answer: gram-negative anaerobic rods


Q88: All of the following are clinical signs of bruxism EXCEPT one. Which one is the EXCEPTION?

A. TMJ symptoms

B. muscle soreness

C. periodontal pocket formation

D. cracked teeth or fillings

E. wear facets on teeth

F. widened PDL spaces on radiographs

Answer: periodontal pocket formation – periodontal pockets are inflammatory


lesions that are not caused by occlusal trauma or bruxism

Q89: All of the following are contraindications to selective grinding in the natural dentition EXCEPT one.
Which one is the EXCEPTION?

A. when pulp chambers are small

B. in the presence of tooth sensitivity

C. when major occlusal discrepancies may require orthodontics or full mouth reconstruction

D. in patients who are poor candidates for full mouth reconstruction because of psychologic factors

Answer: when pulp chambers are small

Q90: Which of the following is not correctly matched with regards to a periodontal treatment plan?

A. preliminary phase: plaque control

B. phase I: mouth preparation

C. phase II: periodontal surgery

D. phase III: restorative

E. phase IV: maintenance

Answer: preliminary phase: plaque control

Q91: Which of the following statements regarding the assessment of tooth mobility is false?

A. teeth normally have a very slight physiologic mobility

B. mobility can result from multiple causes

C. mobility is unacceptable and should be treated

D. periodontal disease can result in tooth mobility

Answer: mobility is unacceptable and should be treated


Q92: All of the following statements concerning T cells are true EXCEPT one. Which one is the
EXCEPTION?

A. antigens are recognized by T cells in association with either MHC class I or class II molecules on the
surface of the antigen-presenting cell

B. they recognize diverse antigens using a low-affinity transmembrane complex, the T-cell antigen
receptor (TCR)

C. they are subdivided based on whether they possess the co-receptors CD4 or CD8

D. they are important in humoral (antibody-mediated) immunity

Answer: they are important in humoral (antibody-mediated) immunity

Q93: The major proportion of the organic matrix of cementum is composed of:

A. Type I and Type II collagens

B. Type III and Type IV collagens

C. Type I and Type III collagens

D. Type II and Type IV collagens

Answer: Type I and Type III collagens

Q94: All of the following statements concerning cementum are true EXCEPT one. Which one is the
EXCEPTION?

A. it is produced by cells of the periodontal ligament

B. the main function is to compensate for tooth wear

C. the deposition of new cementum continues periodically throughout life whereby root fractures may be
repaired

D. the cementum is indistinguishable on radiographs

E. it is lighter in color than dentin, contains 45% to 50% inorganic substance, and its permeability
diminishes with age

Answer: the main function is to compensate for tooth wear


Q95: When occlusal forces increase in magnitude, the periodontium responds with the following changes
EXCEPT one. Which one is the EXCEPTION?

A. widening of the periodontal ligament space

B. an increase in the number of periodontal ligament fibers

C. an increase in the density of alveolar bone

D. a decrease in the width of periodontal ligament fibers

Answer: a decrease in the width of periodontal ligament fibers

Q96: In combined endodontic-periodontic lesions, it is generally wise to treat:

A. the periodontic component first

B. the endodontic component first

C. both components at the same time

D. them any way you want to

Answer: the endodontic component first

Q97: The most common indication to splint mobile teeth is to:

A. improve patient comfort and to provide better control of the occlusion if the anterior teeth are mobile

B. improve oral hygiene

C. prevent a natural unopposed tooth from migrating

D. prevent maxillary central incisors from separating after closure of diastema

Answer: improve patient comfort and to provide better control of the occlusion if the anterior teeth
are mobile

Q98: Extrinsic dental stains include:

A. green to brown stains caused by erythroblastosis fetalis

B. brown, black, green, or orange stains caused by chromogenic bacteria in plaque

C. red to brown stains caused by congenital porphyria

D. gray or brownish stains caused by tetracycline

Answer: brown, black, green, or orange stains caused by chromogenic bacteria in plaque
Q99: Trauma from occlusion can produce radiographically detectable changes in all of the following
EXCEPT one. Which one is the EXCEPTION?

A. lamina dura

B. periodontal pockets

C. width of the PDL space

D. morphology of the alveolar crest

E. density of the surrounding cancellous bone

Answer: periodontal pockets

Q100: Water irrigation devices (oral irrigators) have been shown to:

A. eliminate plaque

B. clean non-adherent bacteria and debris from the oral cavity more effectively than toothbrushes and
mouth rinses

C. disinfect pockets for up to 12 hours

D. prevent calculus formation

Answer: clean non-adherent bacteria and debris from the oral cavity more effectively than
toothbrushes and mouth rinses

Q101: Abrasives:

A. compose 5% to 10% of dentifrices

B. compose 10% to 20% of dentifrices

C. compose 20% to 40% of dentifrices

D. compose 50% to 65% of dentifrices

Answer: compose 20% to 40% of dentifrices

Q102: Which component of Super Floss® is most effective in cleaning around appliances and between
wide spaces?

A. stiffened end

B. spongy floss

C. regular floss

Answer: spongy floss


Q103: The effectiveness of toothbrushing is best measured by:

A. the amount and location of plaque

B. the caries experience

C. the toothbrushing frequency

D. the condition of the toothbrush

Answer: the amount and location of plaque

Q104: To date, the ADA has accepted two agents for the treatment of gingivitis, these are:

A. prescription solutions of chlorhexidine digluconate oral rinse

B. prescription solutions of tetracycline oral rinse

C. nonprescription essential oil oral rinse

D. nonprescription solutions of penicillin oral rinse

Answer: • prescription solutions of chlorhexidine digluconate oral rinse


• nonprescription essential oil oral rinse

Q105: Which of the following oral hygiene aids are of primary importance in gingival massage?

A. Stim-U-Dent®

B. interproximal brushes

C. Perio-Aid® (aka "tooth pick")

D. oral irrigator

Answer: Stim-U-Dent®

Q106: Which of the following is the only local delivery system of antibiotics accepted by the ADA and is
available in the United States and a number of other countries?

A. 10% doxycycline gel

B. 2% minocycline microspheres

C. 25% metronidazole gel

D. Chlorhexidine® (2.5 mg) in gelatin matrix

Answer: 10% doxycycline gel


Q107: The attachment apparatus is composed of all of the following EXCEPT one. Which one is the
EXCEPTION?

A. periodontal ligament

B. cementum

C. alveolar bone

D. gingiva

Answer: gingiva

Q108: Of the choices listed below, which one describes the boundaries that define the attached gingiva?

A. from the gingival margin to the interdental groove

B. from the free gingival groove to the gingival margin

C. from the mucogingival junction to the free gingival groove

D. from the epithelial attachment to the cementoenamel junction

Answer: from the mucogingival junction to the free gingival groove (base of the sulcus)

Q109: The gingival fibers consist of type I collagen.

The gingival fibers are mainly arranged in two groups.

both statements are true

A. both statements are false

B. the first statement is true, the second is false

C. the first statement is false, the second is true

Answer: the first statement is true, the second is false

Q110: Because of the high turnover rate, the connective tissue of the gingiva has a remarkably good
healing and regenerative capacity.

The reparative capacity of the gingival connective tissues is better than that of the periodontal ligament or
the epithelial tissue.

A. both statements are true

B. both statements are false

C. the first statement is true, the second is false

D. the first statement is false, the second is true

Answer: the first statement is true, the second is false


Q111: The principal fibers of the periodontal ligament are arranged in six groups.

The molecular configuration of collagen fibers in the periodontal ligament provides them with a tensile
strength greater than that of steel.

A. both statements are true

B. both statements are false

C. the first statement is true, the second is false

D. the first statement is false, the second is true

Answer: both statements are true

Q112: The length of the junctional epithelium ranges from:

A. 0.1 to 0.75 mm

B. 0.25 to 1.35 mm

C. 0.5 to 2.0 mm

D. 2.0 to 2.5 mm

Answer: 0.25 to 1.35 mm

Q113: Cervical line contours are closely related to the attachment of the gingiva at the neck of the tooth.
The greatest contour of the cervical lines and gingival attachments occur on:

A. the distal surface of anterior teeth

B. the distal surface of posterior teeth

C. the mesial surface of anterior teeth

D. the mesial surface of posterior teeth

Answer: the mesial surface of anterior teeth

Q114: The narrowest band of attached gingiva is found:

A. on the lingual surfaces of maxillary incisors and the facial surfaces of maxillary first molars

B. on the facial surfaces of mandibular second premolars and the lingual surface of canines

C. on the facial surfaces of the mandibular canine and first premolar and the lingual surfaces adjacent to
the mandibular incisors and canines

D. none of the above

Answer: on the facial surfaces of the mandibular canine and first premolar and the lingual surfaces
adjacent to the mandibular incisors and canines
Q115: Which of the following types of oral mucosa is not keratinized under normal conditions?

A. buccal mucosa

B. vermillion border of the lips

C. hard palate

D. gingiva

Answer: buccal mucosa

Q116: Bone consists of:

A. two-thirds organic matter and one-third inorganic matrix

B. one-third organic matter and two-thirds inorganic matrix

C. one-half organic matter and one-half inorganic matrix

D. two-thirds inorganic matter and one-third organic matrix

Answer: two-thirds inorganic matter and one-third organic matrix

Q117: ________ are the most common cells in the periodontal ligament and appear as ovoid or elongated
cells oriented along the principal fibers, exhibiting pseudopodia-like processes.

A. cementoblasts

B. osteoblasts

C. fibroblasts

D. macrophages

Answer: fibroblasts

Q118: Which of the following is a source of blood supply to the gingiva? Select all that apply.

A. vessels of the periodontal ligament

B. supraperiosteal arterioles

C. arterioles from the crest of the interdental septa

Answer: • vessels of the periodontal ligament

•supraperiosteal arterioles

•arterioles from the crest of the interdental septa


Q119: Although the average width of the periodontal ligament space is documented to be about
___________, considerable variation exists.

A. 0.002 mm

B. 0.2 mm

C. 2.0 mm

D. 20 mm

Answer: 0.2 mm

Q120: The sulcular epithelium is a:

A. thick, keratinized stratified squamous epithelium without rete pegs

B. thick, nonkeratinized stratified squamous epithelium with rete pegs

C. thin, keratinized nonstratified squamous epithelium with rete pegs

D. thin, nonkeratinized stratified squamous epithelium without rete pegs

Answer: thin, nonkeratinized stratified squamous epithelium without rete pegs

Q121: A/An ________ of the saliva causes precipitation of calcium phosphate salts by lowering the
precipitation constants.

A. decrease in the pH

B. increase in the pH

C. decrease in the viscosity

D. increase in the viscosity

Answer: increase in the pH

Q122: The source of mineralization for supragingival calculus is:

A. desquamated epithelial cells

B. gingival crevicular fluid

C. phosphatases formed by bacterial plaque

D. saliva

Answer: saliva
Q123: Which of the following forms the major component of dental plaque?

A. materia alba

B. microorganisms

C. desquamated epithelial cells

D. minerals

Answer: microorganisms

Q124: Nearly all human oral bacteria exhibit __________, cell-to-cell recognition of genetically distinct cell
types.

A. adhesion

B. pleomorphism

C. coaggregation

D. organization

Answer: coaggregation

Q125: Specific bacteria are implicated in periodontal disease and are commonly found at the site of
infection. The Orange complex of bacteria consists of all of the following EXCEPT one. Which one is the
EXCEPTION?

A. fusobacterium

B. prevotella

C. Porphyromonas gingivalis

D. campylobacter species

Answer: Porphyromonas gingivalis

Q126: As plaque ages:

A. the number of cocci increases and the number of rods, fusiform, filaments, and spirochetes decreases

B. the number of aerobic bacteria increases and the number of anaerobic bacteria decreases

C. the number of gram-positive organisms decreases and the number of gram-negative organisms
increases

D. none of the above

Answer: the number of gram-positive organisms decreases and the number of gram-
negative organisms increases
Q127: All of the following constitute materia alba EXCEPT one. Which one is the EXCEPTION?

A. bacteria

B. desquamated epithelial cells

C. leukocytes

D. minerals and their ions

Answer: minerals and their ions

Q128: Angular defects are classified on the basis of:

A. the number of osseous walls that were destroyed by periodontal disease

B. the number of osseous walls left surrounding the tooth

C. the number of osseous walls that will remain after surgery

D. periodontal probe readings

Answer: the number of osseous walls left surrounding the tooth

Q129: When evaluating an osseous defect, the only way to determine the number of walls left surrounding
the tooth is by:

A. periodontal probing

B. radiographs

C. exploratory surgery

D. testing for mobility

Answer: exploratory surgery

Q130: Which of the following is the most reliable method for determining the location and extent of
periodontal pockets?

A. probing the gingival margin

B. use of a flowable radiopaque material

C. use of scalers and curettes

D. intraoral radiography

Answer: probing the gingival margin


Q131: The principal differences between intrabony and suprabony pockets are the relationship of the soft
tissue wall of the pocket to the alveolar bone, the pattern of bone destruction, and the direction of the
transseptal fibers of the periodontal ligament.

In intrabony pockets, the base of the pocket is coronal to the crest of the alveolar bone, and the pattern of
destruction of the underlying bone is horizontal.

A. both statements are true

B. both statements are false

C. the first statement is true, the second is false

D. the first statement is false, the second is true

Answer: the first statement is true, the second is false

Q132: Match the types of pockets on the left with their correct descriptions on the right.

• 1. Gingival pocket
• 2. Suprabony pocket
• 3. Infrabony pocket
• A. Bone loss is vertical
• B. No destruction of the supporting periodontal tissues
• C. Bone loss is horizontal

Answer: 1. B, 2. C, 3. A

Q133: Furcation involvements have been classified as grades I, II, III, and IV according to the amount of
tissue destruction. Grade III is:

A. incipient bone loss

B. partial bone loss (cul-de-sac)

C. total bone loss with through-and-through opening of the furcation

D. similar to the above, but with gingival recession exposing the furcation to view

Answer: total bone loss with through-and-through opening of the furcation


Q134: Drug-induced gingival enlargement consists of a pronounced hyperplasia of the connective tissue
and epithelium.

Drug-induced gingival enlargement may occur in mouths with little or no plaque and may be absent in
mouths with abundant deposits.

A. both statements are true

B. both statements are false

C. the first statement is true, the second is false

D. the first statement is false, the second is true

Answer: both statements are true

Q135: When using the periodontal probe to measure pocket depth, the measurement is taken from the:

A. base of the pocket to the CEJ

B. free gingival margin to the CEJ

C. junctional epithelium to the margin of the free gingiva

D. base of the pocket to the mucogingival junction

Answer: junctional epithelium to the margin of the free gingiva

Q136: How should a periodontal probe be adapted in an interproximal area?

A. it should be parallel to the long axis of the tooth at the point angle

B. it should be parallel to the long axis of the tooth at the contact area

C. it should touch the contact area and the tip should angle slightly beneath and beyond the contact area

D. it should be perpendicular to the long axis of the tooth in front of the contact area

Answer: it should touch the contact area and the tip should angle
slightly beneath and beyond the contact area

Q137: How should the periodontal probe be inserted into the sulcus?

A. perpendicular to the long axis of the tooth

B. with a firm pushing motion

C. with a short oblique stroke

D. parallel to the tooth surface

Answer: parallel to the tooth surface


Q138: If you should meet resistance after inserting the periodontal probe into the sulcus, you should:

A. remove the probe and reinsert it in a different spot

B. lift the probe away from the tooth and attempt to move it apically

C. force the probe beyond the obstruction

D. remove the probe and select one with a narrower diameter

E. record the measurement where the probe stopped

Answer: lift the probe away from the tooth and attempt to move it apically

Q139: Which of the following is the most common error when performing periodontal probing?

A. using the wrong type of probe

B. incorrectly reading the periodontal probe

C. excessively angling the probe when inserting it interproximally beyond the long axis of the tooth

D. forgetting to also probe the lingual of every tooth

Answer: excessively angling the probe when inserting it interproximally


beyond the long axis of the tooth

Q140: Toothbrush trauma (abrasion) usually occurs on:

A. centrals and laterals

B. canines and premolars

C. second and third molars

D. first and second molars

Answer: canines and premolars

Q141: The most accepted theory as to the cause of root sensitivity is the:

A. Bayer's theory

B. chemiosmotic theory

C. hydrodynamic theory

D. quantum theory

Answer: hydrodynamic theory


Q142: The most important factor in the control of hypersensitive roots among patients with periodontal
disease after gingival recession has exposed the cervical portions of teeth is:

A. prescription of home fluoride rinses

B. minimal removal of tooth structure during root planing

C. the application of desensitizing agents that contain fluoride

D. thorough daily plaque control

Answer: thorough daily plaque control

Q143: When extensive scaling and root planing must be performed, the best approach would be:

A. a series of appointments set up to scale and root plane a segment or quadrant of teeth at a time
(thoroughly and completely)

B. gross debridement (sub- and supragingival) of the entire mouth, followed by a series of appointments
for fine scaling and polishing

C. perform everything in a single appointment

D. none of the above

Answer: a series of appointments set up to scale and root plane a segment or quadrant of teeth at
a time (thoroughly and completely)

Q144: Which of the following presents the most difficulty in performing a thorough scaling and root
planing?

A. mesial surfaces of maxillary premolars

B. proximal surfaces of mandibular incisors

C. trifurcations of maxillary molars

D. distal surfaces of mandibular molars

Answer: trifurcations of maxillary molars

Q145: Some degree of curettage is done unintentionally when scaling and root planing are performed; this
is called inadvertent curettage.

Curettage accomplishes the removal of the chronically inflamed granulation tissue that forms in the lateral
wall of the periodontal pocket.

A. both statements are true

B. both statements are false

C. the first statement is true, the second is false

D. the first statement is false, the second is true


Answer: both statements are true
Q146: The main objective of root planing is:

A. to remove chronically inflamed tissues

B. to change the bacterial microflora

C. to remove etiologic agents from the root surface

D. to eliminate pockets

Answer: to remove etiologic agents from the root surface

Q147: Maximum shrinkage after gingival curettage can be expected from tissue that is:

A. fibrotic

B. edematous

C. fibroedematous

D. formed within an intrabony pocket

Answer: edematous

Q148: Ultrasonic instrumentation is accomplished with a:

A. heavy touch and light pressure, keeping the tip perpendicular to the tooth surface and constantly in
motion

B. light touch and heavy pressure, keeping the tip parallel to the tooth surface and stationary

C. light touch and light pressure, keeping the tip parallel to the tooth surface and constantly in motion

D. heavy touch and heavy pressure, keeping the tip perpendicular to the tooth surface and stationary

Answer: light touch and light pressure, keeping the tip parallel to the tooth surface and constantly
in motion

Q149: The Nabers probe is an instrument used to measure the furcation involvement in multi-rooted teeth.

When detecting an interdental crater the probe should be positioned parallel to the long axis of the tooth.

A. both statements are false

B. the first statement is true, the second is false

C. the first statement is false, the second is true

D. both statements are true

Answer: the first statement is true, the second is false


Q150: In magnetostrictive ultrasonic units, the pattern of vibration of the tip is linear.

In piezoelectric ultrasonic units, the pattern of vibration of the tip is elliptical.

A. both statements are true

B. both statements are false

C. the first statement is true, the second is false

D. the first statement is false, the second is true

Answer: both statements are false

Q151: If a patient experiences sensitivity while being scaled with an ultrasonic scaling device, all of the
following actions will be appropriate to counter this problem EXCEPT two. Which two are inappropriate?

A. proceeding to another tooth and then returning to the sensitive tooth later in the appointment

B. moving the instrument slower

C. making necessary adjustments to the water spray

D. turning up the power of the device

E. using less pressure

Answer: • moving the instrument slower • turning up the power of the device

Q152: Air is used to deflect the free gingival margin to detect:

A. the CEJ

B. smooth root surfaces

C. subgingival calculus

D. inflammation

Answer: subgingival calculus

Q153: The primary function of which instrument is scaling of ledges or rings of calculus?

A. hoe scalers

B. files

C. chisel scalers

D. Quétin furcation curettes

Answer: hoe scalers


Q154: While scaling subgingivally, the tip of the curette breaks off. All of the following are appropriate
actions to take to try and remove this tip EXCEPT one. Which one is the EXCEPTION?

A. use a push stroke to force the tip out of the sulcus

B. gently examine the gingival sulcus

C. take a periapical radiograph of the area

D. place the patient in an upright position

Answer: use a push stroke to force the tip out of the sulcus

Q155: It is impossible to carry out periodontal procedures efficiently with dull instruments.

A sharp instrument cuts more precisely and quickly than a dull instrument.

A. both statements are true

B. both statements are false

C. the first statement is true, the second is false

D. the first statement is false, the second is true

Answer: both statements are true

Q156: When sharpening, a wire edge is produced:

A. only when using a coarse artificial stone

B. when using a mounted ruby stone only

C. when no oil is used for lubrication of the stone

D. when the last stroke of the stone is drawn away from the cutting edge

Answer: when the last stroke of the stone is drawn away from the cutting edge

Q157: A curette designed to scale and root plane anterior teeth with deep pockets will have a:

A. short, straight shank

B. long, straight shank

C. short, angled shank

D. long, angled shank

Answer: long, straight shank


Q158: Which of the following instruments is best suited for root planing?

A. explorers

B. hoe

C. curette

D. ultrasonic instruments

Answer: curette
Prosthodontics

Q1: When border molding a mandibular custom tray that will be used for a final denture impression, the
distolingual extension is limited by the action of the:

A. superior constrictor muscle

B. masseter muscle

C. lateral pterygoid muscle

D. buccinator muscle

Answer: superior constrictor muscle

Q2: The primary reasons for obtaining the most extensive areal coverage for a mandibular complete denture are:

A. to increase the capacity of underlying structures to withstand the stress due to biting force and to improve
appearance

B. to provide balanced occlusion and to increase tongue space

C. to increase the capacity of the underlying structures to withstand the stress due to biting force and to
increase the effectiveness of the seal

D. to improve retention and to increase tongue space

Answer: to increase the capacity of the underlying structures to withstand the stress due to biting force
and to increase the effectiveness of the seal

Q3: A reline is indicated on any denture when the diagnostic information indicates that a reline will effectively
solve the patient's chief complaint—when the denture base adaptation is the major defect in the prosthesis.

A reline is contraindicated when there is excessive overclosure of the vertical dimension—a large decrease in
vertical dimension. In this case, new dentures are indicated at the proper vertical dimension.

A. both statements are true

B. both statements are false

C. the first statement is true, the second is false

D. the first statement is false, the second is true

Answer: both statements are true


Q4: All new dentures should be evaluated:

A. 3 hours after delivery

B. 12 hours after delivery

C. 24 hours after delivery

D. 48 hours after delivery

Answer: 24 hours after delivery

Q5: Inadequate vertical dimension of occlusion may cause:

A. gagging

B. cheek biting

C. reduced taste

D. sore spots in the vestibule

Answer: cheek biting

Q6: All of the following are true regarding the placement of maxillary central incisors in complete dentures
EXCEPT one. Which one is the EXCEPTION?

A. long axes should be perpendicular to occlusal plane

B. arranged directly over the ridge

C. placement controls midline, speaking line and lip support

D. are responsible for helping to make "F" and "V" sounds

Answer: arranged directly over the ridge

Q7: A patient who wears a complete maxillary denture complains of a burning sensation in the palatal area of
his/her mouth. This is indicative of too much pressure being exerted by the denture on the:

A. incisive foramen

B. palatal mucosa

C. hamular notch

D. posterior palatal seal

Answer: incisive foramen


Q8: The treatment plan for a patient indicates that both mandibular and maxillary immediate dentures are to be
fabricated. The ideal way to do this is:

A. fabricate the maxillary immediate denture first

B. fabricate the mandibular immediate denture first

C. fabricate the maxillary and mandibular immediate dentures at the same time

Answer: fabricate the maxillary and mandibular immediate dentures at the same time

Q9: The first step in the treatment of abused tissues in a patient with existing dentures is to:

A. fabricate a new set of dentures

B. reline the dentures

C. educate the patient

D. excise the abused tissues

Answer: educate the patient

Q10: The most important benefit of an overdenture (root-retained denture) is:

A. the psychological comfort of avoiding the loss of all teeth

B. the continuous functional feedback for the neuromuscular system from proprioceptors in the periodontal
membrane

C. the preservation of the alveolar ridge

D. the improved support and stability for the denture

E. the increased retention of the denture

Answer: the preservation of the alveolar ridge

Q11: Which sounds help determine the labiolingual position of the anterior teeth?

A. linguoalveolar sounds or sibilants (such as s, z, sh, and ch)

B. fricatives or labiodental sounds (such as f, v, or ph)

C. the b, p, and m sounds

D. linguodental sounds (such as this, that, or those)

Answer: linguodental sounds (such as this, that, or those)


Q12: Whistling in a speaking denture patient may be caused when:

A. vertical overlap is not enough

B. the area palatal to the incisors is improperly contoured

C. horizontal overlap is too much

D. all of the above

Answer: all of the above

Q13: A patient has worn a complete maxillary denture for 8 years against mandibular anterior teeth (the
remainder of the mandibular teeth are missing). She complains of looseness of the denture. Examination of the
mouth shows an excessive amount of hyperplastic tissue at the anterior part of the maxillary ridge. The maxillary
denture teeth do not show below the upper lip. Radiographs show poor bone structure in the anterior part of the
maxillae. The principal cause of difficulty with her maxillary denture is:

A. fibrous tuberosities

B. too great a vertical dimension of occlusion

C. a lack of posterior occlusion

D. the maxillary denture teeth that were used are too short

Answer: a lack of posterior occlusion

Q14: The primary indicator of the accuracy of border molding is:

A. adequate coverage of tray borders with the material used for border molding

B. contours of the periphery similar to the final form of the denture

C. stability and lack of displacement of the tray in the mouth

D. uniformly thick borders of the periphery

Answer: stability and lack of displacement of the tray in the mouth

Q15: The ultimate support for a maxillary denture is the bone of the two maxillae and the palatine bone.

The palatal rugae are a primary support area as well.

A. both statements are true

B. both statements are false

C. the first statement is true, the second is false

D. the first statement is false, the second is true

Answer: the first statement is true, the second is false


Q16: An overextended distobuccal corner of a mandibular denture will push against which muscle during
function?

A. zygomaticus

B. orbicularis oris

C. temporalis

D. masseter

Answer: masseter

Q17: After border molding the mandibular custom tray, it is important to check for dislodgement in order to detect
areas of:

A. underextension of the tray

B. overextension of the tray

C. thickness of the tray

D. none of the above

Answer: overextension of the tray

Q18: Before an accurate face-bow transfer record can be made on a patient, which of the following must be
determined?

A. the inclination of each condyle

B. vertical dimension of occlusion

C. centric relation

D. location of the hinge axis point

Answer: location of the hinge axis point — axial center of opening-closing

Q19: A plaster index is used to:

A. maintain the vertical dimension of occlusion

B. maintain bite registration

C. preserve the face-bow transfer

D. all of the above

Answer: preserve the face-bow transfer


Q20: A generalized speech difficulty with complete dentures is usually caused by which of the following? Select
all that apply.

A. faulty tooth position

B. excess vertical dimension of occlusion

C. faulty palatal contours

D. faulty occlusion

Answer: • faulty tooth position, faulty palatal contours

Q21: At the first appointment after insertion of complete dentures, the presence of generalized soreness on the
crest of the mandibular ridge is most likely due to:

A. the newness of the denture

B. defective tissue registration

C. premature occlusal contacts

D. incomplete polymerization of the denture base

Answer: premature occlusal contacts

Q22: The inferior surface of the maxillary occlusal rim should be parallel to:

A. Frankfort's plane

B. Camper's line

C. Fox plane

D. the horizontal condylar inclination

Answer: Camper’s line — which is an imaginary line traced from the inferior border of the ala of the nose
to the tragus of the ear

Q23: The most frequent cause of porosities in a denture is:

A. insufficient pressure on the flask during processing

B. insufficient material in the mold

C. a rapid elevation in temperature to 212°F causing vaporization of the liquid

D. insufficient time for processing

Answer: insufficient pressure on the flask during processing


Q24: All of the following are disadvantages to immediate denture therapy. Which one is considered to be the
major disadvantage to immediate denture therapy?

A. increased post-insertion care

B. increased post-insertion soreness

C. not being able to have an anterior tooth try-in to evaluate esthetics

D. greater complexity of clinical procedures

E. a higher cost of treatment

Answer: not being able to have an anterior tooth try-in to evaluate esthetics

Q25: All of the following statements are true concerning a face-bow or a face-bow transfer EXCEPT one. Which
one is the EXCEPTION?

A. the face-bow is a caliper-like device used to record the patient's maxilla/hinge axis relationship (opening
and closing axis)

B. if the transfer is done properly, the arc of closure on the articulator should duplicate that exhibited by the
patient

C. the face-bow transfer is a maxillo-mandibular record

D. the face-bow transfer is used to transfer the maxilla/hinge axis relationship to the articulator during the
mounting of the maxillary cast

Answer: the face-bow transfer is a maxillo-mandibular record

Q26: The posterior palatal seal for a maxillary denture:

A. is placed 3 mm posterior to the vibrating line

B. is not necessary when fabricating a complete denture on a patient with a flat palate

C. is not necessary if a metal base is used

D. will vary in outline and depth according to the palatal form of the patient

Answer: will vary in outline and depth according to the palatal form of the patient

Q27: All of the following are functions of the posterior palatal seal EXCEPT one. Which one is the EXCEPTION?

A. enhances the support of the maxillary denture

B. compensates for polymerization shrinkage during processing

C. prevents food impaction beneath maxillary denture

D. completes the border seal of the maxillary denture

Answer: enhances the support of the maxillary denture


Q28: All of the following are morphological changes associated with the edentulous state EXCEPT one. Which
one is the EXCEPTION?

A. deepening of nasolabial groove

B. loss of labiodental angle

C. retrognathic appearance

D. decrease in horizontal labial angle

E. narrowing of lips

F. increase in columella-philtral angle

Answer: retrognathic appearance

Q29: In metal-ceramic restorations, failure or fracture usually occurs at the porcelain-metal interface rather than
in the porcelain.

The combination of porcelain and metal, fused together, is stronger than porcelain alone.

A. both statements are true

B. both statements are false

C. the first statement is true, the second is false

D. the first statement is false, the second is true

Answer: the first statement is false, the second is true

Q30: The strength of a solder joint is proportional to its:

A. porosity

B. thickness

C. surface area

D. all of the above

Answer: surface area

Q31: ___________ says that the combined root surface area of the abutment should equal or be greater than
that of the teeth being replaced by pontics.

A. John's Law

B. Ante's Law

C. Dawson's Law

D. Johnson's Law

Answer: Ante's Law


Q32: Endodontically treated teeth that have been restored with a cast post and core, and crown, are subjected to
the high incidence of _______. The main symptom will almost always be __________. The radiograph usually
appears _______.

A. periodontal disease, pain when drinking cold fluids, normal

B. recurrent caries, nagging pain, abnormal

C. vertical root fracture, pain when biting, normal

D. the need for an apicoectomy, a throbbing pain, abnormal

Answer: vertical root fracture, pain when biting, normal

Q33: All of the following are contraindications for porcelain veneers EXCEPT one. Which one is the
EXCEPTION?

A. traumatic occlusal contacts

B. unfavorable morphology

C. the severe imbrication of teeth

D. reduction of spacing in cases when orthodontics would be inappropriate

E. insufficient tooth structure

F. insufficient enamel

Answer: reduction of spacing in cases when orthodontics would be inappropriate

Q34: The absolute maximum number of posterior teeth, which can be safely replaced with a fixed bridge, is:

A. one

B. two

C. three

D. four

Answer: three — and three teeth should be replaced only under ideal conditions *** Any bridge replacing
more than two teeth should be considered high risk

Q35: An extracoronal metal restoration that covers only part of the clinical crown is considered to be a partial
veneer crown. Which of the following is the most commonly used form of the partial veneer crowns?

A. pinledges

B. seven-eighths crown

C. modified three-quarter crown

D. three-quarter crown

Answer: three-quarter crown


Q36: Noble alloys (gold, platinum, and palladium) do not oxidize on casting.

Porcelain adheres to metal primarily by a mechanical bond.

A. both statements are true

B. both statements are false

C. the first statement is true, the second is false

D. the first statement is false, the second is true

Answer: the first statement is true, the second is false

Q37: Periodontal health of the gingival tissues is a major concern when planning any fixed prosthodontic
treatment. For optimum periodontal health, restoration finish lines should be:

A. within the sulcus at least 1.0 mm and away from the free gingival margin without encroaching on the
biologic width

B. terminated at the free gingival margin

C. supragingival whenever possible (at least 0.5 mm from the free gingival margin) to allow for hygienic
cleansing

D. as far as possible subgingivally into the attachment apparatus

Answer: supragingival whenever possible (at least 0.5 mm from the free gingival margin) to allow for
hygienic cleansing

Q38: When casting base metal alloys for metal-ceramic crowns, which type of investment material is used?

A. silica-bonded investments

B. phosphate-bonded investments

C. gypsum-bonded investments

D. none of the above

Answer: phosphate-bonded investments

Q39: All of the following statements regarding metal-ceramic restorations are true EXCEPT one. Which one is
the EXCEPTION?

A. the metal and porcelain must have compatible melting temperatures as well as compatible coefficient of
thermal expansions

B. the metal's melting temperature should be at least 300-500°F higher than the fusing temperature of the
porcelain

C. the metal coping should preferably have sharp surfaces to prevent shrinkage of the porcelain

D. in function, glazed porcelain on the occlusal surface removes 40 times as much of the opposing tooth
structure than gold

Answer: the metal coping should preferably have sharp surfaces to prevent shrinkage of the porcelain
Q40: When preparing a tooth for either a full- or partial-veneer casting, the functional cusp bevel is an integral
part of the occlusal reduction phase. The functional cusp bevel:

A. enhances resistance form when buccal-to-lingual forces are applied

B. serves as a positive stop when the casting is seated during cementation

C. relieves the functional cusp from additional stresses when the restoration is loaded in the long axis of the
tooth

D. provides space for restorative material of adequate thickness in an area of heavy occlusal contact

Answer: provides space for restorative material of adequate thickness in an area of heavy occlusal
contact

Q41: The preparation for a full veneer crown is begun with occlusal reduction. There should be ______
clearance on the functional cusps and about ______ on the non-functional cusps.

A. 0.5 mm; 1.0 mm

B. 1.5 mm; 1.0 mm

C. 2.0 mm; 1.5 mm

D. 2.5 mm; 2.0 mm

Answer: 2.0 mm; 1.5 mm

Q42: The surface hardness, strength, and proportional limit of the metal are decreased with strain hardening,
whereas the ductility and resistance to corrosion are increased.

However, the elastic modulus is not changed appreciably.

A. both statements are true

B. both statements are false

C. the first statement is true, the second is false

D. the first statement is false, the second is true

Answer: the first statement is false, the second is true

Q43: Which of the following best describes the term "quenching"?

A. a metal is elevated to a temperature above room temperature and held there for a length of time

B. a metal is rapidly cooled from an elevated temperature to room temperature or below

C. softening a metal by controlled heating and cooling

D. none of the above

Answer: a metal is rapidly cooled from an elevated temperature to room temperature or below
Q44: Any alloy with a casting temperature in excess of _______ should be cast in an investment with a binder
other than gypsum.

A. 750°F (398°C)

B. 1000°F (537°C)

C. 1500°F (815°C)

D. 2100°F (1150°C)

Answer: 2100°F (1150°C)

Q45: The path of insertion for an anterior ¾ crown should:

A. be perpendicular to the incisal one-half of the labial surface rather than the long axis of the tooth

B. be parallel to the incisal one-half to two-thirds of the labial surface rather than the long axis of the tooth

C. be parallel to the long axis of the tooth

D. be parallel to the cervical one-third of the labial surface rather than the long axis of the tooth

Answer: be parallel to the incisal one-half to two-thirds of the labial surface rather than the long axis of
the tooth

Q46: When treatment planning for fixed prosthodontics, all of the following information can be obtained by
studying diagnostic casts EXCEPT one. Which one is the EXCEPTION?

A. the length of the abutment teeth can be accurately gauged

B. the true inclination of the abutment teeth will be evident

C. the crown-to-root ratio of potential abutment teeth

D. mesial/distal drifting, rotation, and faciolingual displacement of potential abutment teeth can be clearly seen

Answer: the crown-to-root ratio of potential abutment teeth

Q47: The tip of a pontic is usually made to contact gingival tissue. If the part of a pontic contacting the gingiva is
made from a material that is chemically active or collects debris, inflammation will probably result. Place the
following materials in order for their acceptability to the soft tissue (#1 being the most acceptable).

• glazed porcelain
• polished acrylic
• polished gold
• unglazed porcelain

1. glazed porcelain
2. polished gold
3. unglazed porcelain
4. polished acrylic
Q48: A ratio of _____ is the minimum ratio that is acceptable for a prospective abutment under normal
circumstances.

A. 3:1

B. 2:1

C. 1:2

D. 1:1

Answer: 1:1

Q49: To be biologically and mechanically acceptable, a solder joint should be so formed that it:

A. fills the entire interproximal space occlusogingivally

B. extends to the buccal margin of the retainer

C. is circular in form and occupies the region of the contact area

D. is thin occlusogingivally and wide buccolingually

Answer: is circular in form and occupies the region of the contact area

Q50: What type of pontic design looks most like a tooth?

A. the saddle-ridge-lap pontic

B. the sanitary (hygienic) pontic

C. the modified ridge-lap pontic

D. an ovate pontic

Answer: the saddle-ridge-lap pontic

Q51: Resin cements are the least soluble of all dental cements.

The solubility of the hydrophobic resins is inversely proportional to the resin content.

A. both statements are true

B. both statements are false

C. the first statement is true, the second is false

D. the first statement is false, the second is true

Answer: both statements are true


Q52: All of the following describe properties of an ideal cement Except one. Which one is the EXCEPTION?

A. long working time

B. short setting time

C. Low pulpal irritation

D. high film thickness

E. low solubility

F. low microleakage

G. high retention

Answer: high film thickness

Q53: Which of the following is often the margin of choice as it leaves adequate bulk for marginal strength with an
acute angle and produces minimal stress at the margin?

A. shoulder

B. shoulder with a bevel

C. chamfer

D. knife edge

Answer: chamfer

Q54: All of the following statements about the sprue design used in the centrifugal technique are correct
EXCEPT one. Which one is the EXCEPTION?

A. solid plastic sprues are preferred for most of the castings

B. auxiliary sprues are recommended when casting thin patterns

C. large-diameter sprues are generally recommended

D. sprues should be attached to the bulkiest noncentric cusp

Answer: solid plastic sprues are preferred for most of the castings

Q55: When using a nonrigid connector such as a T-shaped key design, the dovetail keyway in the retainer
should be place on the:

A. distal aspect of mesial abutment

B. distal distal abutment

C. mesial aspect of mesial abutment

D. mesial aspect of distal abument

Answer: distal aspect of mesial abutment


Q56: All-ceramic restorations are popular because of their superior esthetics and generally favorable gingival
tissue response. The finish line of choice when fabricating an all-ceramic crown is a:

A. 90 degree, 1.0 mm shoulder

B. bevel

C. chamfer

D. 45 degree, .25 mm shoulder with a bevel

Answer: 90 degree, 1.0 mm shoulder — to prevent unfavorable stress distribution and minimize the risk
of fracture

Q57: All of the following are indications for electrosurgery EXCEPT one. Which one is the EXCEPTION?

A. to remove hyperplastic gingival tissue where it has proliferated into preparations or over crown margins

B. in place of a gingival retraction cord where substantial attached gingiva is present

C. where attached gingival tissues are thin, or where an underlying dehiscence is suspected

D. for crown-lengthening procedures prior to fabricating a provisional crown

Answer: where attached gingival tissues are thin, or where an underlying dehiscence is suspected

Q58: Half-closed eyes can increase the sensitivity of retinal rods to better choose the "value" of the color.

"Blue" fatigue accentuates "yellow" sensitivity. This means that if you look at blue color objects (drapes, charts,
wall-color or any other object around) while selecting the shade, it will help to accentuate the ability to
discriminate yellow shades.

A. both statements are true

B. both statements are false

C. the first statement is true, the second is false

D. the first statement is false, the second is true

Answer: both statements are true

Q59: The molten alloy used for dental restorations shrink upon solidification: gold by approximately _____ and
nickel-chromium alloys by as much as _____. If the mold is not made correspondingly larger than the original
wax pattern, the resultant casting will be that much smaller.

A. .5%, .75%

B. 1%, 1.5%

C. 1.5%, 2.4%

D. 2.5%, 3.5%

Answer: 1.5%, 2.4%


Q60: A seven-eighths crown is a three-quarter crown whose vertical:

A. mesiobuccal margin is positioned slightly distal to the middle of the buccal surface

B. distobuccal margin is positioned slightly mesial to the middle of the buccal surface

C. mesiolingual margin is positioned slightly distal to the middle of the lingual surface

D. distolingual margin is positioned slightly mesial to the middle of the lingual surface

Answer: distobuccal margin is positioned slightly mesial to the middle of the buccal surface

Q61: Which type of dental ceramic is the best in mimicking the optical properties of enamel and dentin?

A. predominantly glass

B. particle-filled glass

C. polycrystalline

Answer: predominantly glass

Q62: The _______ in the composition of base metal alloys is responsible for ductility of the alloy.

A. nickel

B. cobalt

C. chromium

D. magnesium

Answer: nickel

Q63: All of the following are resins used for fabricating provisional restorations EXCEPT one. Which one is the
EXCEPTION?

A. polymethyl methacrylate

B. polyethyl methacrylate

C. polyvinyl methacrylate

D. polyacryl methacrylate

E. bis-acryl composite resin

F. visible light-cured (VLC) urethane dimethacrylate

Answer: polyacryl methacrylate


Q64: Which gypsum type has the lowest percentage of expansion?

A. type I

B. type II

C. type III

D. type IV

E. type V

Answer: type IV

Q65: In mixing dental stone, why should the powder be sprinkled onto the water in the bowl?

A. the addition of powder prevents the mix from becoming exothermic

B. this is not recommended; the water should be added to the powder

C. this process results in better powder mixing and reduced chance for air bubbles

D. the powder is added to the water to avoid using more than one bowl

Answer: this process results in better powder mixing and reduced chance for air bubbles

Q66: The main chemical constituent of dental laboratory plaster, as supplied, is:

A. calcium sulfate

B. calcium sulfate dihydrate

C. calcium sulfate hemihydrate

D. anhydrous calcium sulfate

Answer: calcium sulfate hemihydrate

Q67: Dental plaster and stone are vibrated after mixing to:

A. minimize distortion

B. reduce setting time

C. eliminate air bubbles

D. increase the setting time

Answer: eliminate air bubbles


Q68: Dental stone is produced by:

A. heating gypsum in an open vessel at 150-160°C

B. heating gypsum under steam pressure in an autoclave at 120-150°C

C. by boiling gypsum in a 30% aqueous solution of calcium chloride and magnesium chloride

D. none of the above

Answer: heating gypsum under steam pressure in an autoclave at 120-150°C

Q69: Dr. Lozier requested that you mix alginate and take an impression. While measuring the water, you got
involved in a conversation with your patient and did not notice how hot it was. This oversight will:

A. shorten the gelation time

B. make the mix unusable

C. lengthen the gelation time

D. not affect the gelation time

Answer: shorten the gelation time

Q70: Which one of the following impression materials is elastic, sets by a chemical reaction, and does not
produce by-products during the reaction?

A. polyvinyl siloxanes

B. polysulfides

C. polyethers

D. condensation silicones

Answer: polyethers

Q71: All of the following impression materials set by a stepwise polymerization reaction EXCEPT one. Which
one is the EXCEPTION?

A. polysulfides

B. condensation silicones

C. polyethers

D. polyvinyl siloxanes

Answer: polyvinyl siloxanes


Q72: Which of the following impression materials is inelastic, sets by a chemical reaction, and involves an acid-
base setting mechanism?

A. alginate

B. agar-Agar

C. zinc Oxide Eugenol

D. impression plaster

Answer: zinc Oxide Eugenol

Q73: Which one of the following impression materials is elastic, sets by a physical reaction, and is subject to
syneresis and imbibition?

A. irreversible hydrocolloids

B. reversible hydrocolloids

C. polysulfides

D. polyethers

Answer: reversible hydrocolloids

Q74: The filler in alginate gives the mixed material "body" that allows acceptable handling. Without filler, the
mixed material would be too runny for use. Which of the following is a main component of the alginate powder
and functions as the filler?

A. zinc oxide

B. calcium sulfate

C. potassium titanium fluoride

D. diatomaceous earth (silica)

E. potassium alginate

F. tri-sodium phosphate

Answer: diatomaceous earth (silica)

Q75: Alginate impressions can distort quickly if they gain or lose moisture.

Alginate impressions are wrapped in wet paper towels to serve as humidors to prevent dimensional changes.

A. both statements are true

B. both statements are false

C. the first statement is true, the second is false

D. the first statement is false, the second is true

Answer: both statements are true


Q76: Which impression material has the greatest tear strength?

A. polyethers

B. polysulfides

C. hydrocolloids (reversible and irreversible)

D. polyvinyl siloxanes

Answer: polysulfides

Q77: Which of the following types of materials is routinely supplied as an automixing system? Select all that
apply.

A. Polyethers

B. Polysulfides

C. Condensation silicones

D. Polyvinyl siloxanes

Answer: Polyvinyl siloxanes

Q78: Which of the following represents a major downfall of irreversible hydrocolloids compared to polyvinyl
siloxane?

A. ease of use

B. working time

C. dimensional stability

D. patient reaction

Answer: dimensional stability

Q79: Some impression materials are most accurate when at least 3 mm of space is present between the
impression tray and the oral tissue. Which impression material below shows this characteristic?

A. polyethers

B. polysulfides

C. silicones

D. irreversible hydrocolloids

Answer: irreversible hydrocolloids – alginate


Q80: Today was a very busy day for Ashley, the dental hygienist in our office. Ashley took alginate impressions
on her first patient in the morning, who needed a night-guard. Since she was so busy, Ashley left the alginate
impressions in the lab most of the morning. Ashley decided to place the impressions in a bowl of water so that
they would not dry up before she had a chance to pour them up in dental stone. Which of the following was the
result of Ashley leaving these impressions immersed in water for a few hours?

A. gelation

B. hysteresis

C. syneresis

D. imbibition

Answer: imbibition

Q81: If your patient indicates a tendency to gag while taking alginate impressions, which of the following
maneuvers can help make the procedure easier for the patient. Select all that apply.

A. lessening the time to take an impression

B. using cold water to mix the alginate

C. Having the patient breathe through his/her nose

D. seating the patient in an upright position

E. seating the anterior portion of the tray first

F. mixing the alginate rapidly

Answer: • lessening the time to take an impression


• having the patient breathe through his/her nose
• seating the patient in an upright position
• mixing the alginate rapidly

Q82: What is the typical level of filler (weight percent) in heavy bodied impression materials?

A. 15%

B. 30%

C. 45%

D. 60%

Answer: 60%

Q83: Custom trays are an important part of rubber base impression techniques, since elastomers are:

A. more accurate in uniform, thin layers 0.5 to 1.0 mm thick

B. more accurate in uniform, thin layers 1.0 to 1.5 mm thick

C. more accurate in uniform, thin layers 2.0 to 4.0 mm thick

D. more accurate in uniform, thin layers 5.0 to 6.0 mm thick

Answer: more accurate in uniform, thin layers 2.0 to 4.0 mm thick


Q84: Polysulfide impression materials should be:

A. poured immediately

B. poured within 15 minutes

C. poured within 30 minutes

D. poured within 1 hour

Answer: poured within 1 hour

Q85: After completing a crown preparation on one of your favorite patients, you fabricate a provisional
restoration using an PMMA acrylic resin using the direct technique. After mixing the polymer and monomer in a
1:1 ratio and placing the stint back into the patients mouth, which of the following will occur:

A. an endothermic reaction will occur during polymerization

B. excessive polymerization shrinkage will take place

C. benzoyl peroxide will act as a reaction inhibitor

D. the molecular weight of the polymer will not affect the hardness of the resin

Answer: excessive polymerization shrinkage will take place

Q86: An edentulous patient has slight undercuts on both tuberosities and also on the facial of the anterior
maxilla. To construct a satisfactory maxillary complete denture, you should reduce which of the following?

A. all undercuts

B. the anterior undercut only

C. both tuberosity undercuts

D. none of them

Answer: both tuberosity undercuts

Q87: Hinge-type or rotary motions take place in which compartment of the TMJ?

A. upper (mandibular fossa - articular disc) compartment

B. lower (condyle - articular disc)

C. both the upper and lower compartments

Answer: lower (condyle - articular disc)

Q88: Which semiadjustable articulator has the upper and lower members rigidly attached?

A. arcon articulator

B. nonarcon articulator

Answer: nonarcon articulator


Q89: You are ready to place packing cord around a tooth that was prepared for a crown on a patient with
hypertension. It is recommended to use a cord impregnated with:

A. epinephrine

B. alum (aluminum potassium sulfate)

C. zinc chloride

D. any of the above

Answer: alum (aluminum potassium sulfate)

Q90: The ________ is a thin cleft between the ________ and the ________.

A. maxillary tuberosity, hamulus, fovea palatini

B. hamular notch, maxillary tuberosity, hamulus

C. hamulus, maxillary tuberosity, hamular notch

D. fovea palatini, hamulus, hamular notch

Answer: hamular notch, maxillary tuberosity, Hamulus

Q91: The torus palatinus is a hard bony enlargement that occurs in the midline of the roof of the mouth and is
found in about:

A. 2% of the population

B. 20% of the population

C. 50% of the population

D. 75% of the population

Answer: 20% of the population

Q92: Which of the following is the most important reason for treatment of hyperplastic tissue before construction
of a complete or removable partial denture?

A. it will make the patient feel better

B. it will make the face-bow transfer easier to perform

C. to provide a firm, stable base for the denture

D. the final impression material will flow better

Answer: to provide a firm, stable base for the denture


Q93: When inflammatory papillary hyperplasia is seen on the palate of a patient wearing a maxillary complete
denture, the condition is most likely going to be associated with:

A. a vitamin B deficiency

B. a sudden increase in body weight

C. a hypersensitivity of the patient to the acrylic denture base

D. ill-fitting dentures and a poor state of oral hygiene

Answer: ill-fitting dentures and a poor state of oral hygiene

Q94: The most common cause of dry mouth (xerostomia) is:

A. aging

B. alcoholism

C. vitamin A or Vitamin B deficiency

D. medications

E. diabetes

Answer: medications

Q95: Which of the following conditions is caused by a chronically ill-fitting denture?

A. mucocele

B. myxoma

C. acanthosis nigricans

D. epulis fissuratum

Answer: epulis fissuratum

Q96: Most cases of which disease below are detected because patients complain that their dentures are not
fitting since the bone has become too large for them?

A. Addison disease

B. Paget disease

C. Hashimoto disease

D. multiple sclerosis

Answer: Paget disease


Q97: All of the following are associated with diabetes EXCEPT one. Which one is the EXCEPTION?

A. delayed healing

B. rapidly progressing periodontal disease with marked alveolar bone loss

C. mucosal bleeding

D. increased calculus formation

E. a predilection for periapical abscesses

Answer: mucosal bleeding—bleeding disorders are not associated with diabetes

Q98: After seating a new crown on tooth #19 you need to check excursive movements. You ask the patient to
slide her jaw to the right to make sure there are contacts on #19 during this movement. What muscle does the
patient use to move her jaw like this?

A. left medial pterygoid muscle

B. right medial pterygoid muscle

C. left lateral pterygoid muscle

D. right lateral pterygoid muscle

Answer: left lateral pterygoid muscle

Q99: Assume that a patient wearing complete dentures for a number of years is given an oral examination and it
is determined that the vertical dimension of occlusion has been decreased. This would cause:

A. an increased vertical dimension that leaves the teeth in a clenched, closed relation in normal positions

B. an occluding vertical dimension that results in an excessive interocclusal distance when the mandible is in
the rest position

C. an insufficient amount of interarch distance because of heavy, bony ridges

D. an inability to open the mandible because of temporomandibular joint pathosis

Answer: an occluding vertical dimension that results in an excessive interocclusal distance when the
mandible is in the rest position

Q100: The function of the compensating curve is:

A. to aid in establishing an incisal guide plane

B. the same as the function of the curve of Spee

C. to help provide a balanced occlusion in complete dentures when the mandible is protruded

D. none of the above

Answer: to help provide a balanced occlusion in complete dentures when the mandible is protruded
Q101: The centric relation (CR) is the most unstrained, retruded anatomic and functional position of the heads of
the condyles or the mandible in the ____ of the temporomandibular joints. This is a relationship of the _____ of
the upper and lower jaws _____ tooth contact. The presence or absence of teeth, or the type of occlusion or
malocclusion, _____ factors.

A. mandibular fossae/ bones/independent of/are not

B. mandibular foramen/teeth/dependent on/are

C. mandibular fossae/bones/dependent on/are

D. mandibular fossae/teeth/dependent on/are

E. mandibular foramen/bones/independent of/are not

Answer: mandibular fossae/ bones/independent of/are not

Q102: Which of the following statements concerning selective grinding in complete denture fabrication for centric
relation is not true?

A. primary centric holding cusps are the maxillary lingual cusps

B. secondary centric holding cusps are the mandibular buccal cusps

C. selective grinding of the inner inclines of secondary holding cusps can be done if there is a working side
interference

D. grind only the cusp tips of the upper buccal and the lower lingual (B.U.L.L.) cusps if they are premature in
centric, lateral or protrusive movements

Answer: selective grinding of the inner inclines of secondary holding cusps can be done if there is a
working side interference—this is incorrect, the selective grinding of the inner inclines of secondary
centric holding cusps can be done if there is a balancing side interference

Q103: Working side interferences generally occur on the outer aspects of the:

A. facial cusps of mandibular molars

B. facial cusps of maxillary premolars

C. lingual cusps of maxillary molars

D. facial cusps of maxillary molars

Answer: lingual cusps of maxillary molars

Q104: Where do the occlusal contact possibilities occur during a protrusive movement?

A. on the maxillary mesial inclines and mandibular distal inclines

B. on the maxillary mesial inclines and mandibular mesial inclines

C. on the maxillary distal inclines and mandibular mesial inclines

D. on the maxillary distal inclines and mandibular distal inclines

Answer: on the maxillary distal inclines and mandibular mesial inclines


Q105: Which of the following best describes Camper's line (plane)?

A. it is a line (plane) which is determined by the occlusal surfaces of the teeth

B. it is a line (plane) which extends from the outer canthus of the eye to the superior border of the tragus of the
ear

C. it is the line (plane) running from the inferior border of the ala of the nose to the superior border of the tragus
of the ear

D. none of the above

Answer: it is the line (plane) running from the inferior border of the ala of the nose to the superior border of
the tragus of the ear

Q106: In the intercuspal position, the lingual cusp of a permanent maxillary second premolar occludes where?

A. central fossa of the mandibular first molar

B. distal triangular fossa of the mandibular second premolar

C. distal triangular fossa of mandibular first premolar

D. mesial marginal ridge of mandibular first molar and distal marginal ridge of mandibular second premolar

Answer: distal triangular fossa of the mandibular second premolar

Q107: During non-working, excursive movements, the permanent maxillary first molar's mesiolingual cusp
escapes through:

A. the lingual groove of the mandibular first molar

B. the mesiobuccal groove of the mandibular first molar

C. the buccal groove of the mandibular second molar

D. the distobuccal groove of the mandibular first molar

E. the space between the mesiobuccal and distobuccal cusps of the mandibular first molar

Answer: the distobuccal groove of the mandibular first molar

Q108: A 22-year-old female dental student comes into your dental practice for a regular check-up. She states
that she has never had any problems with her teeth, and upon examination you notice that only one pair of teeth
seem to have contact during lateral movements of the mandible. Which teeth should ideally provide the
predominant guidance through the full range of movement in lateral mandibular excursions?

A. premolars

B. first molars

C. incisors

D. canines

Answer: canines
Q109: When establishing a balanced occlusion, the lingual cusps of maxillary posterior teeth on the balancing
side should contact:

A. the central fossae of mandibular posterior teeth

B. the lingual inclines of facial cusps of mandibular posterior teeth

C. the lingual inclines of lingual cusps of mandibular posterior teeth

D. the facial inclines of lingual cusps of mandibular posterior teeth

Answer: the lingual inclines of facial cusps of mandibular posterior teeth

Q110: In an ideal intercuspal position, the facial cusp tips of permanent maxillary premolars oppose:

A. the facial embrasure between their class counterpart and the tooth mesial to it

B. the facial embrasure between their class counterpart and the tooth distal to it

C. the opposing central fossae

D. the opposing mesial marginal ridge

Answer: the facial embrasure between their class counterpart and the tooth distal to it

Q111: In the intercuspal position, the mesiobuccal cusp of a permanent mandibular first molar occludes where?

A. mesial marginal ridge of maxillary first molar and distal marginal ridge of second premolar

B. central fossa of the maxillary first molar

C. distal fossa of the maxillary first molar

D. the interproximal marginal ridge area between the maxillary first molar and second molar

Answer: mesial marginal ridge of maxillary first molar and distal marginal ridge of second premolar

Q112: During a lateral movement of the mandible, the working condyle moves _____, _____, and _____.

A. downward, forward, medially

B. downward, forward, laterally

C. upward, forward, medially

D. upward, forward, laterally

Answer: downward, forward, laterally


Q113: In the diagram below of Posselt's envelope of mandibular motion, which line segments make up the

posterior border movements of the mandible?

A. PR-T and ICP-RCP

B. RCP-R and PR-T

C. R-T and ICP-RCP

D. RCP-R and R-T

Answer: RCP-R and R-T

Q114: Mandibular movements from centric occlusion are guided by maxillary teeth.

Occlusal interference exists if tooth contact occurs where teeth should be disengaged.

A. the first statement is true, the second is false

B. the first statement is false, the second is true

C. both statements are true

D. both statements are false

Answer: both statements are true

Q115: When the mandible is in its physiologic rest or postural position, the contact of teeth is:

A. maximum

B. not present

C. premature

D. slight

Answer: not present


Q116: Bite registration material used to make an accurate interocclusal record should have what important
characteristic?

A. offer a maximum resistance to the patient's jaw closure and have high flow at mixing

B. offer a maximum resistance to the patient's jaw closure and have low flow at mixing

C. offer a minimum resistance to the patient's jaw closure and have low flow at mixing

D. offer a minimum resistance to the patient's jaw closure and have high flow at mixing

Answer: offer a minimum resistance to the patient's jaw closure and have low flow at mixing

Q117: Which of the following materials available for recording centric relation when fabricating a removable
partial denture is the least satisfactory?

A. modeling plastic

B. wax

C. quick-setting impression plaster

D. metallic oxide bite registration paste

E. silicone impression materials

Answer: wax

Q118: An archaeologist consults a dentist about some findings he had on a dig. The teeth the archaeologist
finds have four cusps - two of them taller and pointed, two of them shorter, rounded, and dull. The dentist tells
the archaeologist that these teeth are similar to our human molars. The broader, more rounded cusps are:

A. non-supporting and working

B. supporting and balancing

C. supporting and working

D. non-supporting and balancing

Answer: supporting and working

Q119: Anterior guidance (anterior coupling) is the guidance provided by the anterior teeth when the mandible
goes into a lateral or protrusive movement.

If anterior guidance can be accomplished, the least amount of force will be placed on the posterior teeth during
lateral and protrusive movements.

A. both statements are true

B. both statements are false

C. the first statement is true, the second is false

D. the first statement is false, the second is true

Answer: both statements are true


Q120: All of the following concepts related to occlusion are true EXCEPT one. Which one is the EXCEPTION?

A. optimum occlusion requires minimum adaptation by the patient

B. bilateral balanced occlusion dictates that a minimum number of teeth should contact during mandibular
excursive movements

C. unilateral balanced occlusion or "group function" calls for all teeth on the working side to be in contact
during a lateral excursion

D. mutually protected occlusion, also called "canine guided" or "organic" occlusion is the one in which anterior
teeth protect posterior teeth in all mandibular excursions

E. mutually protected occlusion is the most widely accepted arrangement of occlusion

Answer: bilateral balanced occlusion dictates that a minimum number of teeth should contact during
mandibular excursive movements

Q121: The determinant factors of occlusion include all of the following EXCEPT one. Which one is the
EXCEPTION?

A. the temporomandibular joint

B. the masticatory muscles

C. the tongue and buccal mucosa position

D. the biomechanics of the temporomandibular joint

E. the dentition and the occlusal table

Answer: the tongue and buccal mucosa position

Q122: All of the following are considered to be the basic principles for occlusal adjustment EXCEPT one. Which
one is the EXCEPTION?

A. the maximum distribution of occlusal stresses in centric relation

B. the forces of occlusion should be borne as much as possible by the long axis of the teeth

C. when there is point-to-surface contact of flat cusps, it should be changed to a surface-to-surface contact

D. once centric occlusion is established, never take the teeth out of centric occlusion

Answer: when there is point-to-surface contact of flat cusps, it should be changed to a surface-to-
surface contact
Q123: In an ideal intercuspal position, the oblique ridge of the maxillary first molar opposes the:

A. the distobuccal groove of the mandibular first molar

B. the buccal groove of the mandibular second molar

C. the mesiobuccal groove of the mandibular first molar

D. the distobuccal developmental groove (located between the distobuccal and distal cusps of the mandibular
first molar)

Answer: the distobuccal developmental groove (located between the distobuccal and distal cusps of the
mandibular first molar)

Q124: In lateral mandibular movements, the condyle appears to rotate in the horizontal plane with a slight lateral
shift in the direction of the movement. What is this movement called?

A. Bennett movement

B. posterior border movement

C. condylar movement

D. opening movement

Answer: Bennett movement

Q125: A dental student is finalizing the temporary crown he fabricated for his patient. The patient's occlusion is in
an ideal relationship, and the crown has ideal centric contacts. The student has a bad habit of forgetting about
working and balancing contacts. He does remember the rule that he should avoid laterotrusive contacts on the
guiding cusps on posterior teeth. Which two of the following are considered to be guiding cusps?

A. maxillary lingual cusps

B. maxillary buccal cusps

C. mandibular lingual cusps

D. mandibular buccal cusps

Answer: • maxillary buccal cusps, mandibular lingual cusps

Q126: The "Glossary of Prosthodontic Terms" defines balanced occlusion as:

A. an occlusion of the teeth which presents a harmonious relation of the occluding surfaces in centric only
within the functional range

B. an occlusion of the teeth which presents a harmonious relation of the occluding surfaces in eccentric
positions only within the functional range

C. an occlusion of the teeth which presents a harmonious relation of the occluding surfaces in centric and
eccentric positions within the functional range

Answer: an occlusion of the teeth which presents a harmonious relation of the occluding surfaces in
centric and eccentric positions within the functional range
Q127: Reducing occlusal interferences (selective grinding) should usually be done:

A. after a fixed bridge or a partial denture is delivered to a patient

B. before constructing a fixed bridge or a partial denture for a patient

C. after a fixed bridge but before a partial denture is delivered to a patient

D. after a partial denture but before a fixed bridge is delivered to a patient

Answer: before constructing a fixed bridge or a partial denture for a patient

Q128: Prolonged sensitivity to heat, cold, and pressure after cementation of a crown or a fixed bridge is usually
related to:

A. recurrent decay

B. a periodontal problem

C. occlusal trauma

D. an open margin

Answer: occlusal trauma

Q129: All of the following are the signs of unstable occlusion EXCEPT one. Which one is the EXCEPTION?

A. joint pain

B. broken and worn teeth

C. muscle pain

D. mobile teeth

E. gagging

Answer: gagging

Q130: Condylar guidance is a factor which:

A. is totally controlled by the dentist

B. is totally dictated by the patient

C. is partially dictated by the patient but can be adjusted by the dentist if necessary

D. can be adjusted by the laboratory technician

Answer: is totally dictated by the patient


Q131: Decreased vertical dimension of occlusion refers to an occluding vertical dimension that results in:

A. a loss of interocclusal distance when the mandible is in the rest position (decreased freeway space)

B. an excessive interocclusal distance when the mandible is in the rest position (increased freeway space)

C. neither of the above, vertical dimension of occlusion does not affect interocclusal distance

Answer: an excessive interocclusal distance when the mandible is in the rest position (increased
freeway space)

Q132: Which jaw position is a muscle-guided position?

A. centric occlusion (CO)

B. centric relation (CR)

C. rest position of the mandible

D. none of the above

Answer: rest position of the mandible

Q133: All of the following statements concerning the functionally generated pathway technique are true EXCEPT
one. Which one is the EXCEPTION?

A. a prerequisite for the use of this technique for the restoration of a single tooth is the presence of a Class III
occlusion

B. this technique allows the cuspal movements of the dentition to be recorded in wax intraorally and
transferred to the articulator in the form of a static plaster cast

C. this static plaster cast is also called the functional index

D. by registering the pathways of the opposing tooth surfaces during mandibular movements, the technique
allows a laboratory technician to provide a restoration with an occlusal surface less likely to incorporate
occlusal interferences

Answer: a prerequisite for the use of this technique for the restoration of a single tooth is the presence
of a Class III occlusion
Q134: Regarding the occlusion of a complete denture:

(A) A decreased vertical dimension of occlusion refers to excessive interocclusal distance (increased freeway
space)
(B) A decreased vertical dimension of occlusion refers to the loss of interocclusal distance in the rest position
(C) An excessive vertical dimension frequently results in cheek biting
(D) An excessive vertical dimension is the usual cause of clicking of teeth
(E) Phonetics helps in verifying the vertical dimension of occlusion
(F) Esthetics helps in verifying the vertical dimension of occlusion

A. (A), (C), and (E) are true

B. (B), (D), and (F) are true

C. (A), (C), (E) and (F) are true

D. (B), (C), (E) and (F) are true

E. (A), (D), (E) and (F) are true

F. all of the above statements are true

Answer: (A), (D), (E) and (F) are true

Q135: In mutually protected occlusion, the centric relation coincides with the maximum intercuspation position.

There is maximum contact of posterior teeth in lateral or protrusive movements.

A. the first statement is true, the second is false

B. the first statement is false, the second is true

C. both statements are true

D. both statements are false

Answer: the first statement is true, the second is false

Q136: Dental porcelain has good biocompatibility, but is very brittle.

The compressive strength of ceramic bodies is greater than either their tensile or their shear strength.

A. the first statement is true, the second is false

B. both statements are false

C. both statements are true

D. the first statement is false, the second is true

Answer: both statements are true


Q137: How does porcelain chemically bond to metal?

A. wetting of the porcelain onto the metal surface

B. mixing of oxidized metal layers with porcelain oxides

C. interposition of an intermediate metal layer

D. mixing of the metal atoms with the porcelain structure

Answer: mixing of oxidized metal layers with porcelain oxides

Q138: A patient presents to your office after a recent cementation of a porcelain crown on #9 saying that the new
crown appears too dark compared to the adjacent teeth. The color property she is probably referring to is:

A. intensity

B. chroma

C. hue

D. value

Answer: value

Q139: Teeth that appear to be color matched under one type of light may appear very different under another
light source. This phenomenon is called:

A. opaqueness

B. metamerism

C. fluorescence

D. opalescence

Answer: metamerism

Q140: Glazed porcelain is:

A. not as durable in its surface characteristics as an over-glazed porcelain

B. nonporous, resists abrasion, possesses esthetic ability and is well tolerated by the gingiva

C. obtained by heating the previously fired body very slowly for 60 minutes at its fusing temperature

D. all of the above

Answer: nonporous, resists abrasion, possesses esthetic ability and is well tolerated by the gingiva
Q141: High-fusing porcelain is usually used for the manufacture of:

A. metal-ceramic crowns

B. all ceramic crowns

C. denture teeth

D. all of the above

Answer: denture teeth

Q142: Degassing of the metal at too high a temperature will affect the formation of the oxide layer, which is
important in bonding of the porcelain.

The number of bubbles formed at the interface increases as the time and temperature of degassing are
increased.

A. the first statement is true, the second is false

B. both statements are false

C. both statements are true

D. the first statement is false, the second is true

Answer: both statements are false

Q143: Feldspars are used in the preparation of many dental types of porcelain designed for porcelain fused to
metal (PFM) restorations. The most important property of feldspar is its tendency to form the crystalline mineral
leucite when melted.

A. the first statement is true, the second is false

B. both statements are false

C. both statements are true

D. the first statement is false, the second is true

Answer: both statements are true

Q144: Sintering of a ceramic:

A. decreases its mechanical strength

B. results in an increase in porosity

C. involves heating the raw materials above the melting point

D. increases its density

Answer: increases its density


Q145: How would you classify a removable partial denture in which a portion of the functional load is carried by
the residual ridge at one end of the denture base segment while the other end of the base segment is supported
by natural teeth?

A. a unilateral distal extension removable partial denture

B. a bilateral distal extension removable partial denture

C. a tooth-borne removable partial denture

D. none of the above

Answer: a unilateral distal extension removable partial denture

Q146: The major connecter is:

A. the unit of a partial denture that connects the parts of the prosthesis located on one side of the arch with
those on the opposite side

B. the part of the denture base which extends from the necks of the teeth to the border of the denture

C. the connecting tang between the denture and other units of the prosthesis

D. none of the above

Answer: the unit of a partial denture that connects the parts of the prosthesis located on one side of the
arch with those on the opposite side

Q147: When designing an anterior-posterior palatal bar maxillary major connector, the anterior, posterior, and
lateral straps should be about:

A. 8-10 mm wide

B. 6-8 mm wide

C. 2-4 mm wide

D. at least 12 mm wide

Answer: 6-8 mm wide — this allows for adequate strength while allowing for maximum tissue exposure
both over the palate and in the area of the marginal gingiva

Q148: Which of the following guidelines should be followed while designing the lingual bar major connector?

preferred if there is a vertical height of not less than 8 mm between the floor of mouth and the free gingival
margin

A. all of the above

B. it should have a vertical height of at least 4 mm

C. it should be located at least 4 mm inferior to gingival margin

Answer: all of the above


Q149: All of the following are advantages of using a cast chromium-cobalt alloy for removable partial dentures
EXCEPT one. Which one is the EXCEPTION?

A. resistance to tarnish

B. low material cost

C. low density (weight)

D. high flexibility

E. high modulus of elasticity (stiffness)

Answer: high flexibility

Q150: The type or form of clasp is generally selected after:

A. surveying the cast

B. talking to the patient

C. looking at the x-rays

D. the initial try-in of the metal framework

Answer: surveying the cast

Q151: Supra bulge retainers (clasps) originate:

A. above the 0.08" undercut

B. above the height of contour

C. below the height of contour

D. above the occlusal surface of most molars

Answer: above the height of contour

Q152: All of the following are advantages of infrabulge retainers as compared to suprabulge retainers EXCEPT
one. Which one is the EXCEPTION?

A. cleaner

B. less distortion of coronal contours

C. more efficient retention

D. greater adjustability

E. esthetically superior in most cases

F. less prone to caries

G. less bothersome to vestibular tissues

H. less tooth contact

Answer: less bothersome to vestibular tissues


Q153: Which of the following indirect retainers will provide the best leverage against lifting of the denture base?

A. the one located the furthest from the clasp tips which is located furthest from the edentulous area

B. the one located the furthest from the clasp tips which is located nearest to the edentulous area

C. the one located the closest to the clasp tips which is located furthest from the edentulous area

Answer: the one located the furthest from the clasp tips which is located nearest to the edentulous area
— see picture below

Q154: Which of the following Kennedy classes of removable partial dentures are tooth-borne? Select all that
apply.

A. Class III

B. Class II

C. Class I

D. Class IV

Answer: • Class III, Class IV

Q155: The following partially edentulous arch would be classified


as:

A. Kennedy Class III

B. Kennedy Class II

C. Kennedy Class I

D. Kennedy Class IV

Answer: Kennedy Class III—unilateral edentulous area with


natural teeth located both anterior and posterior

Q156: What is the recommended treatment for a patient who has lost her four maxillary incisors some time ago
and has suffered excessive ridge resorption?

A. a removable partial denture

B. no treatment

C. a conventional six-unit fixed bridge

D. a Maryland bridge

Answer: a removable partial denture


Q157: All of the following are indications for using a linguoplate as a mandibular major connector EXCEPT one.
Which one is the EXCEPTION?

A. for stabilizing periodontically weakened teeth

B. in Class I situations in which the residual ridges have undergone excessive vertical resorption

C. when the lingual frenum is high or the space available for the lingual bar is limited

D. when the future replacement of one or more incisor teeth will be facilitated by the addition of retention loops
to an existing linguoplate

E. severe anterior crowding

Answer: severe anterior crowding

Q158: It is important that a wrought wire clasp have an elongation percentage of:

A. less than 6%, allowing the clasp to bend without microstructure changes that could compromise its physical
properties

B. more than 6%, allowing the clasp to bend without microstructure changes that could compromise its
physical properties

C. more than 25%, allowing the clasp to bend without microstructure changes that could compromise its
physical properties

D. less than 25%, allowing the clasp to bend without microstructure changes that could compromise its
physical properties

Answer: more than 6%, allowing the clasp to bend without microstructure changes that could
compromise its physical properties

Q159: All of the following are true regarding indirect retention EXCEPT one. Which one is the EXCEPTION?

A. the function is to prevent vertical dislodgement of the distal extension base of a removable partial denture

B. it is located as far anterior as possible

C. it is a rest seat

D. it is usually an MO rest seat on a first molar

Answer: it is usually an MO rest seat on a first molar


Q160: Having been formed by being drawn into a wire, the wrought-wire clasp has toughness and ductility
exceeding that of a cast clasp arm. The clinical effect of this is that there is an increased capacity for deformation
of the wrought-wire without breaking.

A. the statement and the reason are true AND related

B. the statement and the reason are true but are NOT related

C. the statement is true but the reason is false

D. neither the statement nor the reason are true

Answer: the statement and the reason are true AND related

Q161: In the mesial rest, guide plane and I-bar design, reciprocation is achieved by all of the following EXCEPT
one. Which one is the EXCEPTION?

A. guide planes extended around the vertical line of abutments

B. minor connectors

C. rigid plating

D. reciprocal clasp arms

E. contact areas of proximal teeth

F. achieving balanced occlusion

Answer: achieving balanced occlusion

Q162: The primary role of partial denture rests is:

A. restore the occlusion

B. bracing

C. reciprocation

D. vertical support

Answer: vertical support

Q163: The retentive clasp arm should be passive (no active force) until a dislodging force is applied.

In a clasp only the tip of the retentive arm should be flexible, the other components are rigid.

A. the first statement is true, the second is false

B. both statements are false

C. both statements are true

D. the first statement is false, the second is true

Answer: both statements are true


Q164: _______ is the ability of a removable partial denture to resist dislodging forces during function.

A. adhesion

B. stability

C. retention

D. reciprocation

Answer: retention

Q165: A patient of yours walks into your office with the following complaint. "When I smile, my upper denture
does not hold." Which area of the denture base needs to be adjusted?

A. posterior border

B. buccal notch and buccal flange

C. labial notch and labial flange

D. distobuccal flange

Answer: buccal notch and buccal flange

Q166: Free end saddles are liable to be displaced under occlusal pressure (antero-posterior rocking around the
abutment tooth, which acts as a pivot). This is a result of the displaceability of the mucosa. Which technique is
employed to try and prevent this by taking an impression of the mucosa under controlled pressure?

A. the residual ridge technique

B. the altered cast technique

C. the functional load technique

D. the total occlusal load technique

Answer: the altered cast technique

Q167: Most designs of stressbreakers will rather effectively dissipate vertical forces to terminal abutments.
However, this occurs at the expense of what supporting entity?

A. the residual ridge

B. the alveolar support of the abutment teeth

C. the periodontal ligament of the abutment teeth

D. all of the above

Answer: the residual ridge


Q168: All of the following are advantages of precision attachment restorations EXCEPT one. Which one is the
EXCEPTION?

A. the functional load is dispersed down the long axis of the abutments by virtue of the low central loading at
the base of the attachments

B. they are easy to repair

C. they provide retention without an unsightly display of metal

D. if both sides of the dental arch have this type of restoration and are joined by a rigid major connector,
excellent bilateral stabilization is provided to the abutments

E. the restorations permit the patient access to all areas of the tissues when the denture is not in place

Answer: they are easy to repair

Q169: When surveying casts, the clinician/technician must perform an important step in order to correctly record
the path of insertion, the position of the survey line and the location of undercut and non undercut areas. Which
of the following is considered to be that step?

A. placing tripod marks on the cast to record the orientation of the cast to the surveyor

B. the use of indelible lead marker

C. the recontouring of proximal walls of abutments parallel to the path of insertion

D. all of the above steps are equally important

E. fixing the casts with screws on the surveying table

Answer: placing tripod marks on the cast to record the orientation of the cast to the surveyor

Q170: Which primary design-quality of the occlusal rest would categorize it as a "positive" rest?

A. transmit stress down the long axis of the tooth

B. prevent the movement of the appliance

C. allow no tilting of the appliance

D. have a thickness of 1.5 mm

E. form acute angles with the minor connectors that connect them to the major connectors

F. all of the above

Answer: form acute angles with the minor connectors that connect them to the major connectors — this
defines the positive rest and also permits maximum bracing
Q171: The size of posterior teeth for a removable partial denture is determined primarily by which TWO of the
following?

A. the age of the patient

B. the lip line of the patient

C. the characteristics of the denture-supporting tissues

D. the face-bow transfer

E. the useful posterior tooth space

Answer: • the characteristics of the denture-supporting tissues


• the useful posterior tooth space

Q172: A wire-gauge is selected on the basis of its active length. A short arm clasp (<7 mm) should be made in
what gauge of wire for optimum function below its proportional limit?

A. 18 gauge

B. 19 gauge

C. 20 gauge

D. 16 gauge

Answer: 20 gauge

You might also like